留言    标签    主站   
现代探索
 | 理论研究 | 技术应用 |

现代探索
>>> 返回目录   
>>> 追寻引力的量子理论 >>>


超弦——一种包罗万象的理论
超弦 2009/04/19

    英国著名科学专栏作家布赖恩· 阿普尔亚德在其《 理解现在― 科学与现代人的灵魂》 一书中有这样一段话:
    " 1 609 年,加利莱奥· 伽利略使用一架望远镜观看月亮。这一时刻,对世界的意义如此重大,以至人们将它与耶稣的诞生相提并论。因为,就像在伯利恒,自这一时刻,人类生活中的不可能成为可能。”阿普尔亚德据此将科学划分为伽利略之前的科学,或称“智慧”, 以及从1 609 年开始的现代科学。前一科学建立在推理基础上,后一科学建立在观察与实验基础上。经过如此划分,我们习以为常的科学,竟然只有400 年的历史。
    但人类就在这400 年内经历了飞速发展。
    我们有了蒸汽机,有了轮船,有了电话、电报,有了飞机、火箭,有了电视、电脑、互联网络,我们还有重力场理论、元素周期表、量子力学、相对论乃至被称为“自然中最基本物体”的超弦。工业革命、农业革命、信息革命使人类的社会生活发生了前人难以想象的变化。人类改造了自然,也改造了人类自己。回顾这一切,人类完全有理由感到自豪。因为,人类就像上帝,也有自己的“创世纪”。人说,要有科学,就有了科学。科学是好的,它行之有效。
    然而,“创世纪”中写道“到第七日,上帝造物的工已经完毕,就在第七日歇了他一切的工,安息了”。而人类的工却没有完毕,400 年后的今天仍然不能安息。
    就像有光必有影,人在发现、发明、创造、拥有上述一切的同时,还得到了原子弹、氢弹、核泄漏、酸雨、温室效应、臭氧层空洞乃至伴随科学技术而来的种种风险。
    人类曾以为已找到了通往自由王国的必由之路,他将乘着科学的飞船,摆脱一切束缚,重新确立自己在宇宙中的位置。但在科学爆炸的二十世纪,人类终于开始反思:
    科学行之有效,但它是否就是真理?
    为此,我们编辑了这套《 科学与人译丛》 ,陆续分辑推出。其中,有对信息崇拜的批判,有对生命起源的求索,有对技术所导致风险的分析,有对世界最新科学动态和研究方向的展望。数学家用对策论证明,完全的民主实际上并无可能;物理学家提出全新的超弦理论,试图统一描述所有的力、物质的所有基本粒子和时空,继量子力学和相对论之后,成为“第三次物理学革命的重要标志”… … 《 译丛》 汇集了物理学家、数学家、生物学家、天文学家、哲学家、人类学家、伦理学家… … 自本世纪后半期、尤其是在本世纪末打通自然科学与社会科学之间的隔膜,对科学这一决定人类命运的工具的深刻思索。通过这套丛书,我们期望
    
    读者可以对科学的现状、科学的未来、科学的正面与负面效应,有一个较为全面的了解,更好地认识科学、掌握科学、利用科学。中国对外翻译出版公司1 997 年2 月1 996 年8 月22 日,在本书初版一年半后,人民日报、光明日报发表文章指出:“美国科学家认为,… … 物理学领域第三次革命最重要的标志是一种全新的理论― 弦论的提出”。
    
    超弦狂想曲
    一一为《 超弦》 再版而作
    
    1996 年岁末,以《 超弦》 为代表的“科学与人译丛”以全票人选新闻出版署直属出版社优秀选题一等奖。消息传来,捧起这本“玄之又玄”的最新宇宙大统一理论,遐想联翩。
    所谓“弦”,不过是一条细线,“超弦”,不过是一条超乎寻常细小的线性结构,它小到连电子显微镜也看不见。就是这样一条超乎寻常的“弦”,把组成物质世界的各种最基本粒子在它上面安排得井井有条,对它们的各种差异和不同作用,解释得头头是道,从而填平了爱因斯坦相对论和量子力学之间似乎不可逾越的鸿沟,迎来了用一种理论统一解释宇宙的新曙光。难怪越来越多的科学家欢呼:“弦论”标志着物理学第三次革命的开始!
    以笔者愚见,最早提出“弦论”的,并非外国科学家,而是我们的老祖宗― 老子。
    “弦”字的一个解释是系在弓上的细绳,去掉“弓”字,剩下的“玄”, 也就是细绳或细线了。所以“玄”的本意就是“弦”。“弦论”即“玄论”。最早用“玄”来解释宇宙的,就是老子。
    《 老子》 开篇就提出了“玄论”,并指出这是认识宇宙变化的根本途径。老子说:“道可道,非常道。名可名,非常名。无,名天地之始。有,名万物之母。故常无,欲以观其妙。常有,欲以观其激。此两者,同出而异名,同谓之玄。玄之又玄,众妙之门。”
    老子在这里的用词,不是指通常意义上的概念(非常名)。而是用“无”表示宇宙开端,“有”形容宇宙万物。他强调在宇宙开端,要注意研究其变化(观其妙),在研究宇宙万物时,要注意各种物质引力伸展达到的边际(观其激),并用“玄”把宇宙开端(常无)和宇宙万物形成后(常有)的规律统一起来。最后,又指出玄外有玄,玄玄相扣(玄之又玄)这一“弦论”是认识全部宇宙变化的根本途径。
    现代科学的“弦论”则指出,宇宙开始形成的“大爆炸”瞬间,只适用量子力学,宇宙在发展膨胀过程中则适用爱因斯坦相对论。而“超弦”可以把这两者统一起来。
    稍加比较,不难发现,《 老子》 开篇讲的就是“弦论”,就是宇宙普遍规律,而不是一般规律(非常道)。只是人们长期不能理解“玄”的含义,就给了“玄”字一个“莫名其妙”“脱离实际”的释义。在我国古代经典中,论述宇宙普遍规律的,还有一部《易经》 。英国世界科学史学家李约瑟博士曾提到,易经中的太极图和八卦以及六十四卦图,曾在德国科学家莱布尼兹发明现代计算机原理时,给他以极大的启发。笔者由此想到,太极图能否给现代宇宙统一论以启发?例如,
    
    太极图中“阴阳鱼”上的“鱼眼”放大来看,又是一个太极图,其中的小“鱼眼”又是小小太极图… … 。这和《 超弦》 中提到的:人类生活的三维世界之外还有三维或更多维的世界的推论是否有相通之处?既然现代科学宇宙论认为“虫眼”是通往宇宙深处的捷径,太极图中的“鱼眼”是否也就是两个世界之间的捷径?也许“鱼眼”就是“虫眼”?也许… … 也许,我过分地异想天开了,也许,人们高兴得太早了。因为,你要是承认“弦论”,你就不得不承认它必然会推导出来的一系列超乎寻常的、令人难以接受的结论。其中之一就是:在人类生活的三维空间之外,还存在更多维的世界,宇宙可能是十维的。换句话说,就是天外有天,天有九重。再推论下去,中国古代神话中的“天堂”,和浮士德笔下的“地狱”岂不都有了科学根据?于是乎,“科学还是迷信?”的厉声质问,像炸雷一样把我从遐想中惊醒。
    “超弦”至今仍然是一个有争议的理论,但在本书初次出版一年半后,《 人民日报》 和《 光明日报》 于1996 年8 月22 日以“美国科学家认为以弦论为标志― 物理学迎来第三次革命”为题,报道了越来越多的世界顶尖科学家开始对“弦论”寄予极大的希望。本书这次再版,特将这两篇报道附上,以飨读者。
    最后,笔者想起了科学史上的一段往事。老伽利略在接受教庭审判时,虽然低下了头,但却喃喃地说道:“可是,地球仍然在转动着。”前不久,罗马教庭宣布对几百年前的这桩冤案予以平反。这使我鼓起勇气,用钱老学森评论特异功能的那句低沉的,然而却是坚定的话来作本篇的结束语和本书的开篇语:“科学的问题只能用科学的手段来回答!" 周轩进
    
    译者的话
    
    正如本书书名所提示的那样,关于“超弦”是否为一个包罗万象的理论这一问题在1 988 年本书原文版付梓之时尚无定论。可以告诉大家,时至今日,这一问题也还没有得到肯定的回答。尽管如此,作为近年来物理学中最活跃、最激动人心的分支之一,“超弦”理论是人们迄今为止在探索自然界终极理论这一方向上最为雄心勃勃的一次尝试,在概念和方法上均有创意。这一理论本身以及由之带来的一些新的学科分支仍处在不断发展之中。为此,我们将本书译成中文让更多的人了解“超弦”理论。
    作为一本科普读物,本书用直观浅显的语言把当今科学研究最前沿的这些动态介绍给广大的读者,帮助他们了解科学,从而激发起他们,特别是青少年读者对科学的兴趣,这无疑是有意义的。值得指出的是,科普读物有其自身的特点,在表述上多采用生动形象的语言,为的是让一般读者能够对深奥的概念、原理有一个较为直观的了解。然而,这样做的同时难免会带来不够严格不够准确等问题。为改变这种熊掌与鱼不可兼得的局面,科普读物的作者是花费了苦心的,本书采用收集科学家访谈录的方式就是这种努力的一个佐证。尽管如此,科普读物还是科普读物,一般读者在阅读时不要误认为那些有趣的概念、新奇的名词就是科学本身。科学是严肃和求实的,绝大多数情况下是要借用复杂的数学来进行表述和计算的。那些为科普读物所误、连微积分也没学而整日痴迷于离奇荒诞想法或者纯粹思辩之中而不自知的读者不是没有,而这恰l 合是与科普读物作者的初衷相悖的。当然,因噎废食是不明智的,尤其是在当今这样一个科技飞速发展的时代,科普工作更应该得到重视和加强。愿广大青少年读者在阅读时能够注意到上面所提的问题,回过头来认真打好基础,有一天真正踏入科学的殿堂,为人类文明进步做出贡献。翻译过程中对原书中的个别明显排印错误做了改正。由于译者水平有限,夕4 漏之处难免,敬祈读者指正。
    译者
    一九九三年五月于北大
    
    序
    
    在过去几年中,一个全新的非凡的理论占据了物理学家的想象。这个理论,即弦理论或更成熟的形式即超弦理论,声称能统一地描写所有的力,构成物质的所有基本粒子和时空。简言之,它是一个包罗万象的理论。这个理论的核心是认为物理世界是由一些很小的弦构成的。这听起来可能很荒谬,但理论是构筑在美妙的数学观念之上的,而且结论与现实世界是如此自洽。
    弦理论现在已经是理论物理最活跃的研究分支之一了,它吸引了很多著名的理论物理学家。一些科学家深受这个理论的前景的鼓舞,并且充满激情地宣称它可能成功。但是这个理论也不乏批判者,正如你将在书中读到的。
    1987 年,我们决定在英国广播公司三台纪实地回顾超弦研究的状况。这个节目《 寻求超弦》 在1 988 年初开始播出,内容是弦理论领头的支持者和反对者对这个论题的解释和意见。像我们以前的节目《原子中的幽灵》 (牛津大学出版社,1 986 年)一样,它是先以纪实广播形式出现,然后再编辑成书的。我们觉得完整地出版这些采访录并保存它们是很有意义的。我们将尽量保持原始纪录的对话风格,当然,为了更适于付印,我们也不得不对采访录作一些变动。
    本书旨在让物理学家和好奇的读者对弦理论的基本思想有透彻的了解。我们也希望读者通过阅读本书了解那些杰出的物理学家是怎样谈论和辩论当代的重要问题的。因为本书是面向普通读者的,我们要求被采访者尽量少使用专业词语。每一个采访都是独立的,可以分开阅读。但是为了贯穿全书,我们写了一个很长的引言,解释了一些理解本书所需的背景知识,包括相对论、量子论和粒子物理的概述。
    虽然超弦理论仍在发展之中,但理论的精要已经建立,我们希望本书将为读者提供我们时代有可能是最伟大的科学进展的一个有用的而有趣的快照。
    我们感谢伊安· 莫斯博士在文字处理上对我们的帮助,同时也感谢艾琳· 德莱堡小姐记录了录音。
    P . C . W .戴维斯J . R .布朗
    
    超弦
    
    你相信宇宙是由一些极小的弦构成,在人类生存的三维空间之外,还存在更多维的空间吗?
    近年来,“超弦”理论已成为物理学中最活跃、最激动人心的分支之一,吸引了众多著名物理学家的注意,他们声称“超弦”理论是一个包罗万象的理论,能统一地描述所有的力,物质的所有基本粒子和时空。1996 年8 月22 日人民日报和光明日报以“美科学家认为以‘弦论’为标志一物理学迎来第三次革命”为题干lJ 载文章,将“弦论”与量子力学和爱因斯坦的相对论相提并论。
    
    .引言
    
    什么是包罗万象的理论?
    
    物理学是最自负的一门科学,物理学家把理解宇宙的奥秘视为自己的职责。而其他科学家只局限于研究一些具体的东西,像生物学家研究活的有机体,化学家研究原子和分子,心理学家研究人类及其他生物,等等。像神学家一样,物理学家们不承认任何系统在原则上处于他们的研究范围之外。
    当然,物理学家也承认他们实际上对大多数系统的认识是很有限的。像云和雪花这样基本的系统就很难用物理学家熟悉的定律来构造模型。而由于一些生物系统极端复杂,即便最原始的生物如病毒或细菌,物理学家对它也一无所措。尽管这样,任何复杂系统不管多么神秘,其行为最终总是由物理定律所决定的,绝无例外。与此比较起来,那些在实践中的无能为力就显得微不足道了。
    简化主义哲学一直引导着物理定律细致入微地主宰着宇宙的观念。这个思想学派的鼓吹者,包括许多科学家,一直相信原则上心理学、生物学、化学可以依次简化,最后到物理学。换句话说,‘诊释之箭’一直射向现实的最深一层,最终任何事物都可由物质的基本组成来解释。简化主义者因而断定,假如一个关于物质的基本组成的自洽而完整的理论能够被阐明,那么,这个理论就是一个包罗万象的理论。在这里我们不准备争论简化主义的推理是否正确,但要提到最近物理学家讨论的包罗万象的理论(或丁0 〔 )正是贯彻这种精神。这种“包罗万象的理论”不是要解释所有事物,像在数学里面,几何公理并没有真正解释毕达哥拉斯(pythagoras )定理。确实,毕达哥拉斯定理可以从这些公理演绎得到,但是证明是非常繁杂的。这足我们明白,即使明确了物理世界的基本组成元素,我们也不能过分以使期望理解所有事物的复杂特性。所以,物理学家们的包罗万象的理论不能解决许多实际问题,如给我们提供云和雪花的模型的答案,更不用说生命的起源和意识的本质。但是,按照简化主义哲学,对上述现象的解释原则上可以由包罗万象的理论演绎而来。
    我们所知的第一个包罗万象的理论的例子是由公元前5 世纪的古希腊哲学家留基伯(Leuc 1 ppus )和德漠克利特(Democr 1 tus )提出的原子说,他们认为世界是由原子和虚空构成的。原子有许多种,它们是不可穿透和破坏的,因此是基本的。原子没有内部结构,它们不能说是由更小的东西组成的,它们很小,不能直接观察到,也不能在虚空中处于连续运动的状态。原子相遇,就会互相粘合构成延续的物体,而这些原子的重新排列则引起了物理世界的任何改变。
    
    17 世纪,伽利略(Ga 1 1 1 eo )和牛顿(Newton )的工作带来了现代科学的兴起。同时关于物体运动定律的认识也强烈地支持原子说。人们意识到原子的运动也遵循一些熟悉的物理定律。这些进步促使拉普拉斯( Lap 1 ace )发明了他著名的“魔力计算机”。
    
    “一种智力,在任一给定时刻如能知悉所有的自然力和构成宇宙的所有物质的瞬时位置,而且它能分析所有的有关数据,则能用一个式子描述世上最大物体和最微小原子的运动。对它来说,没有东西是不可确定的,历史和将来在它的眼前展现。”
    
    这当然是一个包罗万象的理论的尝试。
    但是,一些事情在这个可能的包罗万象的理论里面是很奇怪的。没人尝试去解释为什么宇宙是由原子构成的,我们不知道它们来自何处,也不知道它们为什么有这样的质量和形状。原子之间相互作用力的本质也是含混不清的。牛顿发现了引力,但并不能完全解释原子间的力。进而,原子通过的空间和度量它们运动的时间完全是在理论范围之外的。时空被简单地认为是存在的,但它们不是物理。总之,伽利略、牛顿和拉普拉斯的工作并不能说是已经给出了一个很令人满意的包罗万象的理论。
    直到19 世纪下半叶,当詹姆斯。克拉克· 麦克斯韦(Jamesc } erk Maxwel 帅的电磁理论补充了牛顿的力学和引力论后,这种情况才有所改变。在那段时间,人们设想所有自然力只是引力或电磁力的一些具体表现。虽然原子的存在,时空的非物理性等问题还没有解决,许多物理学家认为他们将来的任务只是把一些物理测量精确到下一个小数点,正如开耳芬(Ke } vi ' n )勋爵在1 900 年的英国科学进步协会的一次演讲中所说的:“物理中已经没有新的待发现的东西了,剩下的只是越来越精确的测量。”这就是当时对包罗万象的理论的普遍倾向。
    再回顾当时的包罗万象的理论,我们发现一个不是十分令人满意的特点,即需要假设两种基本作用力:引力和电磁力。在1 920 年,数学家西奥多· 卡鲁扎(丁heod 。:Ka }日za )设法补救了这个缺陷,他发现了两种力之间的一个可能的联系。因此如果物理学没有陷入概念的漩涡,一个可能的严谨的包罗万象的理论会在本世纪早些时候出现。电子和放射性的发现,马克斯· 普朗克(Maxp } anck )量子假说的成功和爱因斯坦(〔 instein )的相对论的出现摧毁了牛顿― 麦克斯韦时代物理的整个基础。牛顿的运动定律和他关于时空的常识性假设被抛弃了。甚至德漠克利特的原子假说也被微观世界的更细致和复杂的图像所代替,此时,原子不再是不可分的,具有确定的位置和动量的粒子。显然整个经典物理的基础已经坍塌。
    
    大约到了1 930 年,原来的经典物理已经被新的理论框架:量子力学、广义相对论和原子的更精致的模型所取代。虽然一些细节还没有弄清楚,但是物理学好像又重新回复到一些相对简单的原理上来了。原子被发现只是复合的物体,它们由少数的满足相对论和量子力学的基本粒子(电子、质子、中子)所构成。事实上,受这种高度的乐观主义的影响,早在1 923 年爱丁顿(〔 dd 1 ngton )就发表了他所声称的基本理论,一个基于神奇的数理逻辑关系之上的包罗万象的理论的尝试。在他于1946 年去世以前,爱丁顿一直在设法深入研究这种想法。同时,爱因斯坦也在用他的余年的大部分时间寻找一种用纯几何描述的自然的“统一场论”。但是现在我们知道,20 年代建立包罗万象的理论的这些希望是不成熟的。中微子假设、正电子和p 子的发现,以及原子内核子之间额外作用力的发现都给宇宙的基本定律是在电子、中子和质子水平的简单作用这个想法带来了挑战。实验粒子物理有了长足的发展,大量的亚原子碎片和令人困惑的作用力被发现了,物理学比20 年代所猜想的要复杂得多。
    这些丰富的亚原子水平的现象背后隐藏的更深层次的结构又花费了物理学家半个世纪的时间,我们多少有了令人满意的理论。最近,一些物理学家又大胆地提出一种新的包罗万象的理论。这个理论,即超弦理论是最新的也是最令人信服的一个包罗万象的理论的尝试。这个方案所处理的对象是超微观的,比原子世界或实验高能粒子物理小1 以0 倍。我们能从一个真正令人满意的包罗万象的理论中获得什么呢?首先,它必须能解释物理学家为什么可以观测到许多基本粒子而且能预言它们所有的重要性质像质量、电荷、磁矩等等。其次,它能真实地描写粒子间所有的相互作用,即它不仅能解释自然的四种基本力,而且能解释它们的相对强度。理论计算也必须能准确给出一些粒子间散射振幅,衰变率、分支比等的观测数字。简单而言,理论必须能给出粒子物理中所有已经测量的常数。另外,它能解释时空的几何和拓扑,像可感觉的维数,并给出宇宙起源的令人信服的解释。
    但是这还不是全部,一个包罗万象的理论同时要统一物理。
    
    . 2 自然核心的统一
    
    任何人都可以构造一种包罗万象的理论,他可以随意翻阅一些教科书,写下所有的基本定律,所有已知的亚原子粒子和它们的相互作用力,即需要了解宇宙的所有东西。
    这种方法有什么欠缺呢?从审美的观点来看,这样开出的清单太混杂不清了。一个好的包罗万象的理论必须不能单单是基本定律和事物的汇总,而应具有解释能力,它必须建立自然的不同侧面的联系。对这种包罗万象的理论的探索是某种程度的有信心的举动,而这种有信心的举动是建立在自然必定是简单的这个信念之上的。
    一般来说,一个科学理论所作的独立假设越少,它就越有说服力。理论通常有一些由经验确定的自由参数,更深层次的理论必须能预言它们的数值。一个具体的例子是,1880 年代的斯蒂芬一玻尔兹曼(Stefan - Bol tzman )辐射定律确立了黑体辐射的能量密度与温度的四次方的关系,比例常数由实验确定。1 900 年普朗克的量子假说成功地解释了这个常数不是自然的独立的基本常数,而是可由某些物理常数,如光速、普朗克常数和玻尔兹曼常数导出的。
    科学实验无数次表明我们越深入地探寻事物的本质,就能越多地发现它们之间的联系,同时由直接实验来确定的理论的那些随意性也就越少。现代的原子概念更深刻地表明原子如何由物质的同样的少量成分构成,所以它比原子的经典图像更先进。现今,已经没有必要假设90 种左右对应着不同化学元素的基本物体,不同原子的物理和化学性质可以直接系统地由它们的组成部分来确定。
    最终的包罗万象的理论,理想地说,是不需要参照实验的。所有事物都可以由所有其他事物来定义,仅剩下一个尚未确定的参数来定义理论适用的标度,此参数必须由实验来确定。(在这个最终的包罗万象的理论中,实验仅仅是用来定义测量的习惯,它不能确定理论的任何参数。)这种理论是基于可以导出所有事物的自然的基本原则的,这个原则设想是一个包容所有基本物理的简练的数学表式。按照芝加哥附近的庞大的粒子加速器设施― 费米实验室的主任莱昂。莱德曼(Leon Lederman )的话来说,它是一个简洁到甚至可以让你写在“文化衫”上的公式。
    统一物理的努力有两种不同的方案,第一种是可称为“自上而下”的方案,从某个可能由简单性、美妙性,而且能以简练的数学形式表达的广义的原则出发,我们可以得到对世界的描述,从而得到对特殊事例的预言。
    爱因斯坦的大部分工作是致力于“自上而下”这个方向的。广义相对论是建立在引力和惯性力的等价原则,以及物理是不依赖于描写事件
    
    的坐标系的原则上的。从这些基本的想法出发,爱因斯坦得出了引力场方程。这些方程以美妙性、简单性和紧凑性而闻名,但是,方程的解并不简单,行星运行或双星的引力波辐射非常复杂。所以,60 年后的今天,这个理论蕴含的所有现象还没有被完全揭示。
    另一个或许是更为普遍的科学研究方法是“自下而上”的方案。这种方法从现象学出发,系统地整理实验的原始数据,并推导出某些规律。这些规律被提炼成假想,推广到更普遍的定律,根据定律在新的领域对现象作出预言,再用实验验证预言。通过一点点的积累,科学家们得出了比这些事实的总和更深刻的理论。粒子物理就给这种方法提供了很多佐证,如夸克理论就是由实验所观测到的很多不同粒子的某些物理的相似性,经过逐步摸索,把它们归化为一些类或多重态。“自下而上”方法的重要性在于它总是伴随着实验的进展而发展,如果实验事实偏离了哲学上的优美,那么哲学就不会如想象的那样优美。
    物理学经历了多次统一。牛顿声称天体的运行与地面附近的物体遵从同样的动力学和引力定律。麦克斯韦统一了电和磁,并阐明了光是电磁波的一种形式,从而建立了电磁场理论和光学的联系。爱因斯坦建立了时空,能量和质量的关系,并进一步延伸到时空和引力的联系。近年来,更进一步统一自然的尝试集中于高能粒子理论,因为每当我们寻求更深层次的结构,就必须提高探测的能量。在这个框架里,主要的是两个理论,即相对论和量子论。近年来的统一物理的各种尝试的起点是这两种理论显然的不调和。在考察粒子物理之前,我们先简单地回顾一下相对论和量子论。
    
    . 3 相对论
    
    在19 世纪下半叶,牛顿的力学定律和麦克斯韦的电动力学对相对运动描述的不一致已经是很显然了。按照伽利略和牛顿的观点,在直线上的恒速运动是相对于某个参照系并且不会产生任何物理效应的,因此在平稳飞行的飞机上乘客不会对速度有真正的感受,而只能参照外部系统如窗外或地面上的物体来辨别飞机的运动。另一方面,麦克斯韦的方程预言电磁波,如光在自由空间以恒定的速度传播,光速是自然界的一个普适常数。理论没有提及测量这个速度的参照系,因此有人提出空间充满了不可见的称为以太的介质。以太可以被定义为一个绝对的静止系,而所有的运动在原则上都可以相对于它来加以测量。这好像解决了麦克斯韦的理论与伽利略、牛顿的相对性原则的矛盾。但是,实验测量地球通过以太的速度却给出了零结果。这简直太荒谬了,因为这意味着地球是静止的,而天空却围绕着它转动。
    1 905 年,爱因斯坦的狭义相对论直接解决了这个详谬。他承认相对性原则,即直线匀速运动是相对的,但抛弃了以太假说。他引入了新的假想,即光速在所有坐标系的速度是一致的,这表明不管光源或观察者的运动状态如何,观察者总是接收到相同速度的光讯号。对同一个光脉冲,两个相对运动的观察者看到的脉冲速度相同。
    爱因斯坦光速不变的新原则抛弃了时空的旧观念,牛顿对空间,时间和运动的观点被新的“相对性”理论所取代。时空的相对性概念的核心是空间距离和时间间距在运动状态不同的观测者是不同的,地球上的一个小时在一艘以不可思议的速度飞行的飞船上就可能只有半个小时,对长度也有类似的现象。但是当速度比光速慢很多时,相对论效应是不显著的。高能物理中粒子通常以接近光速的速度运行,相对论的时间膨胀效应就非常显著,如不稳定粒子在粒子加速器中运行时就比静止时寿命更长。
    时空的延长缩短对力学定律有深刻的影响。像一个运动的物体,粗略地讲要比静止时重得多,在加速时其质量会增加。因此质量的概念就有点含混。物理学家通常把物体静止时(相对于进行测量的观测者)的质量定义为质量,与惯性有关的有效质量或相对论质量依赖于它的速度,并且随着速度的不断接近光速而不断地增加。相对论中关于质量、速度和能量的研究揭示了质量和能量的等价性,即质量是能量的一种形式,它们可以相互转化。这同时也隐示着在某些情况下,物质可能会以能量的其他形式出现,反之,能量也能用来产生物质。这些观点包含在爱因斯坦著名的方程〔一mcZ 中,这里〔 、m 、c 分别代表能量、质量和光速。
    这个理论的一个结论是运动物体不能越过光障,即不能从低于光速
    
    的运动加速到高于光速的运动。从物体的质量很容易看出,在以光速运动时,物体的质量将是无穷大,因而需要无法达到的无限大的能量来加速它。
    以上的限制并不是说物体不能以光速运动,显然,光就是一个例子。以光速运动的粒子的静止质量必须是零(虽然不是实际质量,因为这样的粒子永远不会是静止的)。甚而,相对论也不排除超光速运动的可能。这种假想的粒子称为快子(tachyons )。按照相对论,它们不能以低于光速的速度运行,而它们的静止质量是虚数(即负数的方根)。如果快子的静止质量可以测量,这简直是不可想象的。但是,因为它不能以低于光速的速度运行,所以也就不存在静止质量的问题。
    虽然快子在理论上是允许的,许多物理学家还是不太欢喜它。它那比光速还快的速度隐示着在某些情况下它们会从现在向过去运动。如果它与平常物体作用,可能会向过去传播信息,因此会引起所有各种因果性详谬。人们作了一些尝试,想通过重新定义时间箭头来克服这个困难(即把从A 时向B 时运动的快子看作从B 时向A 时运动的快子),但还不清楚这种作法是否自洽。在我们讨论量子理论时,快子的概念将会遇到更大的麻烦。
    相对论预言的时空间隔的转化暗示时空是物理的一部分,而不仅仅是物理表现的舞台。事实上,转化的方式揭示了时空的内桌联系,它们可以看成是一个四度的连续体,称为时空。因而物理学家把世界视为四维,而非三维。爱因斯坦不久就意识到狭义相对论迫使我们不仅要放弃牛顿关于空间和时间的概念和力学定律,而且也必须放弃牛顿的引力论。按照牛顿的观点,引力是瞬时作用的,这表明引力效应的传播比光还快,因此与相对论矛盾。
    到1915 年,爱因斯坦完成了狭义相对论的推广,建立了新的引力理论。在原先的狭义理论中,他主要涉及的是匀速运动。如果物体加速,它的运动就不仅只相对于某些参照系。飞机中的乘客很容易感受到飞机的突然减速或倾斜。为了处理这种更普遍的运动方式,爱因斯坦假设加速产生的惯性力与引力是不可区分的。因此离心运动可以用“虚拟的引力”来形容,火箭的突然加速承受了“引力”。
    伽利略和牛顿早就注意到了加速运动和引力的等价性,但是爱因斯坦首先把它提高到了自然的基本原理的地位。当落体自由下降时,下降所引起的“引力”将抵消了它的重量,自由下落的观测者将经受失重状态。现今的宇航员对这种状态是已经很熟悉了,但是在爱因斯坦的时代,他只能想象一个坐在自由下落的升降机里的观测者。
    因为自由下落的升降机里的东西是失重的,里面的邻近物体的相对位置不会改变。所以,从地面上的观测者来看,这些物体下落的加速是相同的。是这个事实引发了不管物体的重量或构成如何,落体同时到达
    
    地面的观点。(据说伽利略在比萨斜塔用下落的球验证了这一点。)严格说,前面提到的只有在我们忽略了三种情况时才能成立。第一是空气阻力,它虽然很复杂,但只是无关紧要的因素。(更精确的实验可以在没有大气的月球上进行。)第二是升降机里面物体之间微弱的引力,当物体非常轻时,这种效应就可以忽略。那些只是用来检测引力场而相互作用不显著的物体称作检测物体。
    上面提到的忽略的第三个因素是地面的弯曲,虽然实际上这种效应很小,但对引力的理解是很关键的。为了了解这一点,请看图1 。在图中,升降机中两个检测物体自由地落向弯曲的地面。如果地球的曲率可以忽略,则它们下落的路径是平行的,因此它们之间的相互位置不会改变。事实上,每个物体下落都指向地心,因此它们的路径是逐渐会聚的。升降机中的观测者无法看到外面的景物却能够从检测物体的会聚速度来推断地球的曲率。
    
    图1 .两个邻近的物体向地面自由下落,两条路径互相逐渐会聚。
    
    图2 ( a )中4 个检测物体排成菱形。当升降机下落时,中间的一对物体逐渐靠近。最底下的物体靠地面稍近,由于地球引力与到地面距离的平方成反比,所以,它所受的引力也稍强。因此最底下的物体比邻近的物体下落得要稍稍快点。同样道理,最上面的物体下落得要慢。菱形在水平方向上被压缩,而在垂直方向将拉长,最终将成为图2 ( b )的拉长的形状。
    升降机的实验表明,需要一个抵消引力效应的参考系来验证自由下落。但是由于引力场不是恒定的,检测物体所置的位置不同,引力也会有所不同。这些引力的细微影响被称为潮汐力,因为它们在月球引力场中引起了海洋的潮汐。
    
    图2 ( a )排成方阵的四个粒子自由下落。
    图2 ( b )由于它们感受到的引力的不同,方块形逐渐变为菱形。底下的粒子由于更靠近地面,因此受的引力较大,下落更快。最上面粒子由于离地面较远,因此落得比较慢。外边的一对粒子逐渐靠近,见图1 。
    
    可能有人会指出原始引力只是相对于某个参照系,只有潮汐力才是绝对的,才代替真正的引力场。所以爱因斯坦构造了广义相对论。它的基本特征是,像图例中菱形的几何形状的改变是不依赖于检测物体的构成或质量(检测物体的质量不能太大)。这表明弯曲是物体下落空间的基本特征,而非作用于物体的力的结果。换言之,你可以看到物体在弯
    
    曲或卷曲空间的自由下落,而不是受力的作用而下落。爱因斯坦因而得出引力只是几何,空间的几何的弯曲的观点。
    让我们更仔细地解释它。首先,很重要的一点是相对论把空间和时间结合了起来,实际上,在这里有关系的是时空的弯曲而非仅是空间。(弯曲的时空可能但不一定就是指弯曲的空间。)在学校里我们学的是欧氏几何,它适用于平面或三维情况下的平直空间。而在一个弯曲表面上几何规律就有所不同,如图3 所示,在球面不可能画平行线。球面上和直线相对应的是大圆,就像经线。图中画出两条经线,它们在赤道处相互平行,但在北极相交。在这种弯曲面上,线的弯曲类似于在非稳值引力场中落体路径的弯曲。后者的不同主要在于弯曲的几何不是二维甚至也不是三维,它是三维空间和一维时间的弯曲。虽然很难观察四维的弯曲,但是数学上的描述是很方便的。
    
    图3 .弯曲的测地线。因为地球的几何不是平的,两条直线(测地线)虽在赤道平行,但会在北极交汇。这类似于引起下落的升降机中两
    
    个
    
    物体一起运动的潮汐力。
    
    爱因斯坦的广义相对论把引力场看作是时空几何的弯曲或卷曲。在这种理论里,落体不再被认为是在引力作用下的下落,而是沿内在的弯曲时空的可能的最直路径(测地线)的运动。在牛顿的引力理论里,地球绕太阳的轨道的弯曲是因为太阳引力迫使它偏离直线;而在爱因斯坦的理论中,没有这样的引力(虽然我们将继续不精确地说“引力”) , 它认为太阳在地球周围引起了时空的弯曲,而地球在这个弯曲的时空中自由地沿着测地线运动。引力效应可以确切地看成几何效应,因为它是普适的:引力可以相同地作用于所有的检测物体。因此,甚至光线在引力场中也将沿着弯曲的路径传播。图4 是实验测得的星光受太阳的引力而弯曲的图示。更大范围地,星系的分布依赖于空间的几何。
    
    图4 .光线受重力影响而弯曲。太阳引力引起了遥远的恒星A 的星光的弯曲,使得A 看上去处于B 处。这种移动可以在日食时观察和测量。
    
    可能存在宇宙范围的空间规则的曲率这一事实带来了有关宇宙拓扑的有趣问题。如果空间是平坦的,它必须是无限的或者拥有某种类型的边界。但是如果空间是弯曲的,则会有另外的可能。想象一个二维面,弯曲的面可能围成一个球体,或者一个环体(见图5 )。(请记住虽然球体是三维的,但球面是二维的。)闭合的球面的三维推广(称为超球面)
    
    )万月匕
    够想象到的。如果宇宙具有超球面的拓扑,它将拥有有限的体积,
    
    但却没有边界或宇宙边缘。现在虽然还不清楚空间实际上拥有什么样的拓扑,但是,我们将要看到,这在超弦中是很关键的。
    
    图5 .拓扑是研究线、面怎样联接的学科。球体的拓扑与环体的拓扑是不同的,环体中有一个洞。
    
    . 4 量子论
    
    相对论使我们不得不放弃一些关于空间、时间和运动的日常观念。它以抽象的图像,像卷曲时空这样很难想象的概念取代了牛顿的直观物理。量子论也同样需要对物质本质的日常观念作根本的重新评定。量子理论始于1 900 年马克斯· 普朗克关于电磁辐射以分立波包或量子存在的假设,这些量子称为光子。光子在某种程度可视为光的粒子,这与传统的观念即光和其他电磁辐射以波的形式存在是难以相容的。这个显然的矛盾由波粒二象性的概念加以解决。光按它被测量的方法不同而表现出类波动或类粒子的方面,但是,它不能同时以粒子和波动的形式存在。尼尔斯· 玻尔(N 1 el 5 Boh : )把这种事件称为单一现实的粒子和波动互补性,这种现实本身在我们的感觉范围之外。反之,电子,质子和其他亚原子物体,虽然通常被看作是粒子,在某些情况下也表现出波动性。而光子与这些物体的情况一般说来相同,所以也被看作粒子的一种。
    量子论的核心― 波粒二象性隐示着某些与亚原子物体有关的量不能很好地定义。像量子波动有确定的动量,但在空间中它是弥漫的,因此没有确定的位置。另一方面,如果能精确地测定电子或光子的位置(通过光敏屏幕),它的动量就不能确定。因此我们能由一类实验来决定物体的动量,而由另一类实验来决定物体的位置,但是这两类测量是互相排斥的,我们不能同时确定量子物体的位置和动量。总之,量子系统的行为具有著名的维纳· 海森堡(Werne : Heisenberg )的测不准原理所表示的模糊性。
    表述测不准原理的一种方法是假定所有的测量量都受不可预计的涨落的影响,从而使得测量值不精确。不能相容的物理量成对存在,像位置和动量。位置和动量的测不准或模糊性程度如分别记为△ X ,△ p ,那么测不准原理则使得乘积△ 又△ p 永远大于一个普适常数:普朗克常数h 。因此h 代表了自然的模糊程度,h 的数值很小(6 . 63 x 10 一34 焦耳秒), 也就是说虽然原则上它可用于任何系统,量子模糊性只在原子和亚原子层次才很显著。
    另一对测不准原理应用的物理量是能量〔 和时间t 。此时,△ 〔 △ t 不能小于h 。总之,测不准原理的这两种表现说明了只有在动量的测不准程度大的情况下才能有小程度的位置测不准的可能,而时间的测不准程度大可以减少能量的测不准。在很多情况下,这就是说辨别极短长度和时间间隔需要极大的动量和能量。量子论因此对每个时间和空间间隔都给予了自然的能量和动量标度,探测的范围越小,需要的能量和动量越高。所以,物理学家建造了很大的粒子加速器。同时,任何物质终极结构的理论都需要论及最小的距离和最大的能量。这些理论的高能特征是
    
    特别有趣的问题。
    因为量子论本身具有的不确定性,对电子之类的物体,牛顿力学定律不再有效(即便考虑了相对论效应),而必须由全新的量子力学来代替,它是由海森堡、欧文· 薛定愕(Erwin Schr6dinger )在20 年代发展起来的。类似地,像麦克斯韦方程之类的电磁场方程也必须由新的场量子理论取代,这方面工作从30 年代开始发展。
    亚原子粒子通常以光速运动,因此要求描述它们运动的量子理论与狭义相对论自洽,这由保罗· 狄拉克(pau 1 01 rac )在1 928 年建立起来,他的工作正确地预言了反物质。进而量子论应用于场(像电磁场),将能得到与相对论自洽的数学理论,称为量子电动力学或O 印,即便这样,理论也遇到了数学上的困难,这个疑难直到第二次世界大战后才被解决。现在,所有的物理现象的基本理论都必须与相对论和量子论自洽。
    
    . 5 亚原子粒子的世界
    
    已知的亚原子粒子有数百种。电子、质子和中子只是其中的三种,其他粒子从宇宙射线中发现或在粒子加速器中通过其他粒子的对撞而产生。除了少数亚原子粒子以外,多数粒子都是极不稳定的,通常在极短时间内衰变成其他粒子。
    一种给定的粒子的所有成员都是相同的,我们不可能区别两个电子。同时,所有粒子都有它的反粒子,除了质量相同外,所有其他量都相反。所以电子带固定的负电荷,而反电子(即正电子)带有同样数量的正电荷。
    粒子带有一些本征的物理量,其中最重要的是质量和电荷。由于某种现在还不了解的原因,所有已知粒子的电荷都是电子电荷的简单的倍数;而不同粒子的质量就没有简单的数值关系。
    亚原子粒子的另一个重要本征量是它们的内桌自旋。许多粒子具有类似于绕本身的轴转动的内部转动。然而事实上,自旋是量子力学的性质,没有经典力学的对应,它与地球绕地轴转动是不同的。宏观物体的自转的角动量可以在一个连续范围内取值,而亚原子粒子的自旋角动量是量子化
    的,即只能取咨h 的整数倍,h 是普朗克常数h 除以2 兀。为表达方便,自旋咨h 的粒子可以简单地说“自旋咨”。电子自旋为咨,光子的自旋为1 ,
    
    _、1 .、一一二。、,3 、、、、蛇杠寸目证刀一,寺寺。2
    
    内桌自旋的几何特性也非常奇特。通常自转的物体需要旋转360 。才能恢复原来的位置,而自旋奋的粒子则需要转动720 。。换句话说,自旋告粒子需要周转两次才能像自转的经典物体那样回复原先位置,好像这个粒子比我们看到的世界要大。我们的空间相对于粒子的空间是双值的,我们看到世界的两个相同模本,每一个都转动360 。,对粒子就是可以区
    分的。显然自旋咨的粒子的空间的几何与通常空间的几何有本质与微妙的
    
    区别。
    
    粒子自旋的精确值对决定它的物理性质也十分关键。自旋为告毗数倍的
    
    粒子与自旋为告崎数倍的粒子的行为极不一致,前者称为玻色子,后者称为费米子。费米子受泡利(pauli )不相容原理的约束,两个全同的费米子不能处于同一量子态。而玻色子则不遵守这一原理。
    
    物质的基本构造单元可分为两类。一类称为轻子,最普通的轻子是电子。另一种轻子p 子的质量是电子的206 倍,它不稳定,在两毫秒内衰变为电子。还有一种质量大于电子的轻子是:子,在1970 年代被发现,它很不稳定。
    除了三种带电轻子外,还可能有三种电中性粒子,称为中微子。每一种中微子对应着一种带电轻子,因而它们分别称为电子中微子,p 子中微子,和预言的:子中微子(:子中微子未能探测到)。很长的一段时间人们认为中微子无质量,以光速运动。电子中微子质量确实很小,但没有任何令人信服的理由说明为什么它没有质量。现在还没有人知道它的确切质量。
    筋中轻子者提自旋咨的费米子。它们的相到:用相对地较小,也不参与核
    
    作用。反之,核内粒子相互作用很强,除了熟悉的质子和中子,还有很多与核有关的粒子,这些粒子和其他由核作用产生的粒子称为强子。强子一般比轻子重得多。质子的质量是电子质量的1836 倍。较重的强子通常是费米子,这些费米子的强子称为重子。另外有些强子是玻色子,称为介子。质子和中子是自旋为咨的重子。最轻的介子是二介子,自旋为
    
    0 。表1 列出一些常见的强子,大部分以希腊字母命名。所有强子中,只有质子是稳定的,其余的将衰变成较轻的强子或轻子。
    强子如此之多表明它们不是基本粒子,而是有内部结构的复合粒子,这正与轻子相反,轻子一般被认为是基本的粒子。在60 年代早期盖尔曼(Ge } }一Mann )和兹威格(Sweig )猜测所有强子都是由更小的组元即夸克组成的,现在,这猜想已被实验证实。
    像轻子一样,可能存在六种夸克,奇妙地称为夸克的味道。具有的名字是上、下、桨、奇异、顶和底(或真和美)夸克。同样,像轻子一样,
    夸克是自旋为奋的费米子。
    夸克以两种方式组成强子。第一种是三个夸克的组合,按照量子力学
    
    3 一或
    
    的原理,三个夸克的自旋或平行或反平行,所以总自旋为咨
    
    称为重子。由于组成夸克的味道不同,因此不同的夸克组合对应不同的已知重子。例如质子、中子、Q 一粒子分别由两个上夸克和一个下夸克,一个上夸克和两个下夸克,以及三个奇异夸克组成。
    另一种方式是夸克和反夸克组合。量子力学原理则决定最后的产物是自旋为0 或1 的玻色子,对应着介子。因为介子只含一个夸克和一个反夸克,而重子含三个夸克,所以介子一般较轻。但因为桨夸克的质量比
    
    上、下夸克的大许多,所以由桨夸克一反夸克对组成的介子质量比质子实际上大很多。由于三个夸克组成一个重子,所以夸克的电荷量必须为基本单位(质
    子的电荷)的会或哥。这种分数电荷使得夸克在实验上非常奇特,但是一
    
    般认为夸克不能自由存在,它们被禁闭在强子中。所有由粒子对撞来分解强子的努力都失败了,并且从已知的有关夸克的内部结构来看,夸克确实是完全禁闭的。
    虽然物理学家不能研究孤立的夸克,电子射过核物质的实验为夸克的存在提供了一些间接证据。电子散射的花样显示了每个核粒子内存在三个重的微小物体。夸克存在的其他证据来自强子的衰变和高能对撞实验的强子喷注的产生等等。
    多数物理学家认为夸克和轻子代表了物质结构的最基本层次,即所有的物质都由它们构造而来。当然也可以猜测这些粒子由更小的物体构成。事实上,一些物理学家认为,作为基本粒子,夸克和轻子数目太多。(除六种夸克味之外,每一个味道的夸克有三种“颜色”,因而总共有18 种不同的夸克。)但是,承认夸克和轻子是基本的就会带来它们的存在形式的问题。
    作为基本粒子,它们不能分裂也不能通过内部的重新排列变成其他的东西。因此长期以来夸克和轻子被认为是没有内部结构的点粒子。但是,我们将看到,点粒子假设在理论上有很严重的问题,这些基本粒子看来终究还是可能有某种内部结构的。
    
    . 6 四种力
    
    在日常生活中,虽然自然界看上去好像有多种作用力,但事实上所有的这些作用力都可约化到最基本的四种。其中引力是人们最熟悉的,也是首先被牛顿用系统的数学理论所描述的。引力是唯一对任何物体都起作用的力。引力源于物体的质量,因此物体越重,引力越大。除了某些奇异的情形外,引力是相互吸引的一种力。
    引力的作用范围是宏观的,甚至是宇宙范围的,所以被称为长程力。随着距离的增大,引力按平方反比律逐渐减弱。引力的绝对表1 .一些常见的强子
    
    质量单位为Mev (百万电子伏特),电荷单位为质子电荷量,寿命单位为秒,在反粒子与粒子不一样的地方,另有记号(即粒子符号上加一杠,如反质子为歹)。
    强度很小,电子和质子之间的引力大约是它们之间静电力的10 一40 分之一,因此引力在亚原子层次的粒子物理中作用很小。尽管如此,它是自然的一种基本力,任何一种统一理论都必须处理它。
    描述力的一个重要概念是场。牛顿认为引力是“超距的作用”,换句话说,即作用从一个物体瞬时传播到另一物体。以现代观点来看,每一个物体是力的场源(此时是引力场),另外的物体,受这个场的影响,感受到了那个物体的作用。引力随距离的增大而减小,这是因为远离场源处的场的强度依次递减。
    
    
    
    名称
    符号
    质量
    电荷
    自旋
    寿命
    
    Tr 介子
    兀+兀-兀o
    139 . 57 134 . 96
    + 1 一1 0
    0 0
    2 . 6 X 10 一8 0 . 8 X 10 一6
    
    K 介子
    K + K - Ko Ko
    493 . 67 49767
    0 + 1 一1 0
    0 0
    1 . 2 X 10 一8 0 . 9 X 10 一10
    
    们介子
    月
    5488
    0
    0
    25x 1019
    
    质子
    pP
    938 . 28
    + 1 一1
    11 气乙
    > 1039
    
    中子
    nn
    939 . 57
    0
    1 一凡乙
    898
    
    (八)超子
    A 八
    1 1 1560
    0
    11 勺自
    2 . 6 X 10 一10
    
    E 超子
    艺+艺+艺。艺。艺一艺一
    1 189 . 36 1 19246 1 19734
    + 1 一1 0 一1 + 1
    1 12 11 几乙1 12
    0 . 8 X 10 一10 58 X 10 一加1 . 5 X 10 一20
    
    己超子
    它。百。它一百-
    1314 . 9 13213
    0 一1 + 1
    1 1211 八乙
    29 X 10 一10 1 . 6 x 10 一10
    
    几粒子
    几一n -
    16725
    一1 + 1
    11 ,曰
    08 x 10 一10
    
    
    
    
    1915 年爱因斯坦的广义相对论革新了牛顿的引力论。我们已在1 . 3 节中提到过,在这个新的理论中,引力场纯粹是一种时空曲率的弯曲。这种理论使引力明显地区别于其他力。
    在牛顿的引力论之后,建立了理论基础的另一种力是电磁相互作用(电磁力)。电和磁的作用很容易在实验室中观察到。虽然古代人们就发现了电磁现象,但是直至19 世纪才由麦克尔· 法拉第(Michae } 「araday )和其他人发现了电和磁之间的密切联系。尔后,詹姆斯· 克拉克· 麦克斯韦成功地以一组方程统一了电和磁的作用,这是统一自然中各种力的第一个尝试。
    电磁场的源是电荷。并不是所有粒子都带电,因此电磁作用不像引力,它不是对所有粒子都起作用;而它又很像引力,是以距离的平方反比递减的长程力。电磁作用比引力的强度要大得多,而且,因为电荷有正负两种,因此大的物体的总电磁力有可能相消,以致引力在宇宙范围内比电磁力大。
    还有两种力是亚核水平的,它们是在日常生活中不可直接观测的。其中的一种叫强核力,是把质子和中子约束在核内的力。在10 一’5 米范围以外,强相互作用(强力)迅速递减,因此它是短程力。它的性质与长程的引力或电磁力的性质相差极大。不仅质子和中子,而且所有的强子都受强作用力的作用,而轻子则没有。
    强子间的力很复杂,这是因为强子不是基本的粒子而是夸克的复合体,夸克间的力才是基本的相互作用。这个力与电磁力很像但比它大很多。重子中由于三个关联的夸克相互作用而不像电磁力是二体力,因此荷的问题处理起来很复杂。对照于只有一种电荷的电磁力,强力有三种荷,称为“颜色”,记为红、绿和蓝。
    最后一种力称为弱相互作用(弱力)。它作用于所有的夸克和轻子,强度比电磁力小得多,但远远大于引力。弱相互作用主要的作用是改变粒子而不对粒子产生推和拉的效应。它首先是用来解释p 衰变,即一种不稳定核子的放射性效应。在典型的p 衰变里,中子衰变成质子、电子和反中微子。这种由弱相互作用引起的过程改变了中子的夸克味,使一个上夸克变为下夸克。弱相互作用能改变夸克和轻子的味。在后一种情况中,电子可以变为中微子,等等。
    中微子只经受弱相互作用(当然也有引力),因此它们的相互作用很弱。事实上,它能在固体铅中穿越好些光年的长度。中微子的冲击可以在星体的剧烈的死亡过程中被观察到。平均每个星系在几十年内都会有星体的爆发,称为超新星。几个世纪以来,已经观察到很多的实例。最近的一次是1 987 年春天发生的大麦哲伦星云(一个附近的小星系),从地球上能看得见。
    超新星爆发的第一步是星体的核在自身重力下突然塌缩。在这个内
    
    部破裂的过程中,释出了大量的中微子,它们的极强的冲击力炸开了星体的外壳,把大量的物质抛入太空,从而形成一个膨胀的气体云。在1 987 超新星爆发时,中微子冲击力在视觉看到爆发的前几个小时已经在地面上被观测到。
    弱相互作用范围很小。在这个力最早被发现时,曾假设它是接触性的作用,但现在通常认为它的作用范围是10 一’7 米。
    
    . 7 媒介粒子
    
    我们已经讨论过,亚原子粒子的行为必须用量子论处理,任何在亚原子水平上对力的描写因此都必须与量子论自洽。量子论的起始点是普朗克关于光是分立的波包或量子,即光子的假设。电磁波是以光子形式在空间中传播的,它具有粒子性。光子既不是夸克也不是轻子,它属于第三类粒子的家族。
    在1 . 5 节中我们谈到,虽然强子中既有费米子又有玻色子,但夸克是费米子,类似地轻子也是费米子。因此,物质的基本粒子都是费米子。光子与它们不同,它的自旋为1 ,是玻色子。光子的静止质量为零,按定义它以光速运动。
    在讨论电磁力的作用时,必须讨论光子的存在。图6 中是经典图像中两个带电粒子的运动路径。粒子相互接近时,A 粒子的电磁场作用于B 上,排斥力使B 的路径偏折,反之,B 对A 的影响也是如此。在这个称为散射的过程中,粒子间交换动量和能量。
    在量子论中,能量和动量是量子化的,即它们不是连续变化的,而是仅取某些分立的值。因此,图7 的效应的量子解释与图6 有些不同。两个粒子不再是通过电磁场的连续的能量和动量流,而是以交换光子的形式相互作用。图7 中是单个光子的交换,波纹线代表光子。由于测不准原理,光子路径的方向是不确定的,发射和吸收光子发生在不确定的时间间隔△ t 内。从这个观点来看,光子是一种在带电粒子间传播电磁力的媒介粒子。物理学家称,像电子这种带电粒子与光子藕合,从而引起了电磁效应。
    双光子交换也会发生,但是对散射效应的贡献比单光子交换小得多。三个、四个光子交换过程更弱。图6 、7 是按照里查德· 费曼(Richard Feynman )命名的,称为费曼图,而其中的理论称为量子电动力学。对散射和其电磁效应的详细计算证明非常成功,并与实验结果惊人地一致。自然力的所有力都可以这种方法处理。每种力都有它的媒介粒子。引力中光子的类似物称为引力子,它也是一种无质量的玻色子,但自旋为2 。引力子与其他物体的藕合极弱,它们的效应还有待实验证实,但是,非直接的现实和物理的自洽性都预言了它的存在。光子和引力子的无质量与电磁力和引力的长程性密切相关。
    引力子与光子的另一个重要区别是光子与带电粒子藕合,而引力子与所有粒子藕合,包括引力子。就是说引力子也受到重力
    图6 .在经典物理中,带电的粒子的相互作用是由动量的连续传输而引起的。
    图7 .两个带电粒子如电子的排斥力可以从它们之间的光子传递算出。
    
    图8 .引力子相互作用的费曼图一例。
    
    的作用,而且相互作用,如图8 所示。这样的图表示引力是非线性理论,即引力不满足线性叠加律。在线性理论如电磁理论中,光子束可能相互交错,但是没有相互作用。引力的非线性本质使得它的量子化理论很难建立(参阅1 . 12 节)。
    我们谈到过,强作用力类似于电磁相互作用,但它有三种荷,称为颜色,为解决这种复杂性,需要至少有八个媒介粒子。它们被称为胶子,自旋为1 ,是强相互作用的场量子。但是,像引力子一样,胶子之间有相互作用,即胶子像夸克一样带有颜色。这种理论称为量子色动力学(或OCO ) ,是非线性的。夸克之间的力因此与距离的关系与通常情况很不一样。大多数力的强度随距离的增加而减小,但胶子作用力正好相反。在短距离内或高能范围力很小,但当夸克分开时,它们之间相互作用就会加强。这很类似于弹性弦,且力随距离的增大而增大是没有限度的。如果是这样的话,夸克就会永远被束缚在强子内。
    最后,弱作用力也有它的媒介粒子,称为W + , W 一,Z 粒子。它们的自旋为1 ,但与其他的媒介粒子不同,它们的质量不为零。实际上,它们的质量很大,W 、Z 的质量分别是质子质量的80 和90 倍。媒介粒子的质量不为零这一事实说明弱相互作用是短程的力。Z 粒子与光子除了质量以外非常相似,W 粒子是带电的,W 一实际上是W 泊勺反粒子,它们的电荷数值都是一个单位的电子电荷。
    
    8 对称和超对称
    
    虽然研究对称性所需要的高等数学是在本书范围以外的,但是它的基本概念是不难掌握的。为明确概念,想象一些简单的几何图形:正方形、等边三角形、圆(见图9 )。每个图形都有丰富有趣的对称性质。试着在一本书的破折线上垂直地放一面镜子,镜中图像的形状不会改变。在每种情况下,左边的图形都是右边图形的反射。对此,一种有趣的表达方法是:这些图形在破折线轴的反射中是不变的。注意反射对称性的轴的个数对不同的图形是不同的:正方形有四个,等边三角形有三个,而圆形有无穷多个(把镜子置于任一条直线上)。
    
    图9 .几何对称性的例子,每个图形按破折线反射不变。
    
    我们在这些图形中可以发现其他的对称性。如果三角形按中心的点被转动120 。,240 。,360 。,图形不变。正方形有四个不改变其形状的转动角:90 。,180 。,270 。,360 。。可以说它们在以120 。和90 。的倍数角度转动下不变。圆按圆心转动任何角度都不改变其形状。对称性因此可以是连续的,也可以是分立的。圆的转动是一个连续的操作,任何时候都不改变其形状。另一方面,正方形和三角形的转动及反射对称是分立的。
    用什么来表明圆比正方形的对称程度要高是一个有趣的问题。我们注意到正方形比圆形的结构复杂。当把圆稍微变一变形,或在圆上添加一个点时,转动对称性就会被破坏。在这两种情况下,都是在圆上面附加新的结构。通常一个结构简单的系统有更高的对称性。
    结构最简单的系统可能是空的空间。转动它时没有什么会改变,在给定方向的移动也是如此。我们可以说空的空间在所有的方向和位置上都是一样的。(忽略了引力的弯曲效应,在粒子物理中完全可以这么假设。)同时,在镜象反射下空的空间是不会改变的。这一基本的观察可以由于空间的几何结构(距离和角度)在连续的转动和平移和在任何平面的反射下是不变的这种说法而表达得更为精确。
    在这个无结构、空的世界中,时间也具有对称性。在没有事件发生的虚空中,任何时刻都是等同的,也就是说有时间平移不变性。同时时间反演不变,即没有区别过去和未来的时间箭头。
    实际上的世界当然不是空的,它充溢着各种场,粒子和不休止的量子作用。空的空间的对称性被破坏,或许它们将是近似对称的。如在太阳系中,不是所有的方向都是等价的,向着太阳的事物与背着太阳的事物截然不同。但是在许多情况下这种偏离是不重要的,可以忽略。假设一个处于外层空间的孤立粒子,这个粒子可以是一个台球或者
    
    原子(假设忽略量子效应),同时,假设像太阳和其他粒子与它距离太远,以至不能对它的行为产生影响,而且任何场的作用都是很微小的。在没有力的作用下,粒子是不会古怪地向某一方向运动的。我们假设某些外部的力被忽略了,在没有力的情况下,粒子肯定是不能运动的。这种想法的基础完全是我们对空间在平移下是对称的猜测。如果空间的某一部分与其他部分一样,为什么粒子会选择某一特定方向运动,而不选择其他方向呢?
    类似的推理可用于转动。不能想象,不外加作用物体会突然转动。为什么它顺时针而不逆时针转动呢?此外,物体以空间某一特殊的方向转动为轴转动。如果空间在转动下对称,就没有特殊的方向,所以物体不可能自发地转动。
    这些粗略的讨论可以用精确的数学语言描写。数学理论可以提供空间几何对称性与物体动力行为的关系。事实上,不存在自发运动对应着动量和角动量的守恒定律。空间平移不变直接导出粒子的动量守恒,转动对称性则导出角动量守恒。除此之外,能量守恒可以看作是时间平移不变的结果。因此物理中最基本、最深刻的定律是源于空的空间没有结构这样一个基本而平凡的事实,它表明对称性维持着自然世界的秩序。所有的力是否都遵从空间和时间的几何对称性?麦克斯韦的电磁理论和引力显然都满足我们刚讨论过的对称性。在很长一段时间里,物理学家假设核力也满足这些几何对称性。当然,如果在亚核水平,能量、动量和角动量不再守恒,就会带来新的问题。
    物理定律怎样由分立的几何对称性描述呢?假如放映一部关于某个自然过程的影片,并投影到镜中去,我们能发现任何不同吗?我们能在镜中看出任何现实中不能实现的过程吗?而且如果影片从后向前放映,我们能观察到任何违反物理定律的过程吗?
    举个简单的例子,假如影片是关于一个转动的球体(见图1 助。转动轴定义了一个特定的方向,我们画根线作记号,在镜子中它的手征改变了,即转动方向相反,我们看到了一个以相反方式转动的球。但是自转的方向是没有什么特别的,所以镜中的转动与外面的转动有同样的可能存在。当然如果这个球体是地球,镜象的转动是不可能的,因为这时太阳将由西至东运动而不是由东至西运动。但是,我们讨论的是物理定律的对称性,而不是现实世界的某个大的物体。在亚原子世界,不可能用地球上的“大陆”来区别两种粒子,因此这种偶然的复杂性是不存在的。
    转动球体的例子也解释了时间反演对称性,倒着放映的影片中转动倒转。从过程本身我们不能判断它是正着或倒着放映,因
    图1 0 .反射对称性。转动球体在现实世界中顺时针转动,在镜象中逆
    
    时针转动。如果我们不能识别镜子,我们不能区别哪一个球是实体,哪一个球是镜象。两者都是同样可能的过程。镜象中过程在转动球体的影片倒映中也发生。
    
    为球体没有特征,看起来两种都是可能的。同样,在日常生活中我们很容易判断像高楼塌毁,水珠溅射,绘画这样过程的倒着放映的过程是不可能发生的。但是在微观世界中,转动倒转没有什么特别的地方,同样,粒子碰撞和分解的时间反演也不是很奇怪的。只是当很多粒子共同活动的过程倒转了,我们才会怀疑时间反演对称性的正确与否。例如,中子自发分解的质子、电子和反中微子的过程如倒过来看,我们将看到大量的不可能的质子、电子和反中微子的复合。
    日常生活中的对称性虽然可以以许多方式存在,但是在几何上特别显然(见图9 )。时间对称性已经讨论过,还有一些不是直接与空间和时间有关系的,在物理上非常重要的对称性。一个简单的例子是电荷反演的分立对称性。电子和反电子可以看作是互为镜象的粒子,同时正电子也可看作是电子的电荷反演。它们的电荷数值的相等隐示着电荷反演是精确的对称,因此我们猜测物理定律在电荷反演下不变。
    从一些非常弱但又没有人怀疑的假设出发,有一个非常有趣的数学定理,证明了物理定律在空间反演,时间反演和电荷反演这些操作联合作用下不变。这些操作分别记为p ,丁,C 。这个定理称为C 盯定理。在50 年代中期,物理学家在强子的弱衰变的某些过程中遇到困难。两个美籍华人,李政道(Tsung Dao Lee )和杨振宁(chenN 1 ng 丫ang ) 为了解决这个问题,大胆地提出了弱作用可能破坏宇称守恒的观念。虽然那时几乎所有人都假设宇称守恒,但奇怪的是没有人对弱作用作过实。验随之,另一个美籍华人吴健雄(Chien 一Shi 日ngw 日)的实验证明了弱相互作用确实破坏宇称守恒。
    吴健雄的实验包括测定从钻60 核发射的p 衰变粒子的方向,实验的目的是确定这个方向与钻核自旋轴的相对关系。如图11 ,按惯例图中自旋矢量方向与转动成右手螺旋关系,吴健雄发现在与钻核自旋方向相背时,发现电子的可能性更大。在镜中看,正好相反,所以衰变确有偏好的手征,镜象中过程是不可能的。实验直接证明了p 衰变中宇称的破坏。宇称不守恒是弱相互作用的普遍特征。一个简明的例子是p 一衰变的电子e 一和中微子。虽然中微子不能直接观测,但是p 子自旋方向和电子运动方向是很容易测量的。我们发现,虽然电子可以在相对于p 子自旋轴的任何方向发射,但是电子更可能在p 子自旋
    
    图1 1 .宇称破坏。吴健雄的实验证明了Co60 核p 衰变的粒子倾向于在核自旋矢量方向一侧发射。在镜中这个不对称性正好相反。
    
    图12 .当p 一衰变时,发射的电子倾向于图中自旋轴的右侧(图中略去中微子)。显然它的镜象是不对称的,镜象中电子取左侧。这正是反粒子的衰变过程的取向。
    
    看起来是顺时针的一边发射。
    在图12 中,右边是衰变过程的镜象反射。镜中p 子自旋倒转,所以镜中电子更可能在p 子自旋看起来是反时针的一边发射。因此镜象反射改变了p 子自旋方向与发射粒子运动方向之间的关系。类似的分析可用于反粒子过程,即带正电荷的p 子(u + )衰变为正电子(e + )。我们发现的情况正好相反,这种过程的方向取舍正如图12 中的镜象过程,而且这种发射方向的不对称性与图12 中镜象过程完全一致。这种结果与物质和反物质的对称性一致,它隐示宇称反演p 和电荷反演C 的联合过程cP 中p 子衰变过程是不变的。因此,即使宇称p 被破坏,cP 也保持对称性。弱相互作用中宇称不守恒的发现对物理学家来说是一个非同寻常的结果。虽然宇宙中某些复杂结构具有特定的手征性(如ONA ) ,物理定律中优先手征的存在则具有更深刻的涵义。它意味着即使没有复杂结构,自然也能分辨左、右手。物理学发展的历史表明,每当有新的数学发现,总会带来物理的巨大的进展。特别是在对称性的研究中,虽然数学描述的对称性很难想象,但它们能给我们提供对自然的新的重要原则的认识。探索新的对称性已经成为物理学家认识世界的一个重要手段。到这里为止,我们已讨论的连续的对称性都涉及空间或时空。但是具有更抽象性质的连续对称性也能被发现。我们知道,每个守恒定律总是对应着一种对称性,那么电荷守恒定律背后的对称性是什么呢?电荷有两种:正电荷和负电荷。电荷守恒定律指出正电荷的总数量减去负电荷的总数量是一个不变量。如正电荷和负电荷相遇,则两个电荷相互抵消。同样,正电荷可以伴随同样数量的负电荷产生。但是任何电荷的净增或净减都是不可能的。
    电荷如果守恒,有人会问是否存在导致这个定律的对称性。各种动力学守恒定律,如动量和能量守恒定律是与连续的几何对称性相联系的。电荷守恒定律涉及抽象的性质而非动力学的性质,它对应着某种抽象的对称性。作为一个日常生活中抽象对称性的例子,可以考虑通货膨胀的问题。当英镑或美元贬值时,一个拿固定收入的人的财产则随之贬值。但如果这个人的收入是与经济指数相关的,即他的收入随货币贬值而增加,则他的实际收入与货币的价值是无关的。一种学究气的说法是与指数有关的收入在通货膨胀变化下是对称的。
    物理中有很多非几何对称性例子。一个例子是提举重物作的功。这个功只依赖于重物所提升的高度差,而不依赖于提升路径,也不依赖于
    
    绝对位置,即在海平面和地面把重物提升同样的高度差所需作的功相等。因此,存在一种自由选择起点的对称性。
    电场中也存在类似的对称性,这里电压类似于高度。在电场中如果电荷从一点移到另一点,所需的能量只依赖于移动路径上两点之间的电压差。如果在这两个点上电压增加的量相同,需作的功相等。这是麦克斯韦电磁方程的一个重要对称性。
    上面的三个例子是物理学家所称的规范对称性。这些对称性可以看作是分别重新标度钱币、高度和电压。它们不是几何的,而是抽象的对称性,我们不能直观地观察到这种对称性。但是它们仍是有关系统性质的重要标记。的确,正是电压的规范不变性,保证了电荷的守恒。规范对称性在寻求自然中各种相互作用的理想的量子理论中起着核心的作用,而且统一力的尝试都是在规范对称性的范围之内的。
    我们已经看到,物理对称性有两类,即几何对称性如转动和反射,和抽象对称性如规范对称性。在70 年代早期,理论物理学家发现了一个未知的比像转动和平移这种操作更深刻,更有效的几何对称性,它就是超对称。
    在1 . 5 节中已经提到,费米子所感受的空间的几何结构与玻色子所感受的空间的几何结构有着本质的差别。费米子必须转动720 。才能回复到原先的位置。这种双值性决定了它的几何对称操作如转动的代数与玻色子完全不同。玻色子和费米子如此不同的一个原因就是它们的几何特征完全不同。
    超对称的新的特点是它提供了一个可以同样描述费米子和玻色子的框架,这在通常空间的几何操作的框架内是不能处理的。在超对称理论中,在通常的四维时空上附加另外的四维,称为超空间,目的是为了容纳费米子奇异的几何性质。因此这些附加的“费米维数”不是如我们所知的空间或时间的维数。
    附加维数的几何规则是非常奇特的。以转动作为一个例子,很容易证明,如果连续作两次转动,则结果是依赖于两次转动操作的次序。图13 中的书作两次90 。转动,两种情况下最后书的取向是完全不同的。如果两个转动分别记为Rl 和RZ ,则两种情况的差别可记为Rl RZ 半RZRI ,或Rl RZ 一RZRI 半0 。式子Rl RZ 一RZRI 称为Rl 和RZ 的对易子。从这些对易关系出发,我们能建立描述书或玻色子所处空间的转动代数。同样也能类似定义在附加的四个费米子维数中的转动R 。但是,由于这部分超空间必须描写内桌自旋的奇特的几何’!生质,所以我们处理的是反对易子Rl RZ + RZRI ,而不是对易子。看起来很随便的一个符号替换却在数学描写上有极大的区别。自洽地结合这两种数学理论,我们就得到了玻色子和费米子的统一的描写方法。超对称操作能从通常空间转到附加的费米维数,即能把玻色子变为费米子,把费米子变为玻色子。所以我们可以把费米子
    
    和玻色子看成一个几何存在的两个不同的投影。
    直此为至,超对称只被看作一种数学性质。我们想知道现实世界中是否有超对称。如果世界是超对称的,我们将期望能看到费米子和玻色子的直接的物理联系。例如每个已知的费米子都有对应的玻色子,每个已知的玻色子都有对应的费米子,即每个粒子都有其超对称伴侣。从现在已知的玻色子和费米子来看,这种对称性是不成立的。
    
    图13 .转动互不交换。在上边过程中,把书沿垂直和水平轴作90 " 转动,在下边的过程中,次序颠倒,两次过程结果不一致。
    
    但这并不意味着超对称与现实世界无关。首先,在物理中更深层次的对称性经常会在某些系统的物理态中破缺。这种对称性破缺在电弱作用理论中是必然的,而力的更深层次的对称性是隐藏着的。因此有可能自然本身是超对称的,但是在现今所考察的现象中是破缺的。
    其次,没有理由认为已知的费米子是已知的玻色子的超对称伴侣,很可能存在着已知粒子的尚未发现的超对称伴侣。我们可以假设存在光子的超对称伴侣,但是它与通常物体的作用极弱,所以还没有观测到。类似地,我们有胶子、引力子的超对称伴侣,有费米子、超夸克和超轻子。现今这些奇怪的超对称伴侣都是假设的,因此超对称是一个理论上强有力的观点,但缺乏实验证明。
    
    g 力的统一
    
    当麦克尔· 法拉第在1830 年代发现电磁感应现象时他就在自然界的两种相互作用力― 电和磁之间建立了一种明确的联系,虽然直到1850 年代由于麦克斯韦在数学上的洞察力才给出一种完全统一的电磁理论。尽管如此,在1 850 年法拉第就进一步猜测在电与引力之间或许存在某种联系。为探究这一想法是否真实,法拉第制作了几件巧妙的装置,它们可以用来检验比如说落体是否产生电场。图14 的漫画说明了这一装置的大致原理。法拉第的实验结果是否定的,但这并没有打消他的信念,即深入下去,电力和引力是某种普遍的超力的构成部分。
    随后关于统一引力与电磁作用的认真尝试是在1 921 年。那时爱因斯坦发表他的引力理论― 广义相对论还不过几年。正如在第1 . 3 节介绍的那样,这一理论的一个重要特点是空间和时间统一成为一个四维时空。受此启发,德国数学家西奥多· 卡鲁扎决定,作为一种简单尝试,通过附加一维额外的虚空间自由度来写出五维而不是四维的爱因斯坦引力场方程。这一尝试出人意料地富有成果。在通常的四维时空中看,这一五维的爱因斯坦场方程不但给出了通常的四维引力方程,同时还给出另外一组方程,而它们恰好就是电磁场的麦克斯韦方程组。所以,通过在五维时空中表述引力人们可以从一个单一的理论中同时得到引力论和电磁理论。换句话说,按照卡鲁扎的理论电磁作用原本不是一种单独的力,它不过是引力的一个方面,尽管是在一个有着不可见的更高维空间自由度的世界里。
    
    图14 .法拉第试图用这个漫画来揭示电力与引力之间的联系(该图由A .德· 鲁米拉提供)
    A :小球B :垫子C :阻流圈O :电流计〔 :皇家学院「:M .法拉第S :绳子
    
    这一理论的主要弱点在于:事实上现实世界中我们只感受到四维自由度。所以,如果要认真看待五维时空的想法,就有必要解释什么是第五维自由度。1926 年瑞典物理学家奥斯卡· 克莱因(oscarK } ein )对此给出了一个非常简洁的回答。克莱因认为我们没能察觉到那一维额外自由度的原因是,在某种意义上它“卷缩”到一个非常小的尺度。这情形好比软水管。从远处看,水管就像一条弯弯的曲线。一旦走近细瞧,才发现我们先前认作曲线上的一点原来竟是绕管子圆周的一个环(图15 )。按克莱因的猜测,设想我们的宇宙就类似软水管。我们通常当作三维空间的一点其实是绕第四个空间轴的一个小环。从空间中的每一点衍生出一个小圈,它的方向既非向上向下,或向左向右,也非我们所能感觉的
    
    空间的任何其他方向。我们没有察觉到这些小圈― 这使得克莱因能自圆其说― 的原因是由于它们的周长小到令人难以置信的地步。
    
    图15 .从远处看一根软水管就像是一条线。一旦走近就会发现线上的p 点却原来是绕管子的一个圆环。极有可能三维空间中我们通常当作的一点实际上是绕另外一个空间自由度的小环。这种想法构成了卡
    
    鲁
    
    扎― 克莱因电磁与引力统一理论的基础。
    
    对克莱因的想法要花些力气才能习惯。这里的部分困难在于很难具体设想那些小圈是在什么地方环接起来的。这些圈并不在我们的三维空间里,它们将空间扩展,正如一条波浪线刚性地绕某一个圈移动会形成一根管子。在二维空间我们很容易设想这一情形,但在四维却不能。尽管如此,这一想法仍是有意义的。克莱因能够从电子或其他粒子所带电荷单位的已知值以及这些粒子之间的引力强度计算出那些绕第五自由度的小圈的周长。这一周长的值为10 一30 厘米,或约为一个原子尺度的10 - , 7 倍。无怪我们迄今未能察觉这一假想的第五自由度,因为它如果存在的话也将卷缩到一个比我们目前能够分辨的(甚至在亚核粒子物理中的)任何结构要小得多的尺度。
    虽然卡鲁扎一克莱因理论构思精巧,但在50 多年里它始终仅仅是一种数学上的奇思异想。随着1 930 年代弱和强相互作用的发现,统一引力与电磁作用的设想已远非当初那么诱人。任何成功的统一场论都将不得不容纳四种力而非原来的两种。因此,在科学家们对有关弱和强相互作用有了恰当的了解之前,统一是不可能实现的。
    在1 950 年代开展起来的关于亚核粒子及其相互作用的研究揭示出一种令人困惑的复杂景象,它阻碍任何简单的统一。事实上,在已知的四种力中,只有电磁作用存在一种理论(量子电动力学)既内部自洽又与相对论和量子力学这些十分重要的理论相协调。
    其他三种力在当时均未得到人们的很好认识。宇称不守恒的发现要求对弱相互作用的理论进行全面彻底的检修以便使它的作用量中可以包含左右不对称项。这一点做到了,但相应的理论只对一些简单弱作用过程给出有意义的结果并且过程涉及的能量尚不可太高。对于大多数过程来说理论计算的结果毫无意义。所以这一理论在数学上不自洽,缺乏预言力,因而明显地存在根本性缺点。
    强相互作用在当时则全然没有被了解。确实,强子之间的相互作用那时看上去似乎涉及除单一的强核力外不同场与力的复杂纠缠。现在,我们知道强子之间的相互作用事实上只是更为基本的夸克之间的相互作用的一种复杂的残余表现,但在早期人们试图把强子间的力看作是基本
    
    的相互作用。早在1935 年日本物理学家汤川秀树(Hideki Yukawa )通过类比量子电动力学给出了强作用的模型,其中引入了在强子之间传递强相互作用的“媒介”粒子从而导致了对二粒子的成功预言。尽管如此,很快就发现这种交换二粒子的模型只能给强核力一种粗略的描述。而且,像弱相互作用的情形一样,对于强过程的计算往往给出毫无意义的结果。
    在1 950 年代,引力处于一种很特殊的地位。与弱和强相互作用不同,引力在经典(即非量子)水平上存在一个自洽的,非常优美的理论形式,即爱因斯坦的广义相对论。而且这一理论所导致的独特预言也为实验所证实。但是,物理学家在试图给引力一种量子力学的描述时遇到了广义相对论的主要困难。同弱相互作用的情况类似,数学上的不自洽使得理论对于哪怕是最简单的一些过程也无能为力。
    在五六十年代引力论的量子形式所存在的困难为绝大多数物理学家所忽略,因为引力通常只在天文尺度才变得显著而在这一尺度爱因斯坦的经典引力论完美无缺。引力子与其他粒子的藕合太弱以至于它们不被发现并且在粒子物理中不起直接的作用。然而量子引力的困难如果存在的话要比弱和强相互作用所遇到的困难大得多。广义相对论在20 世纪物理学中占据中心位置,这不仅仅是因为它的成功预言。这一理论建立在非常深刻,明晰和优美的原理基础上,它在原理上如此简单;在数学上如此吸引人;它将引力归结为纯粹的几何。所以这一理论在哲学上如此诱人,有一种无法抗拒的美感。
    相比之下,量子理论有着相当不同的地位。它不具备广义相对论那种内在的简明与美学上的吸引力。它的基本假设是与直觉相悖的,就观察者而言它在哲学的自洽性方面也存在着严重的疑问(关于后面这一点,请读者参阅我们的另一本书《原子中的幽灵》 )。但另一方面,量子理论在应用上的成功又远远超过广义相对论。量子力学已成为粒子物理,核,原子,分子和固态物理,物理化学,现代光学,恒星天体物理学以及宇宙学等不可或缺的组成部分。
    人们常说20 世纪的物理学是建筑在相对论和量子理论之上的。前者极为优美令人叹服但在应用上颇具局限性;后者虽然多少有些哲学上的含混却在科学上创造了无与伦比的记录。两者不相协调这一事实意味着在物理学的核心处存在深刻的灾难性的矛盾。所以任何一个成功的包罗万象的理论必须设法消除这一矛盾。
    
    10 统一的规范理论
    
    多年来量子引力的问题一直被认为是完全无法处理的。随着物理学家把他们的注意力转向弱和强的相互作用,这一问题几乎被彻底搁置在一边。在60 年代初期,希尔顿· 格拉肖(Shel donG } ashow )以及其他一些物理学家注意到,尽管弱作用与电磁作用表面上非常不同,但在更深层次上它们却具有相当多的共同点。例如,它们都是通过自旋为1 的玻色子传递的,而且在许多方面,可以类比电荷、电场等概念引入弱“荷”与弱“流”来描述弱相互作用。电磁作用与弱相互作用的主要不同在于光子是无质量的,所以电磁作用是长程力;而弱相互作用的场量子却很重,使得弱相互作用的力程非常短。如果弱相互作用也能具有电磁力一样的长程性,那么两种相互作用就几乎是完全相同的。因而物理学家们开始相信或许有可能在一个共同的理论里同时包括两种力,从而将麦克斯韦在19 世纪开创的统一计划进一步扩展。
    数学上的分析表明光子无质量这一性质是与麦克斯韦电磁场方程组的规范对称性密切相关的。正是后来证明十分重要的这一规范对称性保证了量子电动力学在数学上的自洽性。相反地,弱相互作用的“媒介”粒子具有质量,这将使得相应的动力学可能具有的任何规范对称性遭到破坏。这一性质使得弱相互作用的早期理论不尽如人意。如果能够克服这一困难,那么将有希望找到一个描述弱相互作用的自洽理论并且使之与电磁作用统一起来。
    在60 年代后期,斯蒂芬· 温伯格(5 1 even We 1 nberg )和阿l \杜斯· 萨拉姆(Abd 日ssalam )各自独立地发现有可能在不破坏弱作用内在的规范对称性的情况下使弱“媒介”粒子获得质量。这一质量可以通过弱作用场内部一定的相互作用来自发地产生,而不是在理论的基本层次― 动力学方程之中引入。质量的这种自发产生是一种次级效应,可使动力学所具有的规范对称性免受影响。
    弱作用的规范对称性可能是自发破坏而不是动力学破坏,这一想法是基于同物理学许多分支中人们较为熟习的其他对称性自发破坏形式的类比。图16 描述了基本的经典力学中的一个例子。如图所示,设想一个小球位于“墨西哥帽”顶。小球处在这一位置显然使得整个系统的状态相对于过顶的垂直轴具有转动对称性。而且,因为重力作用在竖直方向,所以对于系统来讲所有水平的方向都是等同的。因而小球所受的力也同样具有转动对称性。于是,在这一位形中小球的位置(也就是系统的状态)反映了作用在它上面的力的内在对称性。然而,这一状态很明显是不稳定的。如果小球受到扰动,它将顺着帽子的顶面滚下,通过消耗势能,最终会
    停在“帽沿”的某一个位置上。(图16 )这时的位形是稳定的,但转动
    
    对称性被破坏了。小球落在帽沿的哪一个位置并不十分重要,事实上它是完全随机的。然而小球处在帽沿上使得体系的状态不再反映作用在它上面的力所具有的对称性。这里,基本的力所具
    
    图1 6 .对称性自发破坏。小球位于“墨西哥帽”的顶端。这种位形是转动对称的。然而这种状态是不稳定的,小球会自发地滚落到帽沿
    
    去并停留在帽沿的任意一点上。这样,转动对称就破坏掉了。系统
    
    牺牲对称性换取了稳定。
    
    有的对称性依然存在,但它被状态的非对称性所掩盖。这种对称性破坏方式称为“自发”破坏。温伯格和萨拉姆提出W 和Z 粒子(弱作用的“媒介”粒子)是通过弱作用内在规范对称性的自发破坏获得质量的。这样,至关重要的对称性依然存在,只不过是隐藏起来而已。假定对于“媒介”粒子质量的这种解释成立,那么弱作用就与电磁作用具有相同的基础,从而可能存在一种共同的描述。然而,在系统真实的量子状态下,尽管通过适当的安排可以使得光子保持零质量从而仍然反映相应的规范对称性,但W 和Z 则由于它们的大质量已不再能够体现内在的规范对称性。为了使上面的论述成立,温伯格和萨拉姆引入了一种附加的量子场,这种场按照它的发明者的名字被称为希格斯(Higgs )场。希格斯场量子是有质量无自旋的玻色子,它与电磁一弱作用场相藕合的效果是给出一种同图16 所示的“墨西哥帽”完全类似的势能(尽管相应的势能曲面是在一个抽象的空间而非图中所画的真实空间)。在这种藕合的作用下,系统选择了最低能量状态(类似于小球处在帽沿)从而使W 和Z 获得大质量。
    格拉肖一萨拉姆一温伯格理论很好地解释了有效的低能弱作用与电磁作用在强度上的差别。按照这一理论,两种力的实际作用强度是差不多的,和电荷e 类似,引入一个弱荷g ,由它得到一个有效的弱藕合常数g / M ,其中M 是W 粒子的质量。因为M 很大(约为质子质量的80 倍), 所以弱藕合的有效强度,正如它的名字所表明的那样,是非常弱的。在理论中,比例e / g 是一个自由参数,通常用关系式e 一95 1 no 中的角度O 来表示。O 的值要由实验来测定,大约为28 。。所以O 决定了两种力实际强度的相对大小。
    对于该理论,一个关键的理论上的证实在于消除了困扰老的弱作用理论的那种数学上的不自洽。而且,新理论的高能行为是完全令人满意的。事实上,对于人们感兴趣的过程,当能量升高时,弱作用与电磁作用的差别就相应地减小,当能量与M ( 80Gev , 1 Gev = 1 09 电子伏特)可比
    
    时,两种相互作用基本上具有完全一样的性质了。
    就实验而言,新的理论预言了一系列可以测量的细微的物理效应。其中之一是中微子在不产生新粒子的情况下对中子的散射。这一过程在老的弱作用理论中是不可能发生的。1 973 年在欧洲核子研究组织( cERN ) ,一项由一束很强的中微子流穿越气泡室的实验无可争议地显示出,中微子被气泡室中原子核所含的中子散射。然而,关于G 一W 一S 理论的最为明显的证实是在1983 年末和1984 年初,那时在欧洲核子研究组织的质子― 反质子对撞机上首次产生了W 和Z 粒子。它们的质量同利用已知数据所作的理论预言吻合得很好。
    这些令人鼓舞的成功已经导致了这样的信念;即弱作用与电磁作用实际上是一种统一的电弱作用的两个方面。然而,参数O 的值仍然不能由理论确定,所以,或许称“合并”比较统一更好些。
    这一成功的关键因素是理论的规范对称形式,它促进了对于用其他一些规范理论描述强作用和引力以及它们同电弱作用最终统一的理论的研究。
    对于规范对称的描写要用到一个数学分支,称为群论。一个群就是数学上一些元素(实际中经常用一些矩阵来表示)的一个集合,集合中的元素可以由乘法(满足一定的技术限制)组合起来。每一种对称性可以用产生它的群来命名。一个非常简单的例子就是圆环上的对称性。一个圆环在绕其中心轴转动任何角度时保持对称。这些转动构成一个群,称为U ( 1 ) ,其中U 代表“么正”的意思,是一种特定的数学性质。碰巧电磁场的规范对称性正是这种U ( 1 )对称,不过是在某一抽象空间中,而非真实的空间。
    电弱作用把U ( 1 )群同一个称为SU ( 2 )的较为复杂的群联合起来,S 代表“特殊”,我们在这里并不关心它在数学上的具体含义。正如在1 . 6 节中所讨论的那样,强作用可以用量子色动力学作生动的理论描述。这也是一种规范理论,只不过是基于一种更为复杂的规范群― SU ( 3 )。在70 年代中期,人们作了许多尝试来统一电弱作用与量子色动力学以便得到一种“大统一力”。这些大统一理论,或简记为G 盯的目的是找到一种更具包容性的较大的群使得量子色动力学的SU ( 3 )和电弱作用的SU ( 2 )以及U ( 1 )统统成为它的子群。在这些方案中,O 不再是一个自由参数,而可以通过大群分解为所要求的子群的特定方式确定下来。大统一理论的一个主要特点是将三种力的源的特性混合在一起。于是标志电弱作用的轻子与强力的源― 夸克联系起来。夸克和轻子(至少就目前已经知道的而言)具有相同的数目,这一事实暗示了它们之间的这种联系。它们之间的混合是通过交换一组新的媒介粒子来实现的,这组新的媒介粒子统称为X 粒子。通过交换一个X 粒子,一个夸克可以转变成一个轻子,反之亦然。
    
    同电弱统一的情形一样,在低能情况下,三种力有着各不相同的特性,而高能时它们归结为同一种相互作用。达到这种统一所需的能量可以由夸克之间的相互作用随间距加大而增大的性质确定下来。回顾一下海森堡测不准原理,它将一定的能量和动量与一定的距离和时间相联系。由此可知,低能实验揭示了夸克间距离较大的行为,而高能实验则探索夸克彼此非常靠近时的短距离行为。因而,有可能计算在多大距离上― 由测不准原理可以相应地换算成能量― 夸克间的相互作用强度降到电弱作用的水平。在这一能量附近,统一变得明显,因为所有三种力具有差不多的强度。相应的大统一能量比电弱统一的尺度大了约1013 倍,远远超出了现有实验的限度。
    所幸的是,大统一理论还有一些关于低能情况的预言。正如已经提到的,大统一理论将夸克和轻子混合在一起。在发生统一的能量尺度上这些原本不同类别的粒子将成为一体。而在相对较低的能量尺度上,这种混同尽管微弱但还是可以探测得到的。夸克― 轻子混合的最为引人注目的后果,就是预言质子不稳定,是可以衰变的。在一个衰变的方案中,构成质子的底夸克转变为一个正电子,而两个顶夸克中的一个变成一个顶反夸克。这一反夸克与原来的那个顶夸克一起构成一个二粒子。寻找质子衰变事例成为检验大统一理论的一个关键。不幸的是,预言的质子寿命随模型的不同而不同,为1 以“年或者更长。直接探测的结果显示,质子寿命远远长于1 田3 年,这或许表明在探测技术上存在无法克服的困难。所以,测量质子衰变的失败只能用来排除一些理论模型。用来寻找衰变事例的方法是,对从大量材料中放射出来的粒子进行监测。同所有的量子现象一样,质子衰变是一种统计过程。所以,比如说,1 田2 年的平均寿命就意味着在含有1 田2 个质子的材料中一年内大约能探测到一个衰变事例。
    已经有几个这样的实验。其中之一是在伊利湖底的一个深盐矿井里(选择这样的地点是为了尽量减少宇宙射线的影响,否则它的效应将掩盖人们真正感兴趣的事例),那里放有一个大水箱,其中悬置着排成阵列的光子计数器。在质子衰变过程中放射出来的快速带电粒子在通过水时将产生特有的脉冲光。实验的目的就是要探测这一次级辐射。截止到本文撰稿时,在这些实验中尚未记录到确信无疑的质子衰变事例。对于大统一理论的另一个潜在的检验来自于一个完全不同的领域一一破单极。所有已知的磁体都是双极的,即它们同时具有南极和北极。这是因为,任何情况下磁源可以归结为电流环,比如原子中电子的运动。沿环路的电流将在环的一侧产生北极而在相对的另一侧产生南极。所谓的磁“荷”从未在自然界中观察到。一个磁荷将表现为一个单独的北极或南极,于是,更为通常地称之为一个磁单极。
    尽管缺乏对磁单极的观测证据,保罗· 狄拉克研究了如何将磁单极
    
    纳入量子物理学。在1 930 年代初期的一篇经典文章中,狄拉克发现,如果磁单极存在,它们的磁荷m 必须与基本电荷单位e 存在一个简单的关系,显然地写出为,em 一h ,或在h 前乘一个整数因子。除其他方面外,这一奇特的结果还意味着,如果宇宙中哪怕只有一个磁单极,那么e 的值将在任何地方都是固定的。从而解释了为什么电荷的数值总是一个基本单位的整数倍。
    然而,狄拉克的工作并未就这一假想的磁单极可能具有的其他性质,比如它的质量给出任何提示。多年来,物理学家们倾向于认为磁单极是自然界规律所允许的却被造物主所遗弃的那些粒子中的一个。这一看法随着大统一理论的到来而发生了巨大的改变。这些理论中不仅可以有磁单极,并且事实上要求必须有磁单极。而且,关于磁单极可能具有的性质,理论提供了一些更进一步的重要细节。
    预计磁单极的质量与X 粒子的质量相当,大约是质子质量的1015 倍。这一质量如此之大(和一个细菌相差无几),从而解释了为什么在粒子对撞实验中没有产生磁单极。而在另一方面,在宇宙的早期或许达到过(产生磁单极)所必需的能量,所以,一些科学家已经就大爆炸遗留的宇宙磁单极进行了搜索。
    如果那些宇宙磁单极存在,并同其他宇宙辐射一起轰击地球的话,它们将产生一系列可以探测的效应。例如,一个磁单极与一个原子核碰撞可能会引起质子衰变。磁单极还可能具有一个明显的电磁特征。如果电流是绕一个超导体环运行的,则环的磁通量是量子化的,即它的值是h 的整数倍。如果一个磁单极穿过环路,那么磁通量会由于电磁感应而增加一个固定量。所以,实验者只需要使环路处于超导状态,剩下的事就是等着磁单极从中穿过。除了在1 982 年的情人节那天得到过一个戏剧性的假信号外,至今尚未在这类或任何其他实验中探测到磁单极。
    
    11 超引力
    
    在1 970 年代,尽管统一电磁,弱和强作用的方案取得了令人鼓舞的进展,引力却仍然孤傲不群。然而,这期间引力学家们也决非无所事事。在70 年代中期,他们对超对称的概念作了重要的推广。我们知道,超对称基本上是一类抽象的几何对称(见1 . 8 节)。而爱因斯坦的广义相对论当然是引力的一种几何理论。所以,有几位研究人员各自独立地发现,超对称几何也可以作为引力几何理论的基础,相应的理论就称为超引力。
    超引力包括了爱因斯坦的广义相对论,但作了扩充。爱因斯坦的理论在极限条件下仍然成立,因而理论同观测的精确符合并未受到破坏。超引力带来的主要特点是,引力子已不再是传递引力的唯一媒介粒子。请回忆一下,超对称是在费米子与玻色子之间提供了某种联系。如果对自旋为2 的引力子实施一个超对称操作(即包含从普通空间到额外的费米自由度作
    转动的数学运算,那么理论就描写一个自旋三的粒子。自然界中尚未发现
    
    的基本粒子,所以这是一种新粒子。这类新粒子称为引力量
    
    子,对于不同的模型,它们的种类数目是不同的,可以从1 到8 不等。引力量子如果存在,将同引力子一样具有极弱的藕合,因而很难在实验中测到。
    进一步的超对称操作可以产生更多的粒子,它们的自旋为1 ,王和。。
    
    在一个人们比较赞同的超引力理论中,引力量子的超对偶粒子总数达172 个,由于这一理论中有8 种引力量子,人们称之为N 一8 的超引力理论。人们尝试把其中的一些超对偶粒子同高能物理中已经知道的粒子去一一对号入座,以便提供一种可能的超统一方案。在这种包罗万象的理论中,其他几种力的媒介粒子― 光子,胶子,W 和Z 粒子― 和引力子一起都将属于同一个巨大的超“族”,即通过超对称联系在一起的一个粒子多重态。于是,有可能把所有的力统一起来,原来的每一种力只代表某个单一的具有超对称的超力的一个侧面。但这还不是理论的全部内容。因为,超“族”中还包括有费米子,它们或许同组成物质的基本粒子― 夸克和轻子有关。这样,有可能对物质以及相互作用进行统一的理论描述。
    尽管这一宏大的理论结构有着如此诱人的魅力,但从已知粒子中寻
    
    找引力子的超对偶还仅仅是一种理想而已。不过许多理论家已经兴奋地宣称超引力可能就是人们长期以来梦寐以求的那个包罗万象的理论。剑桥大学的史蒂芬· 霍金(Stephen Hawk 1 ng )在他就职Lucas 1 an 数学讲座教授的演说中提到,假使N 一8 的超引力大有希望,那么“理论物理的终结为期不远了。”人们为进一步优化理论以及探索它的细节作了巨大努力。为类比起见,发展了其他一些较引力而言容易分析的场论的超对称形式。其中重要的进展是,发现当理论在大于四维的时空中构造时,超引力的几何结构可以大大简化。对于N 一8 的超引力,最为有利的维数是11 。
    80 年代初,当一部分理论家们忙于在11 维时空中重新表述超引力理论时,与之并行地,另外一部分人则开始在克莱因一卡鲁扎理论框架下研究额外自由度的问题。他们的目的是要把原来只涉及引力与电磁作用的理论进行扩充,以便同样能够描述弱作用和强作用。这是可能做到的,因为温伯格和萨拉姆的理论以及量子色动力学为弱与强力提供了非常类似于电磁理论的规范场论描述。
    在克莱因一卡鲁扎的原始理论中,电磁作用仅仅通过附加一维时空自由度(使得总维数为五)而引入的。这是同传递电磁作用只需一种光子这一事实相联系的。换句话说,是同电磁场的规范对称性为最简单的U ( 1 )对称相联系的。而另一方面,弱和强作用则具有较复杂的规范对称( SU ( 2 )和SU ( 3 ) ) ,所以需要一个多重态的媒介粒子传递它们的相互作用。于是,在扩充的克莱因一卡鲁扎理论中它们各自要求的额外自由度均大于一维。汇总起来,时空的维数刚好也是11 。
    在11 维的克莱因一卡鲁扎理论中,只存在一种力― 引力。而电磁,弱和强力只不过是引力的附属品。所以,扩充的克莱因一卡鲁扎理论在一个统一的框架下对自然界所有的力给出一种完全几何的理论描述。这里,成功地描述一个量子场论所需要的那种至关重要的抽象规范对称性与更高维时空的几何对称性是一回事。
    由超引力与克莱因一卡鲁扎理论同样得到11 维自由度,这一巧合是颇具启发性的。于是,物理学家们开始认真谈论起同时具有超对称与高维自由度的包罗万象的理论了。现在人们逐渐认识到,那些一开始作为纯数学手段应用到超引力的额外自由度是真实的物理自由度,它们通过克莱因一卡鲁扎的原始理论中描述的方式卷缩到一个很小的尺度上。不幸的是,这种11 维的理论存在一个被证明是致命的弱点。弱作用的一个明显特征是它破坏左右镜象对称(也就是说,它是宇称不守恒的)。这意味着基本粒子必须具有一定的左右手标志― 或称“手征性”。在日常生活中,我们认为左手与右手之间的差别是理所当然的,但手征性的存在实际上依赖于三维空间更深层的性质。研究表明,确定的手征性只存在于单数维的空间中。这就是说,空间的维数是奇数。因而时空
    
    的维数必定是偶数。否则自然界的规律中将没有手征性。一句话,11 维时空理论不成立。
    
    12 数学的困难
    
    在前面的叙述中,我们曾几次提及相互作用的量子理论描述中所存在的数学自洽性问题。本节中,我们将较为详细地考察一下这些数学问题的本质。
    关于量子场论所遇困难的第一个线索实际上来自经典的电磁理论。一个困难就是电子的结构问题。电子的一个原始图像是一个均匀分布的带电小实心球。因为同号电荷相斥,所以电子中的一部分电荷会排斥另一部分电荷,从而存在将电子分裂开来的向外作用的力。由于平方反比律,如果假定电子的半径很小的话,这一向外作用的力将会非常强。为使电子不致分裂,必须有某种内力作用。这些亲和力应该是无论电子如何运动而始终精确地抵消那种分裂的趋势。以一种与狭义相对论协调的方式描述这一平衡作用的尝试被证明是徒劳的。物理学家们最终不得不回避这一困难,认为事实上电子是点粒子,也就是说它的半径为零,于是不存在力学理论所适用的内部结构。
    这种想法只不过是通过引入一个问题来解决另一个问题而已,因为当电子成为点粒子时,它的静电能存在着困难。把电荷分布在半径为r 的球面上的能量正比于r 一’,所以,如果允许r 为零,那么相应的能量则为无穷大。在狭义相对论中存在质能关系,于是无穷大的静电能意味着电子具有无穷大的质量。
    尽管方程中存在一个无穷力或“发散”项是一个严重的问题,但是,如果这一“发散”项本身并不代表可测的物理量的话,那么这一问题就不一定是灾难性的。在没有引力作用时,能量并不是可测量的,可以测量的是能量之差。于是,人们可以通过相差一个无穷大来重新标度能量的零点位置以使测量到的电子质量是有限的。这种重新标度称为重整化。尽管在中间步骤中存在无穷大,但仍可以得出有限的结果,这样的理论称为可重整化的。
    在1 930 年代,关于量子电动力学的工作就开始了,这是一个描述电子与电磁力的载体― 光子相互作用的理论。在量子电动力学中,有关电子的电磁自作用问题更为微妙。事实表明,无穷大带来的困难比经典理论所遇到的困难更为严重。在量子电动力学中,电磁力是通过交换光子传递的。在这种描述中,自作用表现为带电粒子先发射一个光子然后再将它吸收掉。这种情形尽管难以形象地描绘,但是,海森堡的测不准原理使得我们不必追究上面提到的那个光子到底是如何转过弯来的,它的位置和运动是模糊不清的。这一过程可用图17 中的费曼图表示。
    
    图1 7 .电子发射和吸收光子。这种过程给电子罩上了一层电磁能外衣,于是在计算电子的自能时会出现无穷大。
    
    图17 中的波浪线表示光子,代表了围绕电子的电磁能量。它对电子质量的贡献同经典电动力学中的大小完全一样。如果再次假定电子为点粒子,那么这些光子携带的能量就没有限制。原因可以追溯到关于能量的测不准关系。光子运动的距离越短,则所花费的时间越短,因而能量的不确定程度就越大。对于一个点粒子来说,往返过程几乎完全不花时间,于是,光子可以具有无穷大的能量。具体的计算表明,从围绕其周围的光子那里,电子可以获得无穷大的质量。
    但此时的重整化要难得多。首先,理论中还存在诸如电荷等其他一些无穷大,这些无穷大也需要重整化。其次,图17 只表示对电子质量的一种无穷大贡献,发散项同样还来自发射和重新吸收两个,三个,四个… … 光子。事实上,存在无限多个无穷大项,乍看起来似乎需要无限多个相应的重整化操作。如果真是那样的话,理论显然就变得毫无意义。事实上,只需要单独一种重整化就可以从可观测量中消除所有的发散。证明这一点需要较多的数学。的确,过了几乎20 年人们才做到这一点。这一性质十分稀有同时也很重要,它在很大程度上依赖于理论的规范对称性。
    量子电动力学并不是已知的量子场论中唯一可重整化的理论,但却是目前最为重要的理论。它所给出的预言惊人地准确,因而被视作其他相互作用的典范。相反,关于弱相互作用的旧的理论是不可重整化的,基于爱因斯坦广义相对论的量子引力论也是不可重整化的。在这两种理论中,无穷大无休止地重现,使得理论失去内在自洽性,变得毫无预言力。
    同无穷大问题密切相关的是反常问题。当一个理论“量子化”时,也就是把经典理论按照量子力学的规则重新表述,某些对称性在这一过程中被破坏掉了,这种现象叫作反常。这样的称呼并无恶意。反常的出现意味着所挑选的经典理论中存在某种对称性,这种对称性在相应的量子理论中不再成立。由于对称性同守恒律之间的密切联系,反常可能导致违反一些被奉为神明的守恒律,例如,能量和电荷在相应的量子理论中可以不再守恒。关于这一现象是如何发生的,可以大致这样理解。如果一个量O 是守恒的,那么它的变化率为零,正如我们已经看到的那样,量子化的过程经常涉及到一些量被乘以无穷大的因子。于是,有可能出现这种情况,即O 的变化率被乘以无穷大。这样得到的乘积为0 又co 。这个表达式看起来是毫无意义的,但是经过适当地定义,它可以表示一个有限的量。这正是存在反常时的情形:O 的变化率不再为零,于是关于O 的守恒律被破坏掉了。
    
    13 弦理论
    
    在1 980 年代早期,寻找统一理论― 或许甚至是一个包罗万象的理论― 的工作就已经进入了一个既令人神往同时也让人失望的阶段。有些概念,诸如超对称和高维自由度等,看来似乎为研究提供了很有希望的途径。在构造量子引力理论时曾经困扰人们的诸多有关无穷大的问题在超引力理论中即使仍然存在,至少也是可以得到改善的。同时,扩充的克莱因一卡鲁扎理论尽管不甚完善,仍为统一四种相互作用提供了诱人的理论框架。总之,当时的理论家们非常乐于接受把超对称和高维自由度结合在一起的统一方案,正是在这一点上,他们开始注意到弦理论。弦理论的起源可以追溯到1 960 年代后期以及加布里埃尔· 维尼齐亚诺(Gabr 1 e 1 1 eVenez 1 ano )的工作。当时,很多物理学家致力于解释那些被不断发现的强子,它们是在粒子加速器的高能碰撞中产生的强相互作用粒子。那时,物质结构的夸克模型尚未建立起来。
    特别令人感到困惑的是那些寿命只有10 一23 秒量级的短寿命强子。它们统称为共振态,因为,它们很明显不是基本粒子而像是其他强子的某种激发态。人们可以设想,由于高能碰撞,强子的内部成分被激发到更高的
    
    ,、., , ? , .一一一一一一,11 … 。~一~一,曰一一一二。,11 、一、.卜醚纵上云。饼笼衣明,共甲一些共振忐县们很局阴目证以口一)。里进一2
    
    步,人们发现了这些强子的质量和自旋间的一种规则的关系。为了解释这些事实,维尼齐亚诺专门提出了一种模型。当时,它还仅仅是一种没有任何内在物理图像的数学方案。然而,在随后的进一步研究中变得越来越清楚,维尼齐亚诺的模型是描述一根弦的量子化运动。对于以往那些总是用粒子来构造物质的理论来说,这是一次引人注目的尝试。并且至少在某些方面,与那些传统的粒子模型相比,弦模型同实验符合得更好。
    至少从一种意义上讲,强子的弦理论证明是对的。就目前所知,强子包含夸克,而夸克之间存在相互作用。可以想象,这种相互作用所产生的束缚就好比是连接在夸克之间的橡皮筋。事实上,夸克之间的相互作用力和橡皮筋一样,随着间隔加大而加大。当夸克运动起来时,强子内部的相互作用就像一根扭转的弦。就夸克而言,这种相互作用力很强,致使相互作用能可以同夸克的静质量相匹敌。在这种情况下,橡皮筋比其两端的夸克更显得重要。所以,用一根弦来描述整个强子的运动并非完全不当。
    在早期,弦理论一直被当作不过是一种粗略的近似。另外一个问题是,它看来只适合描述玻色子。尽管如此,个别研究人员仍对弦模型作了深入的探索,并得到某些有趣的结果,它们表明理论是强有力的。然后,在1970 年,约翰· 施瓦茨(Johnschwarz )和安德烈· 尼夫厄
    
    ( Andr6Neveu )找到了描述费米子的弦理论。
    大约在1 974 年,人们发展了量子色动力学,而弦理论作为强子的模型则不再引起人们的兴趣。如果不是施瓦茨和他当时的合作者乔尔· 谢尔克(Joel scher 幼发现了另一个完全不同的并且更加令人振奋的新途径的话,弦理论很可能会就此消亡。早期的弦理论所遇到的问题之一,是它所描述的粒子中存在自旋为2 的零质量粒子,而这种粒子在强子谱中并没有具体对应。然而,它却精确地描述了引力子― 引力的“媒介”粒子。于是谢尔克和施瓦茨建议,或许弦理论事实上是一个引力理论,甚至可能是一个包罗万象的理论。
    经过10 年的时间,这一大胆的设想才获得了更为广泛的信赖。在这期间,包括约翰· 施瓦茨和米歇尔· 格林(在内的一小组理论家处理了所有的数学上的自洽性问题― 快子,无穷大,反常,高维自由度的引入,超对称的引入。令人啼笑皆非的是,当时他们被认为是在一个完全荒诞的理论上白废时间。今天,情况完全不同了,弦理论― 按现在的说法叫作超弦理论― 引起了当今世界上一些最优秀的理论物理学家的重视。
    在下面的章节里,包括施瓦茨和格林在内的一些弦理论的创始人将详细地描述弦模型。他们讲述了弦理论的一些历史和现状,并就将来的发展作了展望。他们还就弦理论是否能够成为一种包罗万象的理论这一关键问题发表了看法。
    毫无疑问,弦理论是极其吸引人的。理论家们大谈它那令人称奇的优美结构和丰富内涵。但是,研究弦理论的另一个动因在于,它最终会给出一种关于自然界所有粒子以及相互作用的定量描述,那么它将是人类历史上最伟大的科学成就之一。的确,人们可以声称它是简化主义科学的顶峰,因为我们终于能够找到构成世界的最小单元并且由此阐明宇宙发展的基本原理。无怪有些研究人员一夜之间放弃从事多年的研究项目而转向弦理论。在粒子物理以及引力理论领域中,几乎没有哪个讲座或发表的杂志不或多或少地涉及到弦。
    然而,并非所有物理学家都对此感到满足。有人争辩说,弦理论家们的努力无论在哲学上还是在科学上都是引人误入歧途的。更有甚者称弦理论是彻头彻尾的谎言。这些批评意见在后面的访谈录中有所反映,读者可以自己判断究竟谁是谁非。但有一点是公认的,即从来没有任何科学事业像弦理论这样胜败悠关。
    
    2 .约翰· 施瓦茨(Johnschwarz )
    
    约翰· 施瓦茨是加州理工大学物理系教授。他的早期工作,特别是同米歇尔· 格林合作的那些工作,对于弦理论这一课题从一潭死水发展为今天强有力的超弦理论起了推动作用。
    
    用弦模型描绘基本粒子的想法由来已久,您能否谈一点儿弦理论的
    
    早期情况?
    
    弦理论有着非同寻常的历史。这一课题的提出最先是为了解决一个同目前弦理论所处理的完全不同的问题。起初,大约在1968 一1970 年,人们发展弦理论是为了解释强核力。就这一点而言,弦理论取得了一定的成功,但决非彻底的成功。而到了1 970 年代中期,另一个称为量子色动力学的理论被提了出来并成功地描述了强相互作用。结果,早期的弦理论尽管有大量的工作要做,但随着量子色动力学的产生,到1 970 年代中期,大多数人就放弃了这一课题。我之所以没有放弃,是因为在量子色动力学发展之前,或者说差不多同时,我正在同当时在加州理工大学访问的法国物理学家乔尔· 谢尔克合作。我们注意到,试图用弦理论描述强核力所碰到的问题之一是,理论中总会出现一种强核区并不存在的特殊粒子。就是说,这是一个零质量,角动量为2 的粒子,在核过程中从未观测到任何这类粒子。可是我们知道,这种粒子存在于爱因斯坦的广义相对论,这是一个引力的理论,而这种粒子通常称为引力子― 一种传递引力且满足量子力学规律的粒子。引力是非常不同于强核力的一种相互作用,在通常情况下,它非常非常微弱,所以,既然我们发现弦理论中总是出现这种粒子,我们就决定放弃用弦来描述强核力的方案转而考察是否可以用它去描述引力。现在看来,当时其他基本相互作用也在经历类似的转变。
    
    事实上是转祸为福。
    
    是的。这需要观念上的巨大转变,因为起码它意味着弦必须比我们原先想象的要小得多。
    
    这里所指的是哪种尺度?
    
    当我们考虑用弦来描述粒子时,我们的想法是,弦应该有一个核子的典型尺度,即10 一,3 厘米大小。当用到引力理论时,就存在一个由引力
    
    结构带来。的自然长度单位,称为普朗克长度,它要比核子尺度小得多― 是核子尺度的1 以0 分之一。人们有时这样形容,普朗克尺度之于一个原子的大小如同一个原子之于太阳系的大小。在我们讨论用弦来统一引力与其他力时,我们所谈论的是极小极小的尺度。
    所以,用弦来描述引力理论及统一理论的想法产生于1 974 年,那时弦理论提出来已有五年了。乔尔· 谢尔克和我当时在继续这方面的工作,不幸的是,六年后他去世了。于是,从1979 年起我开始同来自伦敦玛丽皇后学院的米歇尔· 格林合作。
    在讨论那些进展之前,我是否可以问一下,您关于旧的弦理论中的质子和中子的图像是什么?是否在某种意义上假定每个质子和中子内都存在一根弦呢?
    
    好的,粗略地讲,图像是这样的:强子,如中子和质子,是由夸克构成的。夸克这一概念差不多是20 年前由盖尔曼和兹威格引入的。这些夸克必须由某种力束缚在一起,所以物理图像就是:弦即是对于把夸克束缚在一起的那种力的描述,有点像橡皮筋。可以认为,夸克是系在这些弦的端点上的。
    
    那么,它们整体以某种方式扭转?
    
    是的。
    
    这一想法的主要困难是什么?
    
    存在着几个困难。一个是我已经提到的,这个零质量自旋为2 的粒子的出现在数学上是势所必然,但在核子区可以找到的粒子谱中却没有对应物。
    另一个困难非常奇妙,因为,理论的数学自洽性要求时空维数大于4 。在原始的弦理论中,同时还存在着其他一些不足之处,这个理论要求维数等于26 。1971 年,皮尔· 拉蒙特(Pierre Ramond ) ,安德烈· 尼夫厄和我一起发展了一种改进的弦理论,它的时空维数降为十,而事实上现今流行的正是这一理论的某个变种。就描述核子而言,存在额外的自由度是一个非常严重的问题,因为我们完全清楚,真实世界只有三维空间自由度和一维时间自由度。于是,如果你想要一个现实的理论,就不能有额外的自由度。
    
    当时您是否希望通过某种重新表述来得到一个四维时空中的自洽理论呢?
    
    是啊,那些年人们作了很多努力― 同样我也为此付出了部分精力― 以期找到这两种弦理论的一些变形,使得它们的维数是四而非十或26 。沿着这一线索,人们提出了很多有趣的方案。他们总是从一个数学上十分优美的系统出发,但后来却将它弄得丑陋不堪并且毫无说服力,最终不可避免地导致数学上的不自洽。
    
    原始弦理论所遇到的其他问题之一是存在所谓的快子,它们是一些
    
    比光速还快的粒子。这是否不可避免?
    
    在26 维的玻色弦理论中这是一个不可避免的特征。十维弦理论的优点之一就是有可能通过选择理论的形式使其不包含任何快子。我们知道,这种粒子是同基本原理不相协调的。
    
    }日的弦理论大概也有某些成功之处吧?
    
    是的。这个理论的提出是有充分理由的。它抓住了许多一般特征,而我们知道这些特征是一个强核力理论应该具有的,它们包括,粒子在高能情况下如何发生相互作用的一定特征,不同粒子的质量和角动量,以及描述它们之间联系的模式等一些其他相关特征。
    
    今天,回顾那个阶段,是否可以确定地说,不应再把弦作为核子的一种图像,它已由量子色动力学代替了呢?
    
    量子色动力学已经被高能物理界公认为描述强核力的正确理论。我认为这方面的证据是非常可信的。然而,通过某种方式重新表示量子色动力学会使得弦在其中扮演重要的角色看上去仍然十分可能。但是在这种意义上的弦,其数学上的行为将同15 年前提出的弦完全不同。这样一个弦理论的精确结构只能由我们今天所了解的知识不甚明确地导出。事实上,比起我们目前正在研究的看上去非常含混的超弦理论,它似乎要困难得多。
    
    弦理论发展中的真正转折点是什么?换句话说,把它推至粒子物理
    
    研究最前沿的是什么?
    
    这始于我在1 980 年同米歇尔· 格林的合作。当时我们改进了早先我同乔尔· 谢尔克的工作,建立起十维弦理论的数学行为的细节。关于这一十维弦理论,我要提到的重要特色之一是,它具有一种非常特殊的对称性,称为超对称,这种对称性把叫作玻色子和费米子的两类不同的基
    
    本粒子联系了起来。
    
    您可否谈谈这两类粒子?
    
    所有的基本粒子分成两大类,这两种类型的粒子,即玻色子和费米子在两个重要方面是不一样的。它们所携带的角动量一般称为“自旋”, 一个玻色子的自旋是基本单位的偶数倍而费米子的自旋是同一个基本单位的奇数倍。
    另一个区别是同量子理论的问题的密切联系,它与全同粒子交换时理论的行为有关,表现为,在这种交换下,理论是否不变,或是相差一个负号。交换费米子会带来一个负号因子。
    
    您是在说起对称是将这两类粒子合并为一种描述的方法。
    
    是的,正是这样。或许为了不那么抽象我应该说夸克和电子就是费米子的例子,而光子和引力子则是玻色子的例子。
    我们把费米子当作构成物质的粒子而把玻色子作为在它们之间传递相互作用的粒子,这么说对吗?
    
    我想这是一个好的划分办法。
    
    所以,您所说的超对称在现代弦理论中是一个非常重要的概念。那么,它带来哪些发展呢?
    
    这说来话长了。事实上,早在1 971 年,十维弦理论的发现可以说是超对称理论的真正开始。其中一个方面是作为引力理论的超对称推广,这一理论称为超引力理论,于1 976 年完成,现在被合并到超对称的弦理论中,简称为超弦理论。
    在研究超对称弦理论性质的那些年里,我和格林发现了一系列我们感到激动人心的事情。按传统方法构造一个引力理论时,所遇到的诸多重要问题之一是,当人们设法满足量子理论的要求时,计算常常会导致毫无意义的发散表达式。这种情况类似于一个数被零除,这样的计算是无法进行的。所以,当你试图进行引力的量子理论计算时,就得到这些没有确切定义的结果。这看来是所有那些把基本粒子当作数学上的点粒子进行处理的理论所共有的特征,而这种点粒子处理是一个传统的办法。
    所以,弦理论的重要特点就是用称为弦的一维曲线来代替那些点(粒子),于是,我们发现一个令人振奋的结果,即在弦理论中当我们计算
    
    引力的量子修正时,能够得到有意义的数值,这些数是由有限的表达式给出的。这似乎是首次表明有可能得到一个同量子力学协调并包含引力的有限理论。这是很令人兴奋的。我们大约在1 982 年前后完成了这项工作。
    差不多同时,我们还发现了其他一些超弦理论,其中包含我们所说的开弦(即弦的两端点自由);另外一些弦形成环路,我们称之为闭弦。所以,在原始的超弦理论中既有开弦也有闭弦,但是稍晚一些我们发现,有可能使理论只包含闭弦。这看来是一种重要的区别,因为,目前大有希望的理论正是那些只含闭弦的理论。事实上,从很多方面讲,它们是更容易研究的。
    在我们的基本理论中要对自然界作出解释,而关于自然界的诸多重要事实之一是左右手征存在差别。所以理论不应具有镜象对称― 这称为宇称破坏。这是关于弱和强作用的标准模型具有的一个重要性质,而我们知道,这一理论在低能情况下是适用的。以一种更为基本的观点,特别是在超弦理论的框架下去理解这种不对称,对我们是一种挑战。现在看来,在已经得到的几种超弦理论中,除了一种以外其他的都具有这种左一右不对称,因而,它已经成为十维理论的一个基本特征。所以,这是非常鼓舞人心的。然而,这种具有左一右不对称的理论存在着某种崩溃的倾向并且带来不自治的结果,这种不自洽性不是我刚刚提到过的那种无穷大,而是一个与之密切相关的问题,称为反常。这里的基本想法是,在考虑量子力学之前,理论具有一定的基本对称性,于是,就存在一个非得回答的问题,即量子力学修正是仍然保证这些对称性还是破坏了对称性。如果量子力学修正破坏这些对称性,那么理论就不自洽,因而也将变得没有意义。只要理论具有左一右不对称,这种不自洽总是要发生的。所以,理论具有左一右不对称一方面是令人兴奋的事情,另一方面也是一种巨大威胁,因为它们可能导致相应的反常从而变得不自治。1 984 年,米歇尔· 格林和我一起对其中一种超弦理论作了计算,要看看这种反常到底会不会发生。计算结果令我们大吃一惊。我们发现,一般说来确实存在着反常,使得理论不尽如人意。但是,在一开始构造理论时有可能选择特别的对称结构。事实上,对称性的选择有无穷多种可能性。然而,只有其中的一种可以使得表达式中的反常奇迹般地抵消掉,其他的选择则不具备这一性质。所以,在无穷多种可能性中,只有唯一的一种可以挑选出来试一试。
    
    在处理粒子以及相互作用的传统场论方法中存在两类困难。一类是无穷大项的出现,另一类是这些所谓的反常,反常会在理论量子化过程中带来不受欢迎的破坏对称性的因素。所有这两类困难都会导致理论的不自洽,但是对于超弦而言,这两类问题似乎都有办法得到解决,只要
    
    是在这个唯一选择好的超弦理论中进行。那么如何选择出这个唯一的理论呢?选择的特征是什么?
    
    我已经提到过,这种特殊的对称结构是从无穷多种可能的结构中挑选出来的,而这种结构是在研究反常问题之前就已经存在的。这种对称结构称为50 ( 32 )。
    几乎在这同时,我们还发现存在另外一种称为〔 SX 〔 8 的对称结构似乎也是一种自洽的选择。奇怪的是,当时我们还没有得到一种确定的超弦理论可以具有这种对称性。所以,我们得到一种超弦理论,它具有我们认定的对称结构中的一种;而对于剩下的那种对称结构,我们还没有找到可以具有这种对称性的理论。但是,在那之后不久,来自普林斯顿大学的被称为“普林斯顿超弦四重奏”的四位物理学家发现了两类新的超弦理论,他们称之为异常弦,其中一类具有〔 SX 〔 8 对称性。另外的那一类具有S0 ( 32 )对称性,这是具有S0 ( 32 )对称性的第二个例子。〔 SX 〔 8 理论是最令人感兴趣的,因为这种对称性看来最有希望解释已观测的粒子的唯象结果。
    
    但如此一来,我们似乎有很多可供替换的弦理论。这是不是一件糟糕的事情?
    这个数目仍然是相当小的。当然,原则上讲,你是对的。最好是只存在一种可能的理论,而它能够解释一切。我想,可以说我们已经在这个方向上取得了较大的进展,尽管这方面的情况同人们所听到的并不完全吻合。在十维理论中目前有三类异常弦理论(除我已经提到过的两类外,第三类是后来被发现的)和三类非异常的超弦理论,所以迄今共有六类。然而,很有可能通过进一步的研究发现其中有一些实际上是不自洽的。那么这个数目就会减小。还有,三类异常弦理论可能原本是一种理论的不同形式而已。事实上,它们的相互等价是可以证明的,所以应该把它们当作一种理论。这样,通过上面的推证,极有可能使得这个数目减少到一,即只有唯一的一种理论。
    
    为什么理论必须在大于四维的时空中构造不再成为问题?
    
    一旦我们放弃强子方案― 用弦描述强核力― 转而讨论用弦描述引力以及其他相互作用力,那么额外自由度就成为一个优点,而非缺点。因为,引力理论是描述时空几何的,所以在一个引力理论中,完全可以设想存在额外自由度,只不过由于理论自身的几何原因,这些额外自由度卷缩成紧凑的小球而已。
    所以理论中将会存在额外自由度。但是,理论还应该告诉人们如何
    
    处理这些额外自由度,因为在解方程过程中,如果一切按计划进行的话,就会发现方程的解保证了这额外的六维自由度缩成一个小球,这个小球如此之小以致于人们无法观测到它。
    
    到底有多小呢?
    
    似乎表明应该是我前面提到的那个长度单位― 普朗克长度。那是一个非常小的长度,只有10 一23 厘米。
    
    您是说,在空间的每一点,或者说是我们认作空间的一点,事实上是一个尺度为10 一33 厘米的六维小球。因而我们没有察觉到这些额外自由度就不足为奇了。
    
    它们太小了,根本无法注意到。
    
    那么我们如何来想象这些弦呢?是否应该认为粒子,比如说夸克或者电子,在某种意义上是由这些弦构成的呢?能否认为它们当中存在一根弦,一个圈或者诸如此类的东西呢?
    
    好,让我以一种稍微不同的方式来表述。对于一根弦来说,它可以以不同的方式振荡或摆动,转动等等,其中任何一种可以被认为是描述某种特殊类型的粒子。因而,可以认为电子是一种振动模式,夸克是另外一种振动模式,而引力子又是与它们不同的某一种振动模式。
    
    就是说,其中只有一种类型的弦,但以不同的方式运动,也就是说运动的模式不同而已?
    
    是的。
    
    您刚才提到,超弦理论中最有希望的是〔 SX 〔 8 形式,那么这两种〔 8 的重要意义是什么?
    
    在问题得到彻底解决之前,很难完全弄清究竟是怎么回事。但目前看来令人感兴趣的一种可能性是,这两种〔 8 对称性之一包含了迄今已经知道的粒子物理的所有对称性,这些对称性是从目前所能达到的能量范围内的实验中总结出来的。另外的〔 8 对称则描述一种新的物质,有时称为影子物质(shadow matter ) ,它同我们所熟悉的普通物质没有相互作用或者仅存在很弱的相互作用。如果你打算由此构思科幻小说的话,你可
    
    以想象由这种影子物质组成的各种星系和行星,它们是不可见的,因为它们同我们看得见的光线没有相互作用。
    所以,一种奇妙的可能’!生是,与第二种〔 8 对称相联系的影子物质是完全不可见的,因为它们同我们能够探测得到的光不发生相互作用。
    
    那么此时此刻有可能正好有影子物质从我们谈话的这间屋子中穿过,而我们没有察觉到它?
    
    是的。不过你仍然可以对它有所制约,因为它同我们所了解的引力存在相互作用― 普通物质与影子物质所受的引力是一样的。
    
    所以我们能够探测到一个影子行星?
    
    尽管我们不可能通过光线看到它,但是通过引力效应我们还是能够察觉到它的。
    
    是否有什么证据表明存在这种影子物质?
    
    没有。但它同我们对于宇宙的了解是相协调的。因为有证据表明,宇宙中的可见物质可能只占宇宙总质量的百分之十左右。因此,即便宇宙中有一半质量是影子物质也是十分协调的。对于影子物质来说,是有它存在的余地的。
    
    就粒子及其间的相互作用的性质而言,这个影子世界是否或多或少地与我们的普通世界相同呢?
    
    这个问题依赖于诸如怎样构造理论这样一些细节。一种可能性是,两种〔 8 对称以相同的方式破缺为更小的对称结构。如果情况是这样的话,那么这两种物质构成的世界中物理规律就有相同的对称性。目前看来更为可能的是,这两种〔 8 的破缺模式并不相同。
    
    为什么会是这样的呢?它们是通过什么来区分的?
    
    当尝试求解理论的方程时,在人们已经成功地发现的唯一的解中,对于两种〔 8 引入了各不相同的对称破缺模式。
    所以,真实世界和影子世界是不均衡的?
    
    是的,但并非没有可能发现另外的解,其中两种对称性具有相同的破缺模式。
    
    按照我的理解,超弦方案所遇到的突出问题之一在于如何确定六维额外自由度卷缩到什么样的特殊结构。您认为这是一个无法超越的障碍,还是在几年之内就可以从数学上解决的问题呢?
    
    这是一个巨大的挑战,并且或许是这一课题中迄今为止的两个最基本的问题之一。如果我们知道那个六维空间是什么样子,我们就能由此算出想要的一切。这听起来有点儿令人吃惊。毕竟,正如我们说的那样,这个空间是完全不可见的,因为它太小了,以致于无法直接观测到。然而事实上,这个空间的几何与拓扑的具体情况对于确定可观测能区内可观测粒子的性质起着关键的作用。
    
    您能否给一个例子?
    
    对于这个六维空间,存在一个称为欧拉(〔 u } er )数的拓扑性质,粗略地讲,它可以告诉我们空间中存在多少个空洞。研究表明,这个欧拉数同夸克和轻子族中存在多少代有直接联系。已经发现,夸克和轻子是一组一组地出现,每一组称为一个代。实验上已经观测到三代,而为什么存在三代夸克和轻子,这是诸多未解之迷中的一个。弦理论令人兴奋的事情之一就是这个理论中的代的数目恰好等于六维空间欧拉数的一半。
    
    所以,我们有了一个说明这些不可见的空间自由度的拓扑如何直接影响物理事件的例子,比如,自然界中所发现的不同种类粒子的数目?
    
    是的。
    
    超弦理论存在的问题之一,是目前尚无一个唯一的理论,相反,存在很多种理论,这些理论因多余自由度卷缩的方式不同而不同。那么大体上讲,我们已经得到多少种不同的选择?
    
    首先让我换一个说法来表达你的意思。依我说,理论是唯一的,而不确定性是在求解的过程中引进的― 一个理论可以存在许多种不同的解― 我们的一个大问题是要了解,为什么从某种意义上讲,一种解会比所有其他的解更好,并且好到足以解释自然界。
    我们目前的知识水平尚不足以提供办法从诸多不同的解中作出选
    
    择,我们所能作的只是说某种解看上去比另外一种更好些,但还没有任何的数学判据。
    然而,理论尚未被彻底了解。我们仍在寻找最佳的表述方式。特别是在目前的超弦理论形式中,我们只能通过某种逐级近似对理论进行研究,这样的方案称为微扰理论。我们正在寻找有关理论的某种表述方式使得我们并不局限于这种逐级近似的特殊展开。如果我们得到了理论的这种表述,它可以给出精确的结果而非逐级近似,那么我们就有可能发现,从精确求解的角度看,那些到我们目前研究的任何近似水平上看来成立的六维空间的解其实并非全部是解。
    所以,如果可以精确计算的话就能选出唯一的解来。
    
    是的。对此有句行话说,理论中存在非微扰效应,它将把除一种或几种之外的解排除掉。
    
    但是,就您目前所知,存在多少各不相同的候选的解呢?
    
    这真是很难计算的,但我想你可以说存在几千种吧,还可能更多。
    
    除此之外,您认为理论遇到的最突出的问题是什么?
    
    当属理论的非微扰表述形式。对弦理论而言,我们所处的地位是特殊的,我们得到一些方程,但对于方程背后的原理却并没有真正深入的了解。这同爱因斯坦的引力理论― 广义相对论的发展情况刚好反过来。爱因斯坦是从一个十分优美的原理― 等效原理出发,从而构造了一些方程来进行研究。
    在弦理论中,我们有一组确定的方程,但对于导致这些方程的原理并不十分清楚。然而有一点是明显的,在我们所发现的所有这些令人吃惊的结果背后,存在一个非常深邃而优美的数学结构;一些十分美妙而深刻的原理正等着我们去发现。在过去的一两年里,有很多工作致力于澄清现状,一些最近的工作似乎正在指明正确的途径,但这仍是很初步的,在彻底弄清楚之前尚需作更多的研究。
    所以,不仅存在着构造理论使之与实验相符合的问题,而且在一个更基本的层次上,我们还必须深化对理论内容的理解。
    
    假使不断取得成功的话,您预计理论将如何与实验进行比较?到目前为止,我们已经有了一个很优美的理论,但要切实地获得成功,还必须给出一些可以检验的新预言。
    
    那是当然的,但即便真会那样,也无法事先知道什么时候会获得如此的成功。我希望在本世纪结束之前可以得到一些有关弦理论令人信服的证据,但我肯定无法对此做任何承诺。没人知道这到底要经过多长时间。我们所提出的问题是颇具挑战性的,这是一个雄心勃勃的方案,所以对于它能否成功我们毫无把握,尽管看上去它比任何以往的尝试更有希望。
    
    理论有没有可能预言一些可以在未来的加速器上找到的新粒子呢?
    
    设想我们对于基本原理的探索获得成功,也就是说我们可以得到方程的唯一解,那么由此我们就能够研究六维空间的拓扑性质,从而对低能下存在的粒子种类有所了解,并且仅仅根据拓扑的讨论就可以得出这些粒子的质量比以及它们相互作用的强度。
    这些数据都可以通过在实验室里进行的实验得出。可以肯定,会存在一些尚未发现的粒子,例如那些同超对称或对称性破坏有关的粒子。目前,我们对于这些粒子的质量以及其他一些性质仅有些粗略的了解。如果对于六维空间存在一种确定的紧致化方案以致于能够成功地解释已知的一切,那么大概同时也会对其他那些可以被实验检验的问题作出预
    
    我的印象是,要想取得进展必须对数学的理解有所突破,您所指的这些数学分支本身是新的,并且是理论进一步发展所必需的。
    
    是的。这是整个课题的一个方面,正是这方面令许多人望而生畏,这方面的研究要借助大量的数学工具。事实上,对此数学家似乎也还没有多少作为。还有许多未知的事情要了解,数学上的很多结果需要发展,同时,我们也正在努力去理解其中的物理。能够投身于此,令人备感兴奋。从长远来看,所有的付出都不会是白费的,对此我是乐观的。
    
    人们称超弦方案是一个包罗万象的理论,因为这一理论的最终目标是要解释所有的粒子与相互作用。人们常说,历史上不止一次地出现过这样的时期,似乎那个包罗万象的理论呼之欲出。然而,所有那些理论迄今都已证明是错误的。那么我们有什么理由认为超弦理论会与众不同呢?
    
    是这样的。在以往所有成功的(部分)统一理论中,人们能够描述当时所知道的粒子和相互作用中的一部分,而非全部。而超弦方案却期望解释所有的相互作用,包括引力在内。近年来,电磁作用与弱作用的
    
    统一取得了巨大的成功,并且随后又将强作用包括进来。这些工作是非常成功而令人兴奋的,但却不能说是无所不包的,因为很显然,这一理论把引力排除在外。
    
    过去所提出的那些试图描述引力的方案却又无法解释其他的相互作用。所以,超弦是第一个这样的例子(至少就我所知),它不但包括了引力并且看来同时还是描述其他作用力的一个可能的候选者。这一理论的数学结构十分精巧以致于你没有多大的选择余地。所以,一旦它能够成功地解释实验的话,就很难设想它会是什么更好理论的某种近似。并且我认为,它的结构很紧凑,所以除非你另起炉灶,否则就不可能对它做些什么修修补补。
    所以在这方面超弦理论的确不同于以往的那些理论。那些理论往往被认作未来更深层次的理论在低能情况下的某种近似。
    
    从乐观的角度看,设想整个超弦方案进展顺利,或许有可能在本世纪末对于我们所能观测的结果作出详细的预言,这样人们会渐渐相信,超弦正是描述自然界基本原理的理论。那么理论物理将会如何呢?是否将走到了尽头呢?
    
    我认为那只是一种逻辑上的可能性而已,事实上是不大可能的。依我看来,基本粒子物理是不同于物理学以及科学的任何其他分支的一个领域,这里所关心的是一个非常确定的问题,即自然界中的基本粒子和相互作用力是什么?它们遵循什么样的规律?这是一个原则上可以给出正确答案的问题。我认为所有其他科学领域则是没有终极的,人们总可以提出新的问题。
    从这个意义上讲,你刚才的设想在逻辑上是可信的。然而,我们已有的经验表明,一旦你成功地解答了一个问题,总会随之而产生五个其他的新问题。还没有什么迹象表明这种局面会在短时间内改变。所以,一方面我们希望对于基本粒子和相互作用有一个彻底的了解,这种理想最终可能会实现的;另一方面,我认为这将花费比15 年更多的时间,尽管可以设想在这样一段时间里有可能获得足够的成功以使人们相信已经找到了正确的途径。
    
    在所有那些描述自然界基本粒子和相互作用的尝试背后存在一个共同的假设,即我们可以用一种简单的办法通过各种数学上的技巧来描述自然界,并且我们期望最终会是简单的数学或者至少是优美的数学。这是否仅是某种虔诚的祈望?换句话说,您是否认为世界真的遵循这些简单的数学原则?
    
    看来情况似乎是这样的,至于个中原因,那是一个相当深刻的哲学问题,而我无法回答。任何事物都应该有某种符合逻辑的解释而数学则正是以这种符合逻辑的方式对事物进行描写的,这似乎是合情合理的。在我看来,这种信念主要是基于我们以往的经验,也就是说,数学迄今在描述自然界方面已经取得了巨大的成功,并且在一个越来越深入的层次上这一点仍然是对的。所以我想,认为这种局面会维持下去仅仅是一种逻辑上的外推而已。
    
    当然,可能存在这样的情况,当你达到一定的层次,比如说我们正在探索的亚原子层次,事情会暂时显得非常简单,但是如果深入下去却又会变得十分复杂起来。
    
    是的,许多人都有这样的看法。如果超弦理论不成功的话,这倒不失为一种可以采用的替代的观点。
    
    另一个令人关心的问题是所需要的数学过分复杂以致于凭人类的智力已无法对付!这也是我们不时会有的担心。
    据称,超弦理论是得到一个包罗万象的理论的最后希望,至少对于要求理论所用的数学简单且能被掌握这一点而言是如此。请问您是否这样认为呢?
    
    我不知道是否会这样。我认为人们对于过去的其他理论考虑过这种途径,但是如果由于某种原因而行不通的话,我估计会提出某些其他替代办法的
    
    在结束之前提一个有关您个人的问题,您是什么时候意识到自己正在向一个伟大目标迈进的呢?
    
    在我和米歇尔· 格林的合作期间― 我刚才提到过我们的合作开始于1 980 年― 我们有几个发现,其中我们认为比较重要的差不多每年有一两个,对此我们都很积极地予以发表并且在世界各地举办讲座与我们的同行们进行讨论。每次我都觉得这正是那个足以令人相信为所要寻求的重要课题,所以我感到相当吃惊的是,在一些年里理论物理界的其他同仁并没有对这一工作感到特别的兴趣,或者至少是他们没有表现出这种兴趣,如果他们真地有的话!他们对我们是友善的、宽容的,但可以肯定,并没有人加入进来从事这项工作。
    当1 984 年夏我们发现反常相消的时候,我已经很习惯于外界对我们
    
    工作所表现出来的这种态度,所以它后来实际得到的热情的反应是我始料未及的。我一直希望超弦理论最终会成为实现统一的一种重要方式,但我觉得这种转变将会是循序渐进的。实际上,1 984 年夏之后不到一年里已经有大量研究人员开始这方面的工作了。
    
    那么如今置身于由您的工作而生发的这一热潮之中,您感受如何呢?您是否觉得在某种意义上可以退后一步观其发展呢?显然您在这一领域内仍然十分活跃。
    
    我想继续积极工作并不断作出贡献。目前有众多高明之士工作非常出色,要与他们当中的一些人竞争不是件轻松的事情。一些年轻人尤其了解所需的许多现代数学知识并且做得相当好。当然,对于这样的进展我非常高兴,因为在只有我们两人工作时(尽管我应当提及,我们也曾与拉斯· 布林克(LarsB : ink )合作,所以有时候是三个人一起的), 令人沮丧的事情之一就是进展太慢。有很多有趣的问题,但我们没有足够的时间、精力和能力去全部解决,而我们迫切想要知道这些问题会将我们引向何处。现在则是发展得太快以致于要看完有关文献已不可能一一每天我都收到一大堆文章― 不做任何其他事而只读这些文章就足以占去一个人的所有时间了。
    
    3 .爱德华· 魏廷(Edwardwitten )
    
    爱德华· 魏廷是普林斯顿高等研究院的教授。在转向超弦之前,他对理论粒子物理学和量子场论,特别是量子色动力学以及更高维理论作出过许多重要贡献。他是超弦理论最明智与坦率的倡导者之一。
    
    请问超弦理论所要解决的主要问题是什么?
    
    20 世纪的物理学有两大支柱,一个是广义相对论,即爱因斯坦的引力理论;另一个是量子力学,即关于物质微观运动规律的理论,换句话说,是关于原子、分子以及称为基本粒子的那些更小的粒子的理论。现代物理学中的一个根本问题是这两大柱石不相协调。如果企图将引力与量子力学揉合在一起的话,那么从数学的观点看就将会得到毫无意义的结果。你可以写出自认为是量子引力的表达式,可是它当中包含各种无穷大。对于物理学家来讲,在计算当中出现无穷大是一件极不舒服的事情。
    大多数缺乏物理学素养的人或许会以为物理学家所做的无非是一些极端繁杂的计算而已,但事实上这不是物理学的精髓所在。重要的是,物理学注重概念、如何理解概念以及大自然运行的原理。在那些像广义相对论一样真正优秀的理论中,存在一个定义完好、概念清晰的表达式,在你了解了它的内容之后你会说“是的,这些概念是完美的”,而由此建立起来的理论是这些概念的完美体现。
    量子力学却略有不同。它的发展过程是相当曲折复杂的,并且一直是在实验的推动之下进行的。虽然它是一个内容非常丰富而美妙的理论,但却不具有像广义相对论一样的概念基础。
    我们在物理学中所遇到的问题就是一切都应该以这两种互不相同的理论为基础,所以当我们将它们结合在一起时,总是得到毫无意义的结果。物理学的历史就是不断发现更精确概念的历史,而自然界的规律正是基于这些概念的。当这些概念变得越来越精细时,理论也就只需要越来越少的原理来解释越来越多的事情,而同时,要写出内部自洽的理论也就变得越来越复杂。在牛顿那个时代,要解决的问题只是写出正确的理论― 他从来不会遇到无穷大这样一类麻烦的事情,但到了20 世纪我们有了广义相对论和量子力学这样概念丰富的理论框架。在这个框架内甚至连仅仅做到内部自洽都是困难的,更不要说正确性了。实际上,这是我们在物理学上取得进步的主要工具之一,从这个意义上讲是幸运的。物理学已经进入这样一个领域,其中实验并不怎么困难,当然它不再像五六十年前那样发展迅速。然而,存在一个以自洽性
    
    来提供约束的逻辑结构这一事实是我们仍然能够取得进展的主要原因之
    
    所以,对于弦理论而言,要记住的最为重要的事情就是它以克服数十年来物理学的核心问题― 引力理论与量子力学之间的不协调为目的。
    
    那么它如何克服这一不协调呢?
    
    整个20 世纪最让物理学家们头痛的问题就是,假如把电子这样的粒子当作点粒子并且认真计算它的电场与引力场,就会发现在电场与引力场中均存在无穷大能量。如今这个问题已经历了许多不同的发展阶段。它曾经是经典物理学家头痛的问题,在量子力学建立之后,一度又成为量子力学中涉及电场时的一个棘手问题。
    在电磁理论中解决这一问题的关键步骤是测不准原理,这一原理表明电子不再是一个裸的点粒子,因而讨论它的电场是有意义的。我们在讨论电子的引力场时,仍像一个世纪来大多数物理学家那样把电子当作点粒子是行不通的。然而,在弦理论中电子不再是点粒子而是一根振动着的小弦。振动的弦所具有的这一额外自由度使得我们能够解释它的引力场。我这里只是把电子作为一个例子。电子的无穷大自能是这类问题中最典型的,实际上所有基本粒子都存在类似的问题,而弦理论能够干净利索地处理所有基本粒子和相互作用的无穷大问题。
    
    所以我们不再认为世界是由粒子构成的而认为是由扭动着的小弦构成的,对吗?
    
    对的。我们在考虑粒子的时候应当记住,自量子力学诞生之日起世界上的所有一切都变得有些模糊,也就是说,与我们日常关于粒子的图像相比有些模糊。在弦理论中这一毛绒绒的粒子为一根小弦所替代。这是一根振动着的弦,另外,由于量子力学的效应,它也变得有些毛绒绒的。
    
    有很多种不同的弦吗?
    
    有几种可能的弦理论而大多数弦理论中差不多只有一类弦。你瞧,一类弦可以完成许多种不同模式的运动。你可以想象一把小提琴,当你在上面演奏时一根琴弦能以许多种不同的频率振动,称为谐音。对于一根小提琴琴弦而言,这些频率不同的谐音是决定提琴音色的关键,它是不同的乐器音色不同的原因,即使所演奏的是同一个音符。你可以在钢
    
    琴上或提琴上演奏音符C ,但听起来却大不相同,因为同样的弦可以因谐音不同而具有不同的振动方式。不同的乐器产生各类谐音的比例是不同的。
    对于一根提琴的弦而言,不同的谐音对应于不同的声音。对于一根超弦而言,不同的谐音则对应于不同的基本粒子。电子、引力子、光子、中微子以及所有其他粒子是同一根基本弦的不同谐音而已,正如提琴的不同泛音是同一根琴弦的不同谐音一样。
    
    把自然界的各种不同的基本粒子在某种意义上比作不同的音符是否过于牵强?
    
    不,这是一个非常好的比喻。
    
    这些弦的大小如何?
    
    以电子为例,与它相应的弦大概只有10 一23 厘米大小,所以比起你认为无法想象的小东西还要小得多。一个原子的大小差不多是10 一“厘米,一个原子核要比它小上10 万倍,而代表基本粒子的一根超弦甚至比核子还要小得无法想象。
    
    但无论如何它不是点粒子,这一点是至关重要的。
    
    是的,它不是点粒子。它不是点状物并且有确定的有限大小这一事实对于整个理论方案保持自洽是非常关键的。顺便我要说一下,尽管代表基本粒子的超弦小得令人难以置信,但是如果你有一把得心应手的镊子的话,从原则上讲你能够抓住超弦并且将它神得越来越长。是否会扯断要看具体的弦理论,但对于大多数弦理论而言,弦是不会被扯断的并且实际上可以神得像一个房间这么大,这样就得到一根宏观的超弦。它类似于当今物理学家和天体物理学家所讨论的那种所谓的宇宙弦,有些宇宙弦横越天空,或许用天文望远镜可以看到它们。
    
    您是否在暗示宇宙中可能存在宇宙弦,它们是大爆炸残留下来的超
    
    弦的遗迹?
    
    这是可能的,但我并不打算特别强调这一点。然而,在有些弦理论
    
    中原则上非常有可能存在横越天空的弦,你能够用望远镜看到它们。您是否可以就超弦的拓扑性质谈谈?
    
    在大多数弦理论中弦总是封闭的。所有的弦理论都包括闭弦,即形成环路的弦,而大多数的弦理论只包括闭弦,只有一个称为第一型的弦理论既包括闭弦也包括开弦。
    
    是什么首先吸引您投身弦理论研究的?
    
    大体上说是它所提供的调和引力与量子力学的可能性。在我进入物理学界之前这就是一个核心问题了。量子力学和量子场论是20 年代后期首先发展起来的。从一开始人们就很清楚,引力理论与量子力学之间存在协调性的问题。在早期量子场论中存在许多其他问题,物理学家们无暇顾及这个协调性问题,但在其他问题解决之后,引力与量子力学不相协调这一点就愈来愈成为理论物理的核心问题了,或许是最难克服的大难题。曾经有过一个时期,这个问题并没有引发太多这方面的工作,因为这个问题实在过于困难并且没有什么有趣的想法。
    弦理论之所以如此吸引人是因为引力无处不在。而所有已知的自洽的弦理论都是包括引力在内的,正如我们已经看到的那样,量子场论中是无法包括引力的,这一点必须由弦理论来解决。
    这只是弦理论引人入胜的一个方面。另一个方面是由它所产生的极为丰富的数学结构。我想这是非常重要的,因为,在过去的年代里物理学的进步总是伴随着不断丰富的数学结构。我个人认为,在调和引力与量子力学方面所取得的进步为理论物理学家们带来如此丰富的数学结构决非偶然。
    
    这一理论开创了什么样的数学领域呢?
    
    有黎曼(Riemann )面理论、称为李(L 曰代数的关于一定对称性的理论以及其他一些。数学中许多过去在物理学中无足轻重的领域如今在弦理论中已变得非常重要了。以往每当物理学基本理论取得重大进展时总会发生这样的事情,这次也不例外。
    
    您刚提到的那些领域是几何分支或者说是它的推广,对吗?
    
    这些领域主要是关于几何的,或许也是代数的。弦理论认真讲起来是或者说应当是一个新的几何分支。爱因斯坦在广义相对论中所取得的伟大成就即是把几何作为引力理论的基础,这种几何确切地说是黎曼几何。如果弦理论作为广义相对论的后继者有价值的话,它同样必须具有一定的几何基础,对此我们目前还只是略见端倪。但是,我们当中的许
    
    多人都坚信它的存在。
    
    您是否相信亚原子粒子的许多物理性质事实上有着一定的几何起因?
    广义相对论中引力是以几何原理为基础的,这实际上是一个最完美和令人满意的物理理论所应遵循的。自爱因斯坦以后,物理学家们的最大愿望一直就是要在物理学的其他分支中并且最终在某种统一的物理图像中达到同样的完美境地。
    我本人深信,弦理论的正确框架将被证明是爱因斯坦广义相对论几何概念的某种适当的完善。顺便说一下,我将考虑尝试进一步阐明这一恰当的几何推广在物理学中的核心地位,当然也必定是弦理论的中心问题。
    
    您是否认为我们能够用几何语言来理解诸如电荷这样一类量呢?
    
    我认为弦理论总的说来将会是一种几何理论并且由于它在解释各种相互作用方面的成功,它应该能够对像电荷这样的性质给出一定的可以称为几何的起因。
    
    如今您从事这一理论的研究已有时日,相信对于理论今后的趋向应有某种预感。那么对于它最终被证明为一种包罗万象的基本理论这一点您抱多大希望呢?
    
    我不喜欢就包罗万象的理论做什么猜测,但我要说,我确实相信,同以往我们在物理学中所取得的任何进步相比,弦理论正在把我们引到物理学的一个十分重要的新高度上。同时,我认为人们应该把它作为一种长期的进程来看待。应该记住,如果从维尼齐亚诺模型算起,弦理论迄今已有18 年的历史了。回顾过去,我们可以看到,在10 或15 年前人们面临许多有待解决的问题,今天的局面或许仍是如此。我们可能正处在某个将被证明是十分漫长过程的初期,这类似于我们得到量子电动力学时所经历的过程。量子理论始于1900 年普朗克关于黑体辐射的工作,在普朗克的原始工作中实际上包括一个公式,我们可以称之为电学的量子理论。然而,普朗克作为先驱而努力寻求的量子电动力学过了50 年才出现。
    
    所以,很有可能我们已经走过的18 年也仅相当于量子电动力学漫长发展历程的初期。
    
    它所给出的暗示是否同来自于量子电动力学的同样深刻呢?
    
    我期望关于弦理论究竟是什么这一问题的适当阐述将会给我们对于物理学基本规律的理解带来一场革命,其范围和程度将同以往历次革命一样。
    
    假如这一理论仍处于初创阶段,您能指明它迄今已取得的具体成果吗?是否它仅仅是看起来令物理学家们激动的漂亮数学理论,还是有什么更具体一些的东西?
    
    调和引力与量子力学之间的矛盾就是一个显著的成就。这一矛盾在我从事物理学研究之前很久就一直是物理学中最成问题的问题。
    
    您是说目前这种调和已经显而易见了?
    
    是的,我想可以这么说。我要说在现阶段,事实上,一些年以来这一点已经很清楚,即弦理论确实可以给出一种逻辑上自洽的框架以容纳引力与量子力学。但同时,正确地理解这一点所需要的概念框架尚未出现,它应该类似于爱因斯坦在他的引力理论中所发现的等效原理。我要提醒的是,历史已经表明,调和那些不相协调的物理理论是取得重大进展的一个非常有效的途径。如果我们考察一下20 世纪物理学的某些进展,我们就会注意到,爱因斯坦的狭义相对论正是源于对当时两大理论― 麦克斯韦的电磁理论和牛顿力学― 的调和。同样,爱因斯坦的广义相对论来自于对他的狭义相对论与牛顿引力理论的调和。最后,量子场论也是调和非相对论量子力学与狭义相对论的产物。因而,20 世纪许多最具深远意义的进展都是由于原有的理论不相协调。历史告诉我们,消除理论之间的不协调是取得真正重要进展的一个好方法。
    
    您认为理论目前所遇到的主要的突出问题是什么?
    
    作为物理学家,不仅要学会如何计算同时还应该懂得自然运行的原理。正如我已经指出的那样,物理学主要是发展概念的一种学问。目前弦理论中不能令人满意的一个突出问题就是,尽管这一理论有很多引人注目的特点以及大量了不起的发现,但是我们对于它所应具有的概念框架却知之甚少,这一框架应类似于广义相对论中的几何。我们迫切希望取得进展的中心问题就是设法阐明能够用来理解弦理论的那个逻辑体系。这个问题的解决可能要等上很多年。
    广义相对论是随着它赖为基础的那些原理应运而生的。一旦你同意
    
    将引力理论建诸几何,并且你能够从很少的几个可以用物理学语言形象描述的一般性原理(例如爱因斯坦著名的理想实验、升降机以及其他一些)出发来理解狭义相对论,一旦你抓住这些概念,数学也就随之产生了。数学是那些概念的完美体现,这再好不过了。
    我们所猜想弦理论中存在的,并且也是最希望发现的就是一个类似的概念化逻辑框架,有了它,理解弦理论就如同用黎曼几何理解广义相对论一样的自然。我们想要找到这一逻辑体系,因为无论如何,发现世界的运行规律是物理学家的天职。我们还想找到正确的概念化体系,因为很有可能关于理论的正确理解对于我们所要进行的计算举足轻重。我们要用弦理论来计算基本粒子的质量、藕合常数、寿命、相互作用以及各种过程发生的几率。只有通过这些计算并且同实验进行比较,我们才能知道理论正确与否。
    但是,很有可能在对一个理论的理解还相当粗糙并且对于其基础尚未了解之时,想要做这些计算将是困难的。我认为不仅理智上的收益肯定需要对逻辑框架的理解而且实用上的好处很可能也是如此。可以肯定地说,我如果有什么希望的话就是希望在这个问题上取得进展。
    
    假设实际上很难算出这些质量、藕合常数以及理论在低能情况下的数据的话,是否还有可能在其他能区进行实验验证?理论是否有可能预言新的粒子或现象的存在,而这些粒子或现象也许会在粒子加速器上发现?
    
    我对这个问题的理解是,如果能够用弦理论进行所有的计算,那么就能够在短期内通过实验来验证理论是否正确。同样地,如果能够在所谓的普朗克能量尺度上进行实验的话,由于引力与量子力学的基本问题在这一能区中会变得明显,所以应该能够很快地确定弦理论正确与否。
    
    但那是不能指望的,不是吗?
    
    不幸的是,无法指望在这样高的能量区域进行实验并且目前也无法指望计算出所有的一切。所以,能否在彻底弄懂弦理论之前就找到某些幸运的事件以便做出非同寻常的预言还是一个问题。尽管这种情况是可以设想的,但对于今后几年内就将发生我并不过分乐观。
    
    难道没有关于新粒子或者诸如此类的事情的预言吗?
    
    不,许多弦理论以及有关各种弦理论的许多模型都预言存在具有分数电荷的非囚禁粒子,它们的质量几乎落在普朗克能区,可以设想会在
    
    宇宙射线中发现它们。
    
    我们这里所谈的粒子比起已知的基本粒子要重很多吗?
    
    是的,我们谈论的粒子每个差不多都有一个细菌那么大的质量。
    
    但是却有很特别的标志,它们所具有的电荷大小只是原有粒子电荷大小的若干分之一。
    
    是的,只是电子电荷的若干分之一。
    
    并且这些粒子或许是大爆炸所遗留下来的?
    
    我们发现这些粒子的唯一希望寄于它们会从大爆炸中残存下来。可以做一个粗略的估计看看宇宙射线中会有多少这样的粒子。我们知道在太阳系附近有多大质量并未计算在内,这些质量属于那些所谓的暗物质。在最为乐观的情况下,如果这些暗物质是由那些普朗克质量大小的分数电荷粒子构成的话就有可能在磁单极探测器中找到它们。这就是我刚才说的幸运事件的一个例子,因为没有人能够断定这些暗物质是否真的由那些粒子构成。我肯定还存在许多其他的可以称为幸运的途径尚未被人们想到,但我不愿冒险对什么时候会碰上运气作出猜测。
    
    您一直在说各类弦理论,而这似乎与所声称的有些自相矛盾。人们常说弦理论方案的一个优美之处就在于它并未提供太多的选择余地。那么究竟有多少种弦理论呢?
    
    要正确地看待这一点,就应当记得在传统的量子场论中有无穷多种可能的理论。不夸张地说,理论物理学家对它们当中的数千种都已作过认真的考虑。相比之下,目前的弦理论情况就好得多。差不多只有4 或5 种也许有6 种自洽的弦理论,到底有几种要看如何去分类。
    
    那么什么样的分类标准可以用来把这个数目降下来呢?
    
    对于已经从数百万,或许数千要么就是无穷多种理论裁减到目前的差不多五六种这样的结果我们暂时应该感到满足。即使就此止步不前的话我们也仍然会相当知足的。
    超弦理论的特点之一在于该理论所讨论的弦并未存在于我们所感受
    
    到的通常的三维空间与一维时间而是存在于一个更高维的世界里,这在有些人看来可能有些难以理解,那么承认这些额外自由度的存在是否合乎情理呢
    ?
    由于海森堡的测不准原理以及量子力学的基本观念,自然界的一切事物都是有一点模糊的。假如确实存在某些额外的自由度而它们是如此之小以致于被测不准原理带来的模糊所掩盖的话,那么想要察觉到这些额外的自由度就会格外费力。这也就是说,如果额外自由度太小的话就无法察觉到它们的存在。
    可以这么说,额外自由度这种想法对于任何没有学习过物理学的人来讲听起来多少会有点陌生。所有专业性地从事物理学研究的人都清楚,有很多事情比额外自由度还要奇妙得多。广义相对论是奇妙的,量子力学是奇妙的,反物质是奇妙的,所有这些都是奇妙而真实的。与物理学中以往众多的最终被证明是正确的奇妙概念相比,额外自由度并非离经叛道。
    
    我们能否了解这些额外自由度是如何卷缩到一个如此之小的尺度上的呢?
    
    我们可以设法去理解这一点,并且我们看到通过一些关于额外自由度如何卷缩的简单假设就可以得到粒子物理的有趣而可信的粗糙模型。我不认为在对弦理论本身有一个更好的了解之前能够确知额外自由度是如何卷缩起来的。对理论的真面目只有一个极端简化而粗糙的概观这一点对于取得进步是极为不利的。
    爱因斯坦在建立广义相对论的时候所需要的几何中的基本概念早在19 世纪就已发展完善了。曾经有这样的说法,认为弦理论是21 世纪物理学的一部分,偶然的原因使之提前到了20 世纪。这是15 年前一位最具影响力的物理学家所作的评论,他的意思是说地球上的人类永远不会具有这样一种概念体系使得他们能够主动发明弦理论。弦理论基本上是在一系列的事件中偶然被发明的,这一系列的事件始于1 968 年的维尼齐亚诺模型。没有人刻意发明它,它的被发明是由于一起幸运的偶然事件。按理说,20 世纪的物理学家们是无此殊荣去研究弦理论的。按理说,弦理论的发明应该等到对于一些预备知识的了解足够成熟以致于我们有可能对于整个理论的内容有一个正确的观念。
    
    我们需要21 世纪的数学?
    很可能。按理说,事情的发展本来应该是这样:正确的数学结构要
    
    在21 世纪或22 世纪才建立起来,然后物理学家才最终发明弦理论,作为一个物理理论它将依赖于那些数学结构。如果事情真是如此的话,那么最先从事弦理论研究的物理学家们或许就能够像爱因斯坦发明广义相对论时那样知道自己在干些什么。也许这样才顺理成章,但如此一来20 世纪的物理学家们将没有机会研究这一令人神魂颠倒的理论了。事实上,弦理论已经被发明了,而这原本是地球上的人类做不到的事情,所以从这一点上讲我们是幸运的。无论如何我们是幸运的并且我们正在试图充分利用这一优势。但是我们也为此付出了代价因为我们无法以通常的途径得到它。
    
    再问一句,关于高维自由度,有多少种不同的方式卷缩起来呢?
    
    对这个问题进行研究的物理学家们已经设想了很多可以想象得到的方式而且很有可能还存在一些尚未想到的。事实上,整个过程将会比我们迄今所想的更加微妙。
    
    那么这些不同的方式是否都为理论所允许呢?换句话说,有没有办
    
    法遴选出一种特殊的卷缩方式呢?
    
    我认为在找到正确的途径之前需要对所处理的是什么样的理论这个问题有一个更好的认识。
    
    这样看来,不了解高维自由度如何卷缩很可能是理论进展中的一个相当主要的障碍。
    
    如果能够了解高维自由度是如何卷缩的并因此得知理论的真空态是什么的话,我们会更加高兴的。
    
    只有知道了这些情况,才能够计算粒子质量,电荷或者诸如此类的一些量的细节?
    
    是的。然后我们将如愿以偿地算出基本粒子的质量、寿命和相互作用等等。遗憾的是目前我并不认为在不久的将来会有人做到这些。我觉得在具备做这些事情的能力之前,对于弦理论我们尚有相当多的东西要学。而那只是我的推测。关于额外自由度如何卷缩有很多想法并且许多人在已知的方案上做些可能的变形,修修补补,同时令人感兴趣的想法也不断涌现。事实上,我昨天还刚刚听到一种新的想法。
    
    假如这个中心问题暂时悬而未决,那么目前研究的主要方向何在?
    
    许多物理学家目前正以不同方式研究一个在我看来是基本的智力问题,即弦理论究竟意味着什么。与其他物理理论所遵循的对称性原则相对应的是什么?
    
    会匕之5
    
    认为,竟然有这样相当数量的物理学家对一个或许再无法与实验相联系的理论如此关心多少不能令人满
    
    百能
    
    一些人可
    
    过一代人也很可
    
    意。您认为对这一课题的极端重视是恰当的吗?
    
    我说的只能代表我个人的观点。我觉得当此弦理论不断发展之际投身物理学是一件极值得的事情。我本人相信,在未来的世纪里人们回首过去会说今天是做物理的大好时机。
    
    回顾过去50 年,您是否认为研究基础物理理论的方法在不断改变?在超弦理论中所运用的技巧与哲学思想是否与以往的理论有根本的不同呢?
    
    由于许多原因,物理学的风格在这些年里已经大为改观,最主要的还是因为理论物理已进入了一个新的领域。差不多50 年前,量子场论还完全是一团乱麻,后来随着时光的流逝人们获得了进展。随着对它不断深入的了解,逐渐地它被带入一个新的领域,能够包容新的相互作用,并且赋予某种几何意义。这种几何解释尽管没能达到广义相对论那种程度,至少也是一个有价值的竞争对手。
    
    物理学的进步使人们有可能更为热切地寻求对于一个物理问题的满意解答。记住这一点是有益处的,在19 世纪,物理学家们甚至对解释玻璃为什么透明,青草为什么是绿色的或者冰为什么在一定温度下融化等等这样一些问题也不抱任何企望。这些问题在19 世纪是不属于物理学范畴的,而且那时的物理学家做梦也想不到能够解答这样的问题。他们的愿望要有节制的多,给定关于某种材料柔韧程度的度量,他们所希望做到的无非是能够算出其他一些特定实验的结果,至于由电子和核子满足的基本方程式对所有一切做出预言只是到了20 世纪才成为可能,而这在19 世纪是连做梦也想不到的。
    物理学的进步从来就是这样的,一代人的认识水平在前一两代人看来是连做梦也想不到的。20 年前基本粒子物理也是一片混乱,有大量基本粒子被发现而对于正确描述这些粒子的理论则完全不清楚。引力除外,对于其他已知相互作用的一个令人满意的理论框架是在1 970 年前后
    
    出现的。它在基本粒子的一片混沌中建立了秩序并为在这一领域内思考问题提供了新思维。所以,如果目前我们手头有不同类型的问题并且用不同的方法处理的话,那么大体上是由我们在这一阶段― 当然还有更早时期― 所获得的进展所致。
    
    尽管并非从事超弦而是处理与之类似的那些非常基本的问题,斯蒂芬· 霍金却声称理论物理的终极已经在望了。您是否认为,如果超弦在大概50 年内获得成功的话,那么它将成为理论物理的顶峰?它是否将一古脑儿地把所有的一切都概括在内了呢?
    
    在原子层次上量子力学的第一个真正的尝试是1 914 年玻尔的氢原子模型。在那之后,在原子层次上正确的理论应该是像量子力学一样的理论,这一点已经很清楚,但对于量子力学的具体理论还不清楚。曾经有一段混乱的时期,对于量子力学在多大范围内适用完全不清楚。事后表明,量子力学的适用范围比任何人想象的要大得多。只是在1 925 年薛定愕方程建立之后人们才开始逐渐意识到量子力学能解释什么不能解释什么以及它给人类思想带来多大的影响的问题。
    
    我想,对于弦理论我们处于一个类似的时期。我认为,即使是那些对弦理论抱极大热情的人也还是会低估了它在我们认识物理规律方面的巨大影响。我们正在揭示整个结构的一部分但尚未找到核心部分。同量子力学的情形相似,我认为,在认清弦理论的真实面目之前是很难预见它之后的理论物理的前景的。我想,理论物理届时会达到我们今天无法想象的那样一个程度,那时的问题会怎样,我不想猜测。
    
    4 .米歇尔· 格林(MichaelGreen )。
    
    米歇尔· 格林是伦敦玛丽皇后学院物理系教授。作为现代弦理论的创始人之一,从很大程度上讲正是由于他与约翰· 施瓦茨合作的工作使得弦理论这一课题发展到今天这样的突出地位。
    
    让我们从弦理论的早期谈起,那时它还是一个毫无生气的课题。您能否谈谈最初是如何涉足这一领域的,并且在当时您打算做些什么呢?
    
    好的。弦理论的历史很奇特,因为它起源于另一个完全不同于今天人们感兴趣的物理领域。当时弦被用来描述强子― 像质子和中子这类强相互作用粒子。粗略地讲,可以认为这些粒子是由夸克通过弦连在一起而构成的。这一课题在60 年代后期在强作用物理中引起浓厚兴趣主要是由于意大利物理学家加布里埃尔。维尼齐亚诺的工作。当时我正在做我的博士论文,我立刻就被这些新想法打动了,这部分是因为这些想法完全不同于量子场论的传统思想,而那些传统想法没有办法解释强作用现象所涉及的物理。
    
    姑且可以这样说,应用于强子物理,弦理论看来似乎是一类奇怪的模型,想象一下吧,在强子的内部有一些微小的弦。这看上去不怪异吗?您真的认为这种图像能够描述客观实际吗?
    
    并不是在粒子内部存在着弦。当时的想法是认为粒子本身就是弦状的。比如一个JI 介子,作为最基本的强相互作用粒子之一,粗略地讲可以认为是由弦连接在一起的夸克和反夸克。夸克之所以未被单独观测到其部分原因就是它们被弦连在一起。
    
    这种样子有一点儿像哑铃,并且我猜想,这些拴在一起的夸克反夸
    
    克对是整体高速运动的?
    
    是的。事实上这正是如今出现在那个称为量子色动力学理论中的图像,这是强作用的现代理论。在某种意义上也可以用较早的弦图像来理解它。
    所以在强子的更为现代的图像中仍可见到弦的某些痕迹?
    
    是的,我想这个说法好。
    
    像质子那样内部有三个夸克的粒子情况又是怎样的呢?是不是需要有三根弦来把它们束在一起呢?
    
    是的,并且正是因为类似这样的问题以及其他一些非常严重的技术性问题,弦理论在这方面的应用才最后以失败告终。
    实际过程事实上比我刚才讲的甚至更为离奇,因为维尼齐亚诺的原始建议主要是对强子碰撞中可能发生的事情作些猜想。当时他思想中并没有弦这样一个图像,但他的这一具有启发性的猜想使很多人投入对这个模型的结构的研究。最后,大约过了两三年才认识到如果把粒子想象成弦状的就会出现这种结构。所以是花费了些时日才意识到他的猜想是基于这样一种图像:粒子是弦。
    照您这么说,很显然,尽管曾经有过一些成功的迹象,但弦理论的这一特殊应用并没有怎么深入下去。那么后来的情况呢?它就这样被人们遗忘了吗?
    
    它的早期发展,也就是70 年代初期,适逢人们对弱作用认识的飞跃阶段,这一革命性的飞跃来自于弱作用与电磁作用的一个统一理论。这时,对于强作用的认识也有很大进展,只不过是以一种相当传统的语言,即量子场论的语言,它是研究相互作用的基本工具。由于当时人们对这些相互作用的理论研究方面所取得的巨大理论进展以及在验证这些理论的实验中的一系列成就,我想人们当时的注意力是在这些更为传统的课题而不是弦理论。
    但也正是在这时候有一部分人醉心弦理论。弦理论,一旦你了解了它,是这样的诱人,这样的美妙以致于你无法释怀。我想这比将它应用于物理学的任何具体领域更能打动所有那些研究它的人们。
    
    这是为什么呢?什么是弦理论成功的奥秘?为什么它如此迷人?
    
    这部分是因为弦理论中包括有我们所熟悉的人们通常认为是美的量子理论所具有的结构。例如,理论物理学家们偏爱规范理论,而像电动力学和强作用理论,实际上包括爱因斯坦的引力理论在内的这样一些理论都是规范理论。这些理论被认为是非常优美的理论,因为它们实现了一种对称性,称为规范对称性,使得理论非此而不能自洽。
    
    我们这里所谈到的是关于某些特定的数学对称性质,它们在理论物理学家看来是明了的,但普通人却不太了解。这些性质使理论物理学家
    
    们赏心悦目。
    
    是的,从某种意义上讲是这样的。以一种与爱因斯坦的狭义相对论相协调的方式来描述像弦这样一些扩展了的对象是非常困难的。初看上去,理论将碰到棘手的问题,使得人们认为这可能会导致理论不自洽。
    
    是哪类问题呢?
    
    最为显然的是这样的事实,表面上看,理论似乎描写具有非物理振动模式的弦。它们不是在空间中而是在时间里振动,它们在类时方向的摆动是毫无意义的。
    
    早期弦理论令人着迷之处就在于,尽管它们事实上存在这一明显的问题但却能以某种方式回避它,这令人们想起在麦克斯韦的电磁理论中也曾回避过类似的问题。然而它们所采用的方式却是非常精细的,因为所遇到的问题比那时更为复杂。能够做到这一点是值得称道的。
    
    是如何做的呢?
    
    为了避免这些明显的不自洽性,理论只有在满足一定的条件时才有意义,也就是说,要求当弦在时空中运动时空间的自由度应为固定值。在早期弦理论中,空间必须有25 维,而时空总共是26 维。在后来的弦理论中则要求空间为九维,时空为十维。
    
    早期人们如何处理这件事情呢?
    
    它被当成是一种灾难,因为,我们生活的世界看来只有三维空间加一维时间。所以在当时这一点对弦理论是极为不利的。
    还有另外一个问题,我认为是更为严重的,因为这才真正是一种不自洽。就是说,这些理论中含有运动速度比光速还快的粒子,即所谓的快子。如果你不怀疑量子力学的话,那么存在比光速还快的粒子这件事就值得你认真思考,因为在一个量子理论中看来是无法赋予这种粒子任何物理意义的。
    
    有些人比较熟悉相对论,在相对论中如果有什么东西运动比光还快的话,从因果关系来看可能是糟糕的事情。
    是的,但这并不完全是坏事。在一个经典理论中可以这样来克服这
    
    一困难,即不允许超光速系统与亚光速系统接触。真正的问题出在量子力学理论中,因为如果存在快子则系统的最低能态的概念是没有意义的。那种我们认为是真空的状态将是不稳定的,因为它将衰变成这些粒子。换句话说,真空将破裂成为无数个快子。因此,我们不知道如何使得包含这类粒子的理论具有意义。
    
    所以,这就是70 年代中期的状况,对吗?
    
    是的。在70 年代中期,许多研究这一课题的理论家都为那时发展起来的比较传统的理论的进展所吸引,正像我先前所解释的那样。可以相当准确地寻迹这些物理学家的活动,大体上讲,他们形成两大研究派别。那时,对于某些传统的规范理论问题的理解正在加深,这些问题在数学上大大超出了人们先前的理解,比如说,人们通过一种非常妙的途径认识到这些理论是如何包括了磁单子― 磁荷的,而这是先前从未预料到的。理论研究的第二个新方向是超对称。
    
    超对称究竟是什么?
    
    对称原理在基本粒子物理的发展过程中扮演了一个非常重要的角色,这主要是因为这些原理揭示了把貌似不同的各个粒子联系起来的那些性质之间的关联,并且人们一旦有了这些图像就相应地得到了关于那些潜在的相互作用结构的线索。科学上运用对称性的一个范例就是19 世纪的化学。在上个世纪,门捷列夫(Mendel eev )认识到可以把化学元素按照共同的特性分门别类。
    
    那就是著名的元素周期表吧。
    
    是的。周期表中有各族元素,它们以一定的方式排列在一起,于是人们认识到这许多元素之所以能够如此排列原因在于它们是由原子构成的。可以看出这些排列方式是由于对这样一种现象的认识,即对于把电子束缚在绕原子的轨道上的电磁力的认识。
    粒子物理学的目的就是要通过把粒子按它们的性质归类从而能够获得关于其潜在作用力的线索。
    对于粒子之间相互作用力的研究已经取得了巨大的进展。现在,我们有了关于强力与电弱力的知识,粒子在这些力的作用下以一定的方式归类。然而,就70 年代中期而言,主要分成截然不同的两类。粒子具有称为自旋的性质,这是一种角动量,简单讲可以认为这些粒子在绕某个轴自转,但在量子力学里这种自旋只能取分离值。具有整数自旋的粒子
    
    称为玻色子,而那些自旋为半整数的粒子则称为费米子。
    到70 年代中期,尽管费米子和玻色子各自的分类已经清楚,但人们尚不了解在什么意义上费米子和玻色子彼此可以通过某种对称性联系起来。换句话说,费米子与玻色子看来似乎是完全无关的。而在另一方面,如果我们想要以一种单一的原则对所有粒子的起源有一个真正根本性的认识,则我们确实希望了解这两类粒子之间的关系。
    而70 年代中期出现于理论中的超对称就是把费米子与玻色子联系起来的对称性,于是,如果超对称是物理规律所具有的对称性,那么这明显不同的两类粒子其实就是同一事物的两个不同方面而已。
    
    那么超弦中的“超”字也就是来自于理论所具有的超对称性,是吗?完全正确。
    
    把超对称应用于旧的弦理论会有什么效果呢?
    
    效果是惊人的。把旧的弦理论中的任何一种进行修改使之具有超对称性,立刻就可以消除理论中存在比光速还快的粒子所具有的问题。这类粒子将不再出现在理论当中,并且从这一点上讲,理论显得自洽了,而这在原来是做不到的。
    
    在那个时期是否普遍承认你所得到的是真正非常激动人心的结果
    
    呢?
    
    从1976 年到1979 年,弦理论的发展曾经有过停顿,这期间差不多所有的人都停止了这一课题的研究。现在看来是相当奇怪的,因为事实上在1976 年也就是恰好在那段停顿的前夕所发表的一篇论文〔 由乔尔胡寸尔克,弗迪南德· 葛里奥兹(Fe r d 1 nandG 1 1 0221 )和大卫· 奥利夫( Davido } ive )撰写〕 中已暗示存在这种可能性,即对弦理论作超对称化修正将会是有意思的,但是无人响应,于是这一课题或多或少地停止了。然后,在差不多1979 年前后,我在加州理工开始了与约翰· 施瓦茨的合作,我们按照把弦理论超对称化的想法行事,吃惊地发现这样一来理论看起来自洽了。我得承认,当时我们的同行中没有几个感兴趣,这同样主要是因为在另一个看来令人非常感兴趣的领域― 超引力领域中有了重大进展,也就是把超对称应用到引力理论中。另外,在圈内人看来弦理论根本不值得花力气去了解。
    
    是什么诱使您把超对称同弦理论结合在一起的呢?我能够理解您对弦理论的明显兴趣,但去尝试将它超对称化是显而易见的吗?
    
    我觉得这不失为一种办法。我是说在那些日子里每个人都在将每一种东西超对称化!超对称看来是物理学中的一个漂亮想法,因为它确实在一般的意义上是将不同类型的粒子统一起来的最后一个环节。在这以前,对于对称性如何将不同自旋的粒子联系起来人们一无所知,而超对称填补了认识上的这一空白。所以从一个纯理论的角度看,似乎像超对称这样的概念对于任何企图统一各种不同粒子的理论而言都是基本的,尽管迄今尚未在自然界中找到任何一个有关该对称性的实验证据。
    
    当您着手这项工作时,是否期望会得到戏剧性的结果呢?或者说,事情进展如此顺利您是否感到意外呢?
    
    在开始的一两年里我觉得我们不断地研究这些理论只是因为沉醉于这样一种希望,希望通过越来越深入的研究就能够得到更多的途径使得理论自洽起来。
    有一段日子我们开始相信我们取得了非常重要的进展,那是在1 981 年末。我们证明了,由这些超弦理论之一所作的某种确定的量子力学运算的结果并非毫无意义。我这样说是因为这些理论中包含引力,而到那时为止引力的所有量子理论从我现在所指的意义上讲都是毫无意义的。
    
    所谓毫无意义是在何种程度上而言呢?
    
    如果你尝试用这样的理论来计算两个粒子散射的几率则注定会得到无穷大的结果,这就是我所说的毫无意义。
    
    那么在您的计算中是得到了一些有限的结果曦?
    
    我们发现至少在我们所研究的其中一个理论― 这是一个只包含闭弦的理论― 的最低级近似之下理论的确是有限的。这是非常令人震惊的,因为毕竟这是一个含引力的理论。基于爱因斯坦广义相对论的传统引力理论存在严重问题,哪怕是在最低级近似之下也是如此。所以,对于有可能发现了极有兴趣的结果这件事我们着实有些激动。
    
    我来仔细想想,有限的结果来自于一种只是近似的计算,您能说理论就一定自洽吗?
    
    不能。当然,我们刚才所讲的与现在讨论的情况都是指在对完整理论取某种近似意义下的有限。
    
    人们对于这一开创性的发现反映如何?
    
    很少有人注意。可以肯定这一结果应该得到一定的关注,因为在当时正有相当多的人在从事超引力理论的研究,它同样也是一种描述包括引力在内的自洽量子力学的尝试。但是实际上人们并未注意我们的工作。有一两个人肯定是被我们的结果所打动,特别是爱德华· 魏廷,事实上,他和路易斯· 艾瓦利斯一高梅(LouisA } varez 一Gaum 曰进一步证明了这一理论极有可能不仅是有限的而且也不存在困扰量子引力理论的另一个头痛问题― 反常。所以,他们受到鼓舞而想弄个究竟,这表明他们是很感兴趣的。但我想,他们应该是个例外。绝大多数的人只是感到弦理论与传统的量子场论相去太远。
    
    正如您刚才讲的,至少在一级近似下您所证明为有限的理论是只含闭弦的。但是在当时闭弦被认为对于那些将要告诉我们真实世界究竟为何物的理论来说是一个没有用的概念,是这样吗?
    
    是的。平心而论,对于那些忽略我们工作的人们来讲,这一只包含闭弦的特殊理论的确看上去不大可能同除引力以外的物理学规律发生关系。除了引力以外这个理论并未具有足够丰富的结构以使人们相信可以用它来描述其他的相互作用力,并且对于这一理论本身也并非一清二楚。事实上,这是相当奇怪的,因为大多数新近发现的弦理论也都是只包含闭弦的。如果愿意的话,可以把它们当作我们当时所考察的理论的推广。这些推广确实具备了更丰富的结构,而这些新类型的理论称作异常弦理论(heterot 1 cst : 1 ng theo : 1 es ) ,大多数人认为这些类型的理论可能描述另外几种力。
    
    也许我们应该更多地讨论一下有多少种不同类型的弦理论。初听起来这是个坏消息,因为一般讲,如果要寻找的是一个包罗万象的理论那么通常希望它是独一无二的,而弦理论究竟有多少种不同的类型呢?
    
    那要看是怎么个算法。对于某种特定的算法,有四五种,但是老实讲可能没有什么绝对的算法,所以采用某种不同的算法甚至可以得到几千种。
    我之所以说依赖于不同的算法是因为这数千种类型的理论实际上都可以认为是四五种理论的不同翻版而已。
    我想指明的是,尽管看来存在如此之多的版本是件糟糕的事情,但这一课题其实尚在初期,每当你探究弦理论的一些其他方面的性质时,
    
    总是发现存在不得不克服的新的明显的困难,而为了消除这些问题理论必将比最初设想的要更为具体。
    
    您是说弦理论的这许多可能的版本在某种意义上讲尚不完全,而在
    
    对它们的机制有了一个全面的了解之后或许发现其中一些是自毁性的,就是说它们无法对客观世界作出自洽的解释。
    
    这是我的主观设想,但却是来自于对弦理论早期历史的经验。例如,在1 982 年前后,当我们因为这些理论看来是有限的而激动不已之时我们曾经一度认为所有超弦理论都将是有限的。当时我们认为可以有选择地引入任何的对称性以便能够包括自然界中除引力以外的其他相互作用。在70 年代后期,相互作用的统一工作所处的局面是人们感到应该存在某种他们称为大统一的理论或大统一对称性,这种巨大的对称性是一组数学关系,它们将自然界中我们发现的所有粒子联系在一起并且可以将除了引力以外的所有相互作用纳入一个框架。
    在传统的大统一方案中,人们基于实验数据而不是出于任何理论上的考虑选择了一种特殊的对称性。在那时,挑选某种特别的对称性并没有任何理论依据,所以,正是由于深受这一影响,我们认为同样可以在超弦理论中引进任意对称性来描述那些非引力部分之间的联系。所有的选择从理论上讲都是同等的而我们同样依据实验的情况来作出最终判决。但是,实际情况并非如此。所以此例表明,尽管看上去存在无穷多可能的理论,它们具有各不相同的对称性,但事实上我们后来发现,真正自洽的理论的数目是非常有限的。
    
    那么异常弦与1982 年您所发现的那些弦有什么不同呢?
    
    异常理论是相当奇妙的。可以认为它们是最早的弦理论即原来所谓的玻色弦理论与超弦理论的复合体。所以,异常弦将26 维时空中的弦理论同十维中的弦理论结合在一起!当然这是没有意义的。对于相同的弦时空维数不可不同。实际的情况是,26 维中的十维作为普通的时空自由度,这样,异常弦就是在十维时空中运动,而同时还具有16 个所谓的内部自由度。它们所带来的额外结构应该描写除引力以外的其他力。所以这些力的来源是具有一个相当几何化的图像的。这实际上是因为26 减10 等于16 !这相差的16 维自由度表明理论的某些特定对称性。它们称为50 ( 32 )和ES X ES ,是理论中粒子之间关系的数学名称。50 ( 32 )和Es X 〔 8 是数学上的对称群,在异常理论中它们自然地同玻色弦理论与超弦理论相差的16 维自由度相联系。
    
    所以,异常理论中额外的16 维自由度以某种方式同非引力相互作用相联系,对吗?
    
    是的,异常超弦理论同其他有可能与物理相联系的弦理论(即一些开弦理论)之间的差异在于,开弦理论中与作用力相关的荷― 比如电荷、强作用力的荷等等― 位于弦的端点,而异常理论中弦是没有端点的,它们是闭弦,可以设想荷是涂抹在弦上。这是两种理论之间的主要物理差别。
    
    您如何用闭弦的图像来描绘一个电子?电子是带电粒子;可以把电荷当作是涂抹在弦上的吗?
    
    在一个异常型闭弦理论中,类似的图像是正确的。弦可以以无穷多谐振模式中的任一种方式振动,并且任一特定的频率对应于一个粒子或是一组粒子。于是,我们在自然界中观测到的粒子,诸如电子、夸克、光子、其他粒子,所有我们能观测到的粒子其实是弦的最低可能振动模式,从某种意义上讲在这些模式下弦根本没有振动。
    
    您说自然界中的粒子全都对应于弦的非振动模式。那么,在非振动模式下弦是如何能够描述众多不同类型的粒子的呢?
    
    对于弦理论而言,不仅仅只有弦在空间振动这样的简单图像。在最早的弦理论中,这种简单图像是对的,但这样的理论没有包含我们观测到的粒子,并且它还有其他一些不自洽之处。在更为现实的理论― 超弦理论中除了弦在空间振动外还有其他结构。弦上有电荷、弱荷以及其他一些荷,正是这些荷的性质区分了像电子,中微子,夸克等一些不同的粒子。所以,弦在空间的任意给定振动方式对应于一组粒子而非仅仅一个粒子。特别地,弦的基态,也就是零振动态,同样不只描写单独一个粒子而是一群粒子,我们假定能在实验室中观测到它们。
    
    如果为了解释粒子的多重性而不得不依赖于荷在弦上可以以不同的方式分布这一概念的话,是否有损于弦理论的一个优点,即用几何语言解释一切,这当然也包括电荷在内?
    
    我用这种语言对理论所作的描述只是为了直观地理解所发生的事情。对于荷的分布并不是任意的。这些荷的产生以及如何分布是由理论明显确定的。只有那些以非常确切的方式分配特定荷的理论才有可能同量子力学相协调。以随便的方式对荷进行分配的弦是不可妄谈的。只有
    
    特定的理论才是自洽的。
    
    有多少种不同的荷?
    
    我们开始讨论的理论含有50 ( 32 )和ES X ES 对称· l 生,这些理论有16 种不同的荷,496 种类似光子的规范粒子,它们是用来传递由那些荷产生的相互作用的。在这些理论中人们不能随便作什么更改。这一特点使得弦理论不同于原来基于点粒子的理论。
    
    所以您所谈到的这些荷并没有必要同大多数人所理解的荷,如电荷有什么对应关系?
    
    它们包含电荷,还有弱荷和强荷这些概念在内。在对某种统一描述的探索中人们尝试建立这样一个图像,其中不同种类的荷被统一在一个更大的结构里,而这一更大的结构除了可以直接在实验室中观测到的以外还描写许多其他的荷。它们当中的一些我们或许最后能够看见而有些可能是一些质量很大的粒子以致于我们永远无法看到它们。两三年前所说的弦理论有496 这样众多的规范粒子,这当中有我们见过的但更多的尚未看见过。
    
    据称,曾经有一次在您的计算中出现了496 这个数字,当它出现时您突然意识到发现了重要的东西。您能就此谈谈吗?
    
    可以。在1 984 年夏天,我和约翰· 施瓦茨曾努力地思索这样一个问题:这在物理学中可能会令人感兴趣的弦理论从含不含有反常的角度看是否自洽呢?1981 年和1982 年的工作已明显地说明了闭弦理论是自治的,但这并没说明它同物理学有什么直接的联系。我们所希望的是开弦理论,是那些能彻底避开反常问题、在物理上令人感兴趣的理论。然而这些希望目前还仅仅停留在想象上,我想其他大多数人都认为弦理论总是存在反常问题的,因为,从最普遍的意义上说弦理论似乎是不可能避开反常的;另一方面,我们又非常乐观,觉得弦理论是如此奇妙,它总能避开反常问题。然而,我们却惊奇地发现实际情况正介于这两者之间。事实告诉我们几乎所有的弦理论的确都是有缺陷的,它们几乎都具有反常,但在我们寻找的理论中有一个独一无二的理论是能够避免这个问题的。在我们发现自己受骗的同时,我们所使用的方法却提供了存在巧合的可能性,因为在众多的理论中我们所研究的偏偏能够避开反常。终于有一天,我们进行了更为复杂的计算,其中我们同时考察了所有可能的反常,而这一计算可行的条件是在许多很特别的数字之间必须存在一种
    
    巧妙的抵消关系以使它们加起来的总和正好是496 。而事实正是如此!
    
    您能否告诉我目前关于这些理论有限与否的论断是什么?因为我得到的印象是人们对此尚有争议。
    
    是的,这一局面还没有澄清,但我认为对于人们想象的即将发生的事情是存在某种共识的。人们对于弦理论从来都是并且现在仍是以一种近似方案进行讨论的。我们还从来没有精确解出过任何的弦理论。在这一方案中,人们做一级比一级更好的近似,并且在每个阶段都要关注上级近似是否给出有限的结果,因为在这种方案中任何一级都可能出现麻烦。
    一开始,我们考察最低级的近似,这是最简单的情况,结果是有限的。仅此一步本身就马上引起轰动,因为即使在这一级近似上以往的有关引力的量子理论也从未给出有意义的结果。然而弦理论的机制使得人们有理由相信,一旦得到最低级近似并且发现是自洽的话,从这种意义上讲在近似方案的所有级上都将是自洽的。所以尽管确实存在突出的问题并且许多人正在探究为什么理论会到所有级都自洽,但我想人们还是普遍相信它们会自洽的,至少对于那些在最低级自洽的理论如此。然而不管怎么说,这是一个非常有趣的探索。通过尝试在近似方案的任一级上解决这个问题,可以发现理论所具有的那些超越任一特定近似步骤的性质。这实际上是目前阶段弦理论研究的主题。
    
    让我们回到历史的叙述上来。1982 年您突然发现能够在引力的计算中得出有意义的结果,那时您已经投身于这一了不起的事业,相信正在描述的是差不多类似强作用物理的理论。
    
    但当时情况并非如此。当刚刚完成理论的超对称化,换句话说就是在得到超弦理论的结构之后,立即变得明显的一件事就是它在某种程度上同超引力理论有一种紧密的关系。
    
    很显然,您已经预料到会从这些理论中得到关于引力的某种描述。
    
    我们确知超弦理论以某种形式包含了超引力在内。超引力事实上是包含在对超弦理论的近似当中的。它是这样一种近似,就其本身而言是不自洽的,但当超引力作为超弦理论的一个部分之时就变得有意义了。
    
    所以这超出了超引力这个当时非常流行的观念而发展了某些新的概念
    
    是的。弦理论与以往的任何理论都完全不同,原因就在于所有以往的理论― 从麦克斯韦的电动力学到广义相对论和超引力― 都包含粒子,诸如电子、引力子、夸克和其他的粒子,它们是点状的从而是没有内部结构的。弦理论则不同,它的组成单元是延展了的客体― 弦。尽管这一点听起来可能是一个不大的差别,但实际上却给理论的结构带来巨大的差异
    
    容易看出它是如何带来差异的吗?
    
    在量子力学中处理点状物体如果不是不可能的话也是非常繁杂困难的。描述量子力学的一种途径是通过所谓的测不准原理,利用这一原理可以容易地论证,所处理的尺度越小相应的能量就具有越大的不确定性。而在一个引力理论中这就意味着,当试图描述尺度小的出奇的物体时(这里的小尺度是就质子的大小相比较而言的),所考察的能量所具有的涨落可能会大到足以构成一个小黑洞。所以,如果我们考虑在足够小的尺度上(这一尺度称为普朗克距离,它等于10 一33 厘米)进行观测的话,我们就不得不把哪怕是真空想象为在很短时间内游来游去的涨落着的黑洞构成的海洋。这当然彻底改变了我们关于空间的概念并且成为一种灾难,因为我们不再能真正了解正在发生些什么。空间本身由无穷多点构成的这种观念可能不再具有任何意义。
    
    但是,难道就不会有在这样的时空背景下运动的点状物体吗?
    
    弦小的不可思议。平均来讲它的长度是普朗克长度的量级,也就是说比一个质子的尺寸小1 以0 倍。所以很多情况下它是弦状这一事实并非举足轻重。如果不去仔细辨别是不会注意到它是一根延展的弦。
    
    您是说除非在非常短的距离和很高的能量下,它的行为同点粒子一
    
    样?
    
    是的。并且无论如何这样短的距离是我们永远也没有希望在实验室里直接测量的。然而,量子引力理论的所有问题都是出现在这一尺度并且也正是在这一尺度上弦理论开始明显不同于爱因斯坦的理论,或确切地说是不同于以往任何其他理论。
    
    设想如果一根微小的弦可能是首尾连在一起的一个圈,粗看上去它就像是一个点粒子,但是如果注意细节的话就会看到各种蠕动并且正是
    
    这样的运动对高能行为有所修正,这样的一种图像是错误的吗?
    
    从某种意义上讲可以使用这种图像,并且当今我们大多数人确实是用这种图像来理解理论的。但实际上理论很可能要深刻得多,因为正是在观察到蠕动的尺度上,姑且这么说吧,时空的整个结构不得不被修正。所以,甚至连设想物体以我们通常在连续时空中考虑的方式来运动可能都是不正确的。
    
    弦实际上是在一个移动的时空背景中蠕动着的?
    
    在一个引力理论中其实是不能把时空结构同那些和引力相关的粒子分离开来的,并且既然我们已经扩展了引力的含义,以致爱因斯坦的引力论只是这一理论的一小部分,那么我们也不得不大大扩展空间和时间的概念。
    
    您是说在某种意义上,是空间和时间建立在弦之上而不是反过来?
    
    是的。弦的概念是同它在其中运动的时间和空间分不开的,因而如果对产生引力的粒子的概念作重大修改从而使之成为弦状的话,就应当同时在某种层次上抛弃时间和空间结构的传统观念。我所说的某种层次是指在与普朗克长度相关联的尺度上。
    
    让我想想是否理解对了。在绝大多数情况下我们可以忘掉在小尺度上空间和时间的移动与星云状结构并且想象粒子以通常的方式存在于普通的时空背景中。但是如果我们考察更精细的结构就会发现弦,并且空间、时间和弦以某种微妙的方式交织在一起了。
    
    一点不错。是某种微妙的方式,关于它迄今还没有真正得到正确的认识。目前许多研究工作正关注如何准确了解它的机制。
    
    1984 年那些有趣的弦理论只在时空维数等于十时才有意义。我们应当如何想象这个十维时空背景与我们感知到的时空(当然只有四维)之间的关系呢?
    
    很显然,这些额外自由度一定有某些不同寻常的特点,因为否则的话我们应该感觉到它们的存在。人们应该习惯于这样一种观念,即任何包含引力的理论本身就定义了空间的结构。人们还应该习惯于空间可以弯曲,于是自由度可以卷缩并在某种意义上变得很小。
    
    这一观念难以把握,不过还是可以非常粗略地打一个简单的比方,其中自由度似乎比实际应有的少了一维。比如,想象有一个软水管,它是一个二维曲面,是一个有环向自由度的长长的玩意儿。如果不在很近处观察它会认为它是一个只有一维自由度的物体,事实上就是一根线。但是如果靠近看就会意识到,原来还有另一个很小的自由度― 它是一个细细的管子。
    把这个例子推广,就可以有好几个额外的自由度,它们卷缩得很紧以致于除非有极高的分辨率,否则是注意不到它们的。
    
    这意味着在空间的每一点,或者说是我们认为的一点,实际上是一个更高维自由度卷缩起来的东西。
    
    是的。
    
    看上去有点奇怪,在一个由十维自由度出发的理论中我们最后有四
    
    维而另外六维却卷缩起来,为什么是六维呢?
    
    可以肯定,目前我们还没有真正懂得这一点。我想在于对这些理论的数学方面我们还刚刚开始触到整个领域的一个小角而已。通过进行一些相当平常的计算而得到极不寻常的结果,我们从几乎是一系列偶然的事件中认识到理论的不同寻常。但是关于理论的完整结构尚不了解,并且这类问题在我看来,要给出恰当的回答只有等到理论能够以一种数学上更彻底的方式重新表述的时候。例如,近来的发展已经淘汰了几种超弦理论,这些理论直接在四维时空中起作用― 额外自由度在某种意义上自动卷缩掉了。
    
    假定有一天您终于认识到为什么是六维自由度卷缩起来,您是否认为也将能够了解它的卷缩方式?一定存在许多方式,许多不同的拓扑去把六维自由度卷缩起来。
    
    我个人相信是会的,但事实上关于它是否属于那种我们最终能够弄懂的事情还是一个有争议的问题。在逻辑上可以想象有很多可能的方式,并且完全出于偶然我们碰巧生活在这样一个宇宙中,这里额外自由度以这样特殊的方式卷缩起来。
    
    会不会出现这样的情况,如果它们以某种方式卷缩的话,宇宙的环境将使生命无法存活?
    
    这是一个可能的论点,但不是那种我认为有吸引力的论点。
    
    正如您刚才讲到的,空间和时间的结构在非常小的尺度上会变成蜂窝状或像泡沫一样,或者无论您偏爱哪种描述方式,并且您大概希望从一个传统的时空背景出发来表述弦理论,这样是否会产生某种问题?
    
    是的。很明显这不是一条完全正确的途径。它至多也只能是对真实世界的一种近似描写,但在目前却是所能做的最好的一种。然而,甚至在这一水平上,我们已经看到理论只在很有限的一类对称下才有意义,而这已经是非常令人感兴趣的了。的确,这类对称性是特别令人感兴趣的,因为起码在理论上可能的这类对称性中的一种惊人地相似于早期的纯粹基于实验提出的描述粒子的对称性。
    我认为这样说是真实的,即目前超弦理论所引起的很大轰动是由于这些可能的理论中的一种,称为〔 SX 〔 8 理论,包含所谓的例外群。这是非常特别的数学对称性,在数学上有很特殊的作用,并且据猜想出于某种原因在物理学中也将有非常特别的作用。现在我们终于有了一个理论上的根据来说明为什么超弦理论会从物理学中萌生出来并且我想这也正是它令人激动的原因。
    
    您是说自然界挑选了数学上相当特别的一种群― 例外群,而且正以某种方式发挥作用?
    
    是的,并且我认为这对理论物理学家是特别有吸引力的。目前这些理论与物理发生联系的方式使得计算具有某种困难,因为容易的计算是关于那些极小距离或等价地极高能量尺度下可以测量的量,而我们在实验室中无法做到这一点。所以,必须做的是了解如何从很短距离的物理进一步外推以及它对于这种尺度下地球上的加速器实验室所能测量的量的预言。这种工作是十分困难的。
    尽管如此,目前所作的一切还是很吸引人的,因为对于所能出现的情况存在各种相当严格的理论限制。比如,尽管我们不能证明那些额外自由度确实卷缩起来并且小得无法察觉,但如果我们假定理论的方程最终保证这些额外自由度卷缩的话,那么所有的其他结论都立刻可以得到。如果我们假定那些额外自由度― 它们是我们不希望要的― 事实上非常微小,那么就存在一条清晰的途径,通过它那些可以由实验室中高能物理方面的工作所察觉到的对称性就能由理论来预言,这一点是非常诱人的。
    您提到过更新近的进展已经呈现出不仅仅在十维而且在其他维数时
    
    空中发展弦理论的可能性。
    
    1 984 年我们对于在时空维数等于十的近似下理论所应具有的形式差不多是唯一地确定下来了。在这种情况下只有两种可能的理论:一种具有50 ( 32 )而另一种具有〔 SX 〔 8 的对称性。当然,十维自由度并不是我们所生活的时空自由度,并且人们很快认识到如果六维额外自由度卷缩起来并且很微小的话,则这些异常理论似乎可能给出四维时空的物理学。即使在这个阶段很显然还是存在许多不同的方式。开始的理论或多或少是唯一确定的,但是后来在四维时空中的理论却有许多不同的可能解。我们不知道如何挑出正确的解,尽管我们有一个差不多是唯一确定的初始理论。
    现在人们已经找到构造直接在四维自由度中成立的新解的途径。换句话说,这些解从不需要经过十维的阶段。这些只是与我早先所讨论的形式不同而已,所以称之为不同的理论是错误的。它们可被看作是十维情况下同一理论的不同的解。所以我们面临的局面是,为数相当少的理论却有非常多种解。
    这儿可以打个比方。设想展示在你面前的有冰、水和蒸汽。你恐怕要花点儿时间才能认识到它们实际上是同一物质的不同的相,并且支配这些相的微观性质的物理也是相同的。这只不过是物质存在的条件不同而已,是你观察物质的条件使得水的三个相互不相同。
    超弦理论的情况也大致如此。和理论的这些不同的解所对应存在大量不同的相,并且我们必须分辨它们内在的结构。这事实上是近来多数工作的主要目标:试图找到超弦理论的一个更为基本的框架,于是我们得到一组方程,它们的近似解就是我们目前的这许多不同“理论”。希望那时我们能够断定哪个解是描写我们观察到的物理,如果这样的解存在的话。
    
    有件事使我对于直接在四维中表述理论感到有些困惑。我觉得反常相消只有当理论是在十维中写出时才可行。
    
    我已经说过了,所有这些不同的“理论”实际上是同一个基本理论的不同方面。反常相消对于它们都是适用的。人们用越来越精确的近似来表述理论,在这个图像中人们认为粒子是在或多或少同我们已经了解了的经典时空类似的时间和空间中运动的弦状物。但弦理论要更为深刻。我已经解释过了,实际上应该和我们改变粒子的概念一样地去改变时间和空间的概念。如今弦理论的这个方面,即弦在其中运动的时空本身却为弦所改变,这个真正深刻的方面还没有在弦理论的表述中有所反映。我们真正需要的是关于弦理论表述的一个体现了这一原则的基本的
    
    新观念。其次,当然应该去证明我们所用的近似来自于这一更为基本的图像。但同时或许在这一点上就能够懂得理论诸多不同形式或者不同解之间的差别。然后也许我们会更有机会对实验上观测到的物理作出预
    
    在这些四维理论中,能否认为十维理论中的另外六维自由度仍然存在,只是形式不同而已?
    
    情况要深刻得多。实际的情况是,在任何弦理论中并没有四维或者十维自由度,那只是一种近似而已。在理论更为深入的表述中,时空的一个维数这样的概念都要全部改变。在我们的正常概念里时空是一个平滑的点集。时间或空间的任何位置都是由一个点标定的。应该在一个更大的空间― 有些类似于由一根弦的所有可能位置构成的空间里表述弦理论。事实上,这是一个大得无穷多的空间,所以当我们说一个理论是在十维或四维自由度中时,实际是说这个大得无穷多的结构的某种近似。在这个大得无穷多的结构里,一种表述是在四维中抑或在十维中写出,差别是很小的。我们之所以使用十维或四维这种语言是因为目前我们还不得不以一种近似的方法来讨论弦理论,并且只有在这种近似下,一个小的有限数目的时空自由度的全部概念才有意义。
    
    更高维自由度卷缩起来的想法以及对于它们卷缩方式的兴趣现在是否已经消失?
    
    绝没有。可以肯定,它的某些方面不如其他方面那么受到强调,但决没有消失。事实上,从某种意义上讲它已经归入这一更大的结构。所谓十维或四维仅仅是这一更大的弦空间的一种近似,这一空间其实有无穷多的自由度。
    
    所以还必须为这个弦空间的更高维自由度如何卷缩找到某种途径?
    
    假定我们有无穷多的自由度,用弦的语言来讲,是否有六维额外自由度就无关紧要了。自由度卷缩的框架现在已扩展为一个更大的框架,这里关心的是弄懂弦状时空的含义以及在何种程度上物理学表现为这一更丰富结构的某种近似。
    
    您是否认为我们将能够设想出弦状空间的含义?
    一旦人们真正地理解了理论的正确的基本表述,这也许就是出奇简
    
    单的事情了。物理学中往往如此。当最先发现一个有趣的新结构时,事情总好像非常复杂,但当它被真正理解之后事情就变得清楚而简单了。当然我不知道这一新的表述会是什么样子,但肯定希望它是简单的。能否以一种非常具体的常识性语言来设想它,还是只在那些对深奥复杂的数学有良好素养的人看来才简单,这个问题我无法回答。
    
    现在我们来讨论超弦理论的科学地位这一问题。费曼对超弦理论是持完全否定态度的,因为他认为这一理论没能同诸如基本粒子质量和藕合常数这样一些实验数据取得联系。对此您有何评论?
    
    我并不认为超弦理论这种办法会是费曼所喜欢的研究物理学的途径。说目前的理论与对基本粒子的特定测量作出非常具体的预言尚有差距我认为是公正的。人们正在非常努力地工作以期能够理解这些预言的含义,并且我不怀疑最终会有更多的了解。
    我前面说过了,到目前为止理解理论的途径是逐级近似,但是存在一定的问题,解决它们只有等到我们突破这一近似框架之后才有可能。我们观测到的粒子的质量问题就是一个这样的例子。在目前的近似水平上所有粒子的质量都是零:在这一级近似中粒子是无质量的。事实上这是一个好的近似。要知道我们测量这些质量是在所谓的普朗克尺度下进行的,这个尺度是质子质量的101 ”倍,所以我们在实验室里能观测到的粒子质量在这一尺度下是非常小的。因此,认为它们的质量为零是一个很好的近似。
    当然,我们实际上观测到的粒子并非零质量粒子,它们总是有一定的重量的,我们应该能够预言它们的质量。这类预言― 这些粒子具有质量的事实以及它们的质量的具体数值这一切在弦理论的目前表述中都是很难给出的。
    另外一个非常有趣的问题也得等到我们对理论有了更好的认识之后才能解答。例如,如何在这一理论中描写黑洞。这是一个包含广义相对论的理论,所以它一定含有黑洞。如何用弦来描述它们呢?其他一些问题涉及早期宇宙学。在宇宙的早期曾经有过一段极热的时期,弦理论对此十分重要。为了弄懂弦理论对早期宇宙的演化应有什么样的解释我们不得不再次突破目前讨论弦理论的近似框架。所以对于很多有趣的问题,由于对理论本身尚无足够的了解而无法提供答案。
    我个人的观点是,理论尚在初期,它的成功与否无论如何不应该以能否对我们已经测量的细节进行预言作为判据。肯定地,如果这样一个全新的理论是正确的话,那么它代表了物理理论的一种转变,这种转变足够巨大以致于对我们尚未真正想过要测量的量会有某种暗示。应该有某些让人大吃一惊的新东西。
    
    您相信会有吗?
    
    我的确以为我们还没有了解它,我们还不能推演出理论的所有预言,但是已经有了一定的想法。我必须承认,作为理论可以测量的和确定的预言它们确实还不怎么吸引人,但无论如何就它们的性质而言还是引人注目的。例如,这些理论的预言之一是说宇宙中存在一种全新的物质。这种被称为影子物质的东西除了它的引力效应外我们是无法直接看到它的,尽管影子物质的粒子之间可以有强的相互作用。
    
    这种影子物质可能存在于我们的周围,对吗?
    
    是的。我不是说我相信了理论对它的预言,但这的确是理论所能给出的一种预言。
    
    您是否在暗示在某种意义上宇宙存在两份拷贝,一个是我们生活于其中的,而另一个影子宇宙我们没有办法看到,我们只能通过引力效应来感觉它的存在?
    
    理论可能预言它的存在,我这样说必须要慎重。然而,这种物质是否真的存在于我们的周围依赖于宇宙历史的细节而这在任何情况下都是非常难以计算的。
    
    如果一个影子恒星或行星穿过我们的太阳系的话,我们差不多应该注意到
    
    是的。
    
    如果通过寻找引力效应来检验一个理论将会是很不可靠的。
    
    是的。即使这种物质存在,它也不能作为一个很容易验证的例子。
    
    关于超弦理论值得实验检验的预言您能给我一个其他例子吗?
    
    当然我们还不知道有什么确实的预言。然而,一个可能的预言一定与弦理论存在有奇怪的拓扑性质的额外自由度这一点有关。可能存在这样的自由度,像炸面包圈一样中间有洞。于是像弦一样的东西可以缠绕在这样的一个洞的周围,这些陷入圈套的弦将有一些奇怪的性质。例如,
    
    它们会产生我们当作新粒子的东西,这些新粒子将非常重,带有奇怪的电荷― 分数电荷。这些粒子如此之重以致于无法在实验室里产生但却可能产生于大爆炸,那时候宇宙是非常非常热的。
    我应该强调,对于弦理论可能带来的完全不同于常规粒子物理的那些效应的建议是相当遥远的事情。所以,尽管这类预言目前尚不能过分当真,但它的确表明弦理论存在着与传统理论的不同。现在还处于早期并且我们希望找到其他同样独特的效应。
    
    希尔顿· 格拉肖对弦理论也持批评态度。他说这些理论能损害人们进行实验的主动性,因为它们给人们的印象是理论学家已经包揽了一切。您对此的反应是什么?
    
    我赞同这样的看法,即目前这些理论距能够直接解释加速器上所测量的量还十分遥远。承认它们是完全不同于以往的理论,那么它们应该预言一些我们还没有想象过要进行测量的全新现象。只有当爱因斯坦写出了广义相对论之后,他才懂得哪种现象可以测量,可以反过来检验理论。水星近日点是早就知道的,但只有在爱因斯坦得到了广义相对论之后才认识到这一实验测量中的特殊反常是十分重要的。所以,在超弦理论中我们所需要的正是像水星这样的例子。一些独特的实验证据有可能已经得到了,只是还没有引起重视,因为没有人意识到它与检验一个基本理论有关。
    
    就我的理解,格拉肖认为弦理论处理物理的方法是根本错误的,它们采用的是称为自上而下的方法,即从一个一般的表述出发然后试图由此得到一个对真实世界的描述。格拉肖喜欢从实验物理的发现开始并且设法逐渐地建立一个理论,也许是朝着一个普遍的理论迈进,但一定是从实验物理出发。您是否认为在此您应该考虑一下一分为二的问题?
    
    我认为两者是不冲突的。历史上理论物理的进步曾受益于这两种方法,并且这两种方法你都能找出先例。的确我承认超弦理论研究工作背后的动力曾经是并且现在仍是其优美的理论结构以及要解决本世纪物理学的基本理论佯谬这一宏伟的愿望,这个佯谬就是量子力学与广义相对论之间的矛盾。可以肯定这是我本人的动机,也是其他人的动机。我想,有另一批人采用所谓自下而上的方法也是重要的,这两批人能够相安无事并且大概还可以相得益彰。
    
    回顾弦理论的早期,那时尚无太多人对它感兴趣,是否有这样的时候,即您觉得因为一直置身于物理学的这个领域别人好像有意在回避自
    
    己呢?
    
    没有。我不认为我们是被人回避。我想在很大程度上我们是被忽略了,这一部分是因为弦理论无论从概念上还是在技术上同当时流行的理论都大不相同。在1 980 年代初期,对于那些没有从事弦理论研究的人们来说需要太多的力气去学习它的技巧以便决定是否可以相信这个理论。除了少数例外,人们是不愿花力气的。从某种意义上讲那些日子的生活是非常愉快的,因为粒子物理一般说来是一个竞争非常激烈的学科,所以能够按部就班没有压力地从事研究是令人愉快的。
    当然在1970 年代的初期有过一段弦理论的时代,到1970 年代中期就已经开始衰退,所以从个人声誉的观点来看当时是决不应该从事弦理论研究的。可能在美国比英国更是如此,但可以肯定,粒子物理的主流― 那些头面人物所从事的领域― 决不是弦理论并且如果是从事弦理论研究的话在当时是难以找到工作的。我想当时没有其他人研究这一课题的事实本身就证明了这一点。
    
    您实际上是如何开始同约翰· 施瓦茨合作的呢?
    
    我们原本就互相认识但从没有合作过,直到1 979 年夏天我们碰巧都正在欧洲核子研究组织访问。欧洲核子研究组织是一个理想的地方,人们汇聚在一起交换看法,我们就讨论超对称和弦,这些是我们共同感兴趣的,后来就开始合作了。
    
    展望未来,爱德华· 魏廷曾经说过,超弦理论是21 世纪的理论,只是偶然地降生于20 世纪,并且他认为这一理论在未来的50 年内将要在物理学中占主导地位。您也是这么看吗?
    
    我完全相信超弦理论的发展将在很长一段时期内成为理论粒子物理的主流。但我乐意换个不同的说法。就是说我现在无法设想例如那些先前从事广义相对论的人和那些现在从事弦理论的人会重新回到广义相对论的研究中去而不引入弦。那似乎是不可设想的。
    
    弦理论是否终将成为一个包罗万象的理论呢?
    
    我只能说由于我们对这一理论的深刻结构还知之甚少,所以我反对所谓包罗万象的理论这类老生常谈的说法。我们还不知道理论预言些什么并且我们也还不清楚要问些什么问题。我的感觉是,随着对理论更深入的了解所有的争论和问题都将被提出来,而在这一点上理论或许给不
    
    出任何解答。所以我觉得所谓它是一个包罗万象的理论只不过是说看来它们似乎能够对目前我们认为粒子物理中重要的问题予以解答而已。
    
    弦理论至少是自称要解决所有的粒子与所有的相互作用如何联系在一起这样一个问题的,是吗?
    
    是的。它处理这个问题并且事实上关于答案会是什么样的它显然已给出一些非常有趣的提示。
    
    所以它的确把相互作用、构成世界的物质以及将其容纳在内的空间统统合并在一起,这在我听起来已经是所有的一协了!
    
    但我们还不知道如何表述理论才能将时空与弦状粒子统一起来。我们不知道理论在超出普朗克尺度的情况下会是个什么样子,而这一点在我们目前的想法中却是占十分重要地位的。
    
    所以可能还有更深的层次?
    
    可能存在全新的问题而对此理论却无法回答,并且我认为只有当我们能够以一种更合逻辑的方式理解理论以后才能知道问题是什么。例如,看来似乎这会使得我们关于量子理论的观念产生重大改变,那将是非常激动人心的。
    
    改换一下思路,不是深入下去而是考察一个更大的尺度其中可以有更为复杂的系统,那又会怎么样呢?人们同样可以反对称之为包罗万象的理论如果它不回答诸如生命起源一类的问题。
    
    是的。存在各种复杂的问题,对此在微观层次上物理学的知识是没有多大帮助的。
    
    但您是否同意超弦理论一旦成功的话将标志着2500 年来对客观物质世界最终组成单元探索的顶峰、是简化论者的一大胜利?
    
    我个人就不认为存在所谓的“最终组成单元”。我无法相信在20 亿年后还没有人得到一个更好的理论。我肯定这暂时是一个好的理论并且它将会持续很多年。弦理论同如此之多的数学分支有联系这一事实本身就表明它包含着深刻的道理。
    
    所以弦会保留下来?
    
    在一个相当长的时期里会的。
    
    5 .戴维· 格劳斯(Dav 1 dGross )
    
    戴维· 格劳斯是普林斯顿大学的尤金· 希金斯(E 日gon 。日1 99 1 ns )物理学教授,基本粒子界非常著名的理论家,对量子色动力学作出过重要的贡献。作为“普林斯顿超弦四重奏”中的一员,他是异常弦模型的创始人之一。
    超弦理论的奇异特征之一是必须建立在超过四维的时空之上,这意味着存在某些我们并未见到的空间自由度,您对此作何解释?
    
    存在三维以上空间的想法由来已久,并非弦理论所特有,虽然弦理论是一定要建立在高维空间的。最开始这一点被认为非常糟糕,但现在我们了解到,存在多少维数的空间只是一个实验问题。如果多余的维数卷成小环(或更复杂的表面),并且非常小,我们通常是觉察不到的。
    
    您的意思是,我们通常认为的三维空间中的一点,实际上是高维空间中的某种小块,是这样吗?
    
    对。一根麦杆远看像一根线,但当你凑近时就可以看到它还有一个环状的维度。在某种意义上,任何一点都可能有更多的维数伸向那些未知的方向。弦理论中我们需要六个这样的维数,如果理论与至今为止我们并未发现的事实相符,那么它们一定是卷曲的而且很小。超弦理论有一个非常非常小的自然尺度(10 一23 厘米),所以认为这些小块很小是有道理的。这样,展现在我们面前的只是三维空间,它们是大而开放的。
    
    假如我们拥有观察这些细微结构的仪器,并看到这些多余的维数,
    
    它们会是什么样子呢?
    
    我们怎样才能真正看到呢?我们的观察手段是建造巨大的加速器,以探索极短距离的物理。
    
    这只是一个设想吧?
    
    是的,一个设想的加速器,能量比目前的加速器大101 “倍,得花费比我们出得起的钱多1 以0 倍的资金,这样才有可能探索那些多余的维数。不管怎么样,我们是不能在显微镜下看到它们的。假如能这么做的话,它们看起来将像向左,右或上延展开来,但是在这些方向必须转圈并回到起点,所以它们是环状并且封闭的。
    
    是否有可能计算出这额外的六维空间的形状?
    
    从爱因斯坦开始,时空几何的问题一直很热门,它应该是由物理决定的。原则上,我们可采用弦理论并解出弦方程,然后把弦方程的解作为时空结构的理论,来决定时空拥有什么样的几何。
    实际上,目前从异常弦理论出发所做的只是探索可能的经典解(非量子解),通常是间接地推导理论中运动方程的解。关于异常弦理论很重要的一点,是我们已经找到了整个一族解,实际上是百万个可能的解。其中一些几何地描述了我们的世界,有三维空间、一维时间和六个卷缩的小的自由度,它们卷成极奇特的数学流形或曲面,具有数学家所乐道而物理学家不得不去了解的一些性质。所以异常弦理论有与现实世界相一致的解。当然还存在与现实世界不相符的解,它们具有更多的开放空间维数,我们尚不清楚是什么物理原理从十维、八维或六维等解集中选择了四维时空解。
    
    有许多解是三维空间的吗?
    
    对,有成百万的解具有三维空间,可能的经典解是极其丰富的,它们甚至可以进行量子力学的处理。当我们对它们进行量子修正时,到微扰论的任一级,并没有发生通常极有可能出现的无意义和不稳定(这里假定经典解基本正确,只须作略微的量子修正)。
    最初,解的丰富程度是令人高兴的,它提供证据说明异常弦理论能非常好地描述真实世界。除了具有四维时空自由度外,这些解还有其他类似真实世界的性质― 正确数目的粒子种类,如夸克和轻子,以及正确的相互作用种数。这些是― 或至少说可以― 从理论中自然地导出。在两年前这让人们颇为激动了一阵。
    可是,让人稍觉不安的一点是:我们有如此多的解,却没有好的选择方法。更令人头痛的是除了一些期望的性质外,这些解似乎潜在着灾难性的性质。其中包括理论的对称性不存在于真实世界中,因而必须以某种方式破缺掉;有些零质量粒子并没有被观察到,实际上已被实验排除。所以现在我们所有的解都存在一些问题。人们愿意相信某些在微扰论中没有展现出来的动力学效应可以解决这些问题,并从那些同样好的解当中挑出一个唯一的解来。
    
    是不是这样,现在的理论基于一种近似的方法,即微扰论,通过考虑一系列小的修正来作计算。但这些近似解除了有些令人不十分满意之处,不仅数量太多,而且都有一些缺陷。您认为如果能发展一套求精确解的数学方法就能解决问题吗?
    
    是这样。在许多其他的理论中也有这样的情况,比如量子色动力学,它是关于夸克和胶子的理论,描述核力及原子核的结构。强子(核子)的性质只能通过极其复杂的非微扰机制来得到。微扰论得到的结果是没有意义的,如同我们在弦理论中遇到的一样。
    现在我们在弦理论中还只会使用微扰论。我们对它没有充分的理解,甚至没有足够的公式来让我们处理非微扰问题,但有充足的理由说明微扰论是不够的。
    
    有什么理由呢?
    
    首先,如果弦理论是正确的,那么微扰论必然不充分,因为其结果与实验不符!
    第二,弦理论容纳了许多我们熟知的理论,如量子电动力学,而微扰论对它来说是不充分的。
    第三,该理论没有任意参数,没有可调整的常数。如果你找到一个解,则它是无法改变的,任何东西都可通过计算得出。很难想象在这样的理论中做微扰― 用什么量来展开呢?通常是用一个可调整的常数作为小量来展开,但这里不存在这样的常数,理论中每一个常数都是可以计算出来的。
    第四,如果你提出一个包含所有物理的理论,它必须处理物理学中一个非常基本的问题,即宇宙常数的问题。
    
    请说说这个问题。
    
    这是个宇宙背景能量的问题。在通常的物质理论中总是忽略引力,能量的绝对尺度是不重要的,人们并不关心。人们只关心相对能量,而没有测量绝对能量的方法。可是,引力是与能量相藕合的力。通常我们说引力与质量藕合,但爱因斯坦告诉我们质量就是能量。因而在某种意义上,引力“知道”一个给定物体的能量。这可推广到整个宇宙,宇宙有它自己的能量密度。
    
    甚至在真空中吗?
    
    对。你可以测量真空的能量密度,因为能量密度越高,宇宙因引力造成的弯曲就越大。所以通过观察宇宙的球状结构,可以测量宇宙的背景能量密度。这已经被大致测量过了,得到的只是一个上限,因为结果似乎非常接近于零。事实上,在我们所遇到的等于零的量当中,这似乎
    
    是在实验上最准确的!以普朗克质量(引力的自然质量或能量标度)为单位,它在10120 分之一的精确度上等于零。这意味着,如果你坐下来研究包括引力在内的任何现代物理理论,在缺乏观测的情况下估计宇宙的背景能量密度,那么你的估计将比实测上限大10120 倍。观测结果是如此之小,以致于人们确信它就等于零!但我觉得这似乎没什么道理,它应更大些才合理。不仅如此,即使你人为地让能量密度为零(物理学家并不喜欢在小数点后120 位上调整一个数),你将发现丢掉了一些量子效应,而据我们所知,量子效应是足以产生相当大的宇宙常数的。自爱因斯坦引入宇宙常数以来,它的数值之小一直是个谜,人们发现必须让它为零却说不出为什么。
    只要你不声称拥有一个包罗万象的理论,就不用为此伤神。但如果你试图建立这样一个理论,就必须解决这个问题,因为要么产生一个宇宙常数,要么不。如果是不,并且仍能再现我们所见的现实世界,那么一定需要某些目前未知的物理机制,那显然不会是微扰论可以处理的。到目前为止,弦理论中宇宙常数保持为零。异常弦理论的某些解提供了四维可观测的维数,而这四维的维数看起来像现实世界;这等于说没有宇宙常数。如果有的话,将不存在我们所见到的四维时空,那三维空间将卷成比原子更小的小球。当然这种事情并未发生。究其原因是与超对称相联系的,超弦是超对称的,正是这种对称性阻止了宇宙常数的产生。而任何对称性破缺的想法将导致宇宙常数的产生。我们找不到办法使对称性破缺(它必须破缺,因为没有关于它存在的证据)而又不产生宇宙常数,人们所尝试的任何一种破缺对称性的方法都产生宇宙常数。
    所以在物理世界中有一些很奇怪的事情。一些新的原理或对称性破缺的新途径似乎会在某种程度上解决困难,如果用弦理论来做,则必须通过一种与微扰论图像很不一样的动力学机制。
    
    您是否认为这个宇宙学的问题的解会成为弦理论的一个基本组成部
    
    分?
    
    那是很可能的。但没有多少证据,因为只有唯一的解能给出结论,而就我们所知,理论中宇宙常数为零,同时超对称是不破缺的。这两件是相互联系的,一好一坏。现在我们希望设法使对称性破缺而不产生宇宙常数。没有证据表明会这样,只是这么希望,希望理论能描述客观世界。但若真能做到,我们将会发现一些非常迷人的动力学过程或机制,是现在的微扰论方法无法描述的。
    如果说微扰论更为直接,并且从数学的角度来说比精确解更容易的
    
    话,那么进一步发展的途径将会是什么呢?您是否认为必须学会新的数学方法?
    
    当然,这是多数人选择的方向。人们想从理论中发现些什么,多半是出于物理的动因。就我们所知,理论的微扰解具有解释低能实验现象的大多数要素,我们缺少的是对某些基本问题的极其困难的解答。所以我们该何去何从呢?在其他学科以及十年前的粒子物理,我们总是等待做实验的朋友们提供些线索。过去我们一直靠这种方法取得进展,但现在最好不要再抱这样的奢望。
    
    就因为没有足够的钱来建造一个足够大的加速器吗?
    
    即使所有国家的钱加在一起也不够,那是个天文数字。不仅如此,目前也还不具备建造这么大的加速器的技术。我们非常希望能够修建比目前所拥有的大十倍的加速器来探索下一个能区中令人感兴趣的物理,但要达到普朗克质量的能量尺度在可以预见的将来是不可能的。所以这些能区也许不会提供直接的线索,而我们将不得不从宇宙学或者低能物理中寻找间接的线索,并将更为依赖数学上的启发以探求理论的分支并寻找新的数学结构。对于理论物理来说,这是个冒险但却充满机遇的途径,但若没有别的办法,我们也只有沿着这条路走下去。
    
    当然,怀疑论者会说,历史上曾经有过这样的情况,物理学家们声称已经找到一个统一的理论可以包容世间万物,但最终总是证明错了。您是否觉得正在冒险继续这种神话呢?
    
    即使不脱离实验,也还总是存在这样的危险。这样的危险总是存在的,所以必须认真持久地检查自己的想法,才不致于闭着眼睛在死胡同里浪费时间。许多理论看起来只差那么一点点,但很可能就是错的。这些理论的结构无疑是非常丰富的,并且在很大程度上包含了我们已有的知识,或者至少看来能够容纳低能物理,而以前那些所谓的统一理论实际上做不到这一点。但这可能是虚幻的,可能会冒出比超弦和十维时空更奇怪的东西。如果不试一试,就无法辨别对错,而这种尝试性的工作正在进行并且会持续多年直到找到证据表明它根本就是错误的,或者提出更好的想法。实际上,好主意比相互矛盾的实验结果更重要,因为物理学家必须有事可做!在这种情况下,如果没有更吸引人的想法,他们将继续从事超弦研究。
    弦理论似乎对理论物理学家有特别的吸引力。在我的经历中还从未
    
    见过对一个理论有如此的狂热。是不是有某种本质上令人满意的东西保证了超弦可以作为一个包罗万象的理论的结构基础呢?
    
    这两年来该领域如此热门有两个理由。最重要的一点,是没有什么其他的好主意。多数人是因为这个理由来做超弦的。当人们开始对弦理论感兴趣时,他们对其一窍不通。事实上多数人的最初反应是觉得这个理论荒谬且令人不快,至少在数年前当人们对弦理论的了解尚十分粗浅时是如此。人们去了解它并且对它发生兴趣是不容易的。所以与其说人们被它所吸引,不如说是无路可走。相比之下,构造大统一理论的所有其他途径从一开始就保守得多,只是逐步变得越来越激进,但都以失败告终,而只有超弦理论还未遭厄运。另外,一开始就发现弦理论很有潜力,可以给出比其他途径多得多的东西。
    第二个理由是,当你研究它时,当理论得以发展时,越来越多的人为它的美妙所折服。弦理论确实是一个非常美妙的理论,虽然我们对它的理解还很初步。可以期望,随着理解的加深,我们有可能发觉它愈加美妙。总的说来,目前没有其他好主意,而人们越来越为弦理论的深刻结构所吸引。
    
    我们是在普林斯顿,这里曾是爱因斯坦的故居。如果他活到今天,
    
    您认为他会对超弦理论做些什么呢?
    
    人们总是想知道爱因斯坦对各种事物的看法会怎样。我经常这样问自己,爱因斯坦对此怎么想,爱因斯坦对那又怎么想?当然,你得设法让爱因斯坦容忍超弦是一个量子理论,还得向他解释超对称,这是对他的时空观的大胆扩充。我想他不会不喜欢超对称的,它不需要量子力学,实际上从很多方面来讲,这种时空对称性的扩充部分地实现了爱因斯坦的目标。爱因斯坦有两个目标。一个可能是错误的,即量子力学可以从高度约束的经典理论中得出,量子条件由方程的约束提供。现在没人相信这一点。人们相信量子力学是正确的和根本的。
    但爱因斯坦也相信几何将决定动力学。他常就他那著名的场方程作这样的评注,即广义相对论方程的左边是时空的曲率而等号右边是物质的能动量(作为弯曲时空的源)。爱因斯坦常说他喜欢方程的左边,那是美妙的、几何的,是时空的曲率;但他不喜欢方程的右边,那牵涉到“物质”,而这是不那么确定的。所以他总说方程的左边漂亮而右边丑陋。他后半生致力于把右边的东西移到左边并把物质理解为一种几何结构。而从几何中构造物质,正是弦理论所为。特别是对于异常弦理论来说,它本质上是一种引力理论,其中物质粒子和其他相互作用力如同引力一样都产生于几何。爱因斯坦应该对此感到高兴,即使不是关于具体
    
    结果,至少就其要实现的目标而言应该如此。
    
    大概他会喜欢有一个基本的原理统一所有的物理吧!
    
    他会喜欢有那样一个基本的几何原理― 不幸的是对此我们尚未真正弄懂。
    
    6 .约翰· 埃里斯(JohnE 1 1 15 )
    
    约翰· 埃里斯是欧洲核子研究组织的理论物理学家,他在建立旨在统一各种相互作用的超对称的以及规范的场理论方面起过重要作用。他因尝试将粒子物理中的新思想(目前是超弦)与观测宇宙学相联系而闻名于世。
    
    我想请您简要地总结一下您认为超弦方案能到达的目标。
    
    我认为超弦是第一个能统一自然界中基本相互作用的理论,它将约束行星绕太阳转的引力,使电子绕原子核转的电磁力、以及保持原子核完整的强相互作用或核力、还有与许多辐射衰变相关的弱相互作用统一起来。到目前为止,有些相互作用已经部分地统一起来了,但尚无真正可靠的证据表明可以在一个单一的数学图像中统一所有相互作用力。
    
    该理论的要点是什么?
    
    按照超弦的观点,所有我们原来认为是基本的粒子,也即无内部结构的点粒子,实际上并不是点,而是弦构成的小环,它们在空中移动并振动。
    
    这些弦到底是什么?我们应该怎样想象它们?
    
    让我们考虑以往的关于基本粒子的图像,这时只有一个点,当粒子运动时,可以想象它刻画出一条线,即所谓的“世界线”( wor }引1 ne )。现在,在超弦理论中,任何时刻粒子都是个小环,可想象为绳套或诸如此类的东西。随着时间的发展,这个小“绳套”在空间运动并描出一个管状的东西,叫作“世界面”( wo : ldsheet ) ,这就是超弦方案中的粒子轨迹。
    
    所以我们必须认为粒子是有内部运动和内部结构的物体,是吗?
    
    对。就如原子,我们知道它是复合粒子:电子环绕原子核旋转,而原子核由质子和中子组成,质子和中子又是由所谓的夸克构成。按照超弦理论,夸克也是有大小的,但不是由某些更基本的成分构成的。我的意思是,夸克中并没有亚夸克。它们由弦构成,但弦没有任何内部结构― 它有大小,其典型值我们相信在10 一33 厘米左右,粗略地讲,是原子核尺度的十万亿亿分之一。
    
    如果说所有不同的粒子都是由这些小环构成的,那么我们怎样区分
    
    粒子呢?为什么有如此多种我们曾以为是基本的粒子呢?
    
    我想最好用经典弦来考虑,比如我们熟知的小提琴弦,要知道当拨动琴弦时它会以不同的频率振荡― 它有不同的谐波。超弦和它有点类似。我们认为,不同的基本粒子对应于弦的不同振动方式,这同在一根小提琴弦上能拨出不同的音极为相似。事实上,超弦的不同振动方式是有限的。我们今天所见到的基本粒子(那些组成我们本身的粒子)仅仅对应于最低的谐波,好比你在琴弦上拨出的最低音。
    
    这么说那些区别,如上夸克和下夸克之间,差不多完全是因为弦的振动方式不同所造成的吧?
    
    对。超弦除了像小提琴弦在空间振动以外,还有一定的内部自由度,这些不能简单地按空间振动来想象,其真正的区别,如上夸克和下夸克之间的区别,大概是由这些内部性质以及空间振动的某种组合所造成的。
    
    能否想象有一天我们拥有足够先进的仪器,可以直接探测到这些小
    
    环― 真实地将其展示出来而不是仅仅相信它们的存在?
    
    原则上是可以的。但实际上我认为那是极端困难的,为了真正看到粒子内部的这些环状结构,必须作实验探测101 " Gev 的能区,这大约是现在的粒子加速器所能达到能量的一亿亿倍。建造这样一个加速器的费用恐怕是难以想象的,而且可能根本不具备所需要的技术。
    
    我想您是对的。不过假设我们可以达到那样高的能量的话,是否有可能把那些环剪斯从而得到开放的而不是闭合的弦呢?
    
    我相信那大概是不行的,当然那也是一种观点。有些人认为弦可以剪开,如你所说,可以有开弦和闭弦。我本人则倾向于只有闭合的弦的理论。
    但若把物质加热到极端的高温,完整的弦也许会分解。不过这仅是推测,我们无法证实。
    
    关于这些闭合的弦还有一个问题,即对于带电粒子,是不是环上带有电荷,而电荷均匀分布其上呢?
    
    让我们回到原先的出发点。不应该认为一个带电基本粒子,比如电子,是包含了更基本的组分,而这些组分各自荷载子电荷从而加起来等于电子电荷。实际上,我们所谓的电荷应该是弦的一个整体性质,当弦的振动方式不同时,它将带不同的电荷。
    
    也就是说,电荷可以看作弦的一种运动性质而不是加在粒子或基本物质上的某种东西。
    
    对。那样想很好。
    
    人们常问电荷是什么,并且通常只能把它当成仅仅是一种基本的特
    
    性,但您现在似乎说可以用某种动力学的方法来解释它?
    
    让我们回想一下电荷的真正含义。我们的理解是,有所谓的电磁场与电荷藕合,电磁场使电子绕原子核运动,我们熟知的无线电波也是电磁场。
    电磁场实际上是与光子相联系的,而光子也是弦的一种不同振动方式,如同电子一样。所以我们认为电荷实际上是那些以稍微不同方式振动着的弦的某种藕合,而光子与电子是同样基本的。
    
    超弦理论一个不同寻常之处在于这些小环是存在于十维时空而不是普通的三维空间。这有什么必要性?
    
    研究表明,在考虑了量子修正的情况下,要建立一个自洽的弦理论,而同时弦又没有其他的内部自由度的话,理论就必须建立在特殊维数的时空上。如果理论只包括像光子一样整数自旋的玻色子,则最少需要26 维,恐怕我无法给出一种十分简单的描述,这也正是需要数学的原因。
    
    如果我们让理论更全面一些,加上费米子(自旋为半整数,如电子), 则临界维数是十。当然,这离我们生活的三维空间加一维时间仍相差甚远。
    
    如果真有十维的话,怎样才能使之与我们感受的三维空间加一维时
    
    间协调起来呢?
    
    一种将计就计的办法是,为了去掉多余的六维或22 维,可以把它们像卷报纸一样卷起来。可以把通常的两维空间想象为一张纸,原来是平
    
    面的,而如果将它卷起来成为一个小管子,这时本来是两维的平面就被压缩为一维,这一维对应于管子的长度。
    所以可以类似地处理多余的时空维数,把多余的六维或22 维弯曲、卷缩起来,留下四维,像报纸管的长度一样。
    
    一定有许多不同的方法做到这一点吧?
    
    当然。人们最初的尝试是写出一组条件,而它们似乎是使卷曲机制自治所必需的,虽然这些条件十分苛刻,仍然有成千上万种不同的方式来完成卷缩。现在一些物理学家正疲于在这成千上万种可能中寻找真实世界的影子。
    
    您的意思是,它们会导致我们所观察能区中不同的物理?
    
    不错。例如某一种卷缩方式可能给出两种光子而不是一种,而另一种方式甚至可能给出三种。也许不同于我们在现实世界中看到的三种电子(这里我包括了p 和:子,它们除质量递增以外,与电子极其相似), 可能会有第四种更重的电子。但是有许多理由使人相信这是不真实的,所以人们试图建立只有三种电子和一种光子的理论。
    
    所以粒子与相互作用力的数目和性质取决于这些多余维数的卷缩方式― 即不同的拓扑结构,是吗?
    
    是的,实际上可以把电子这类粒子的数目与多余维数卷成的孔数相联系。仍以报纸打比方,可以说卷起来的报纸有一个孔。如果对着管子看,只看到在中间有一个孔。另一方面,可想象(至少抽象地)这样卷报纸,使之多于一个孔。按照这些理论中的数学,孔的数目将决定类电子粒子的数目。
    
    似乎我们在用拓扑,也即这些多余维数连接的形状来解释世界,而不是按传统的方法,认为事物原本就是那样。
    
    对。我说过,在超弦理论中,不能将目前似乎是基本的粒子看成由更基本的物体构成。人们通常设想,不同的类电子粒子是由那些更小的组分不同组合而构成的。
    但是在超弦理论中情况不是这样的。如前所述,不同的电子,是对应于想象中的“报纸”卷缩起来所得到的不同孔洞。
    
    让我们再来讨论不同的拓扑结构,您似乎是说现在还完全弄不清楚哪一种特殊的拓扑对应现实世界,并且有许多种可能性。这大概是理论的一个弱点,因为它显然不能确定一种唯一的可能性。这是否仅仅由于我们的无知?换句话说,进一步的研究是否可能找到描述现实世界的唯一拓扑结构,还是说永远无法弄清楚?
    我认为目前无法回答这个问题。一个可能是当我们进一步更好地了解了该理论,可以发现只有一种确定的方法卷“报纸”,那必然对应于真实宇宙。
    但另一种可能是,实际上有许多不同的方法卷“报纸”,而宇宙的不同区域各自选择了不同的方法。那可能意味着,在宇宙的某些区域确实存在两种光子或四种电子。现在我只能说无法在这两种可能中作出选择。
    
    我们之所以确实见到这些粒子或许是与这样一个事实有关,即除非宇宙正好是现在这个样子,否则我们将不存在― 就是说,生命无法形成。
    
    我想如果类电子或类光子粒子数目有出入,仍有可能建立某种与宇宙相关的有趣的东西。它看起来将并不正好是我们的宇宙,可能坐在一块讨论宇宙结构的物理学家与我们所见的也不同,但我想物理学家可能总是存在的。
    
    我们是否可以从动力学来理解这种卷缩呢?是否从某种意义上说存在着使多余维数卷缩起来的力,还是仅仅作为某种抽象的数学呢?
    
    在某种意义上存在这样的力。例如弦本身存在张力。比如先前提到的小提琴弦,可以通过改变弦的张力来调整音调。与此相类似,超弦也有一定的内部张力。这时内部张力取决于理论的基本结构,但它确实有着类似的性质。所以这种张力是弦所具有的内真力。
    
    是这种力使多余维数卷缩起来的吗?
    
    弦的张力确实起了重要作用。从拓扑结构来看,显然不同的卷“报纸”方式之间存在某些界限,有某种东西阻止报纸自动打开,但我们不知道在超弦的情形下那是什么。另外,我们不知道从纯理论出发,这个“报纸管”直径应是多大。我先前估计它在10 一33 厘米左右,也可能是10 一34 或10 一32 ,现在没有办法计算其绝对大小,希望将来能找到一些方法。它可能与量子修正之类的高阶效应有关,例如在导电平板间产生作用力的
    
    卡塞米尔效应。也许在超弦中有某些类似的东西,但是这尚未得到证明。
    
    关于这种自发紧致化(多余维数的卷起)的动力学似乎是一个尚未被彻底了解的课题。
    
    完全正确。也有可能整个紧致化的想法下星期就会被抛弃。现在有人试图直接在四维时空而不是26 维或十维建立理论,这样就不存在维数紧致化的问题。
    
    那怎么可能呢?
    
    粗略地讲,这是把原来认为是时空的自由度,那些多余的维数,换成纯粹的内部空间的坐标,像先前谈到的电荷。人们发现玻色子理论的26 维和包括费米子理论的十维是不必要的。如果以正确的数学方法转换这些自由度,可以建立少于26 维或十维的理论,这种数学方法讲述起来相当困难。
    
    我觉得那似乎在后退,弦理论一个引人之处是通过构造几何结构,用多余维数的形式来代替原来的抽象的内部自由度和内部性质,去除这些难道不是退步吗?
    
    也许说退步是太感情用事了。我们仅仅是跟着数学和物理发展的脚步,总的说来应是进步而不是退步。
    但很可能这些理论与原先的26 维或十维理论只是方法的不同。很有可能我们是在谈论同样的事情,仅仅语言不同而已。
    
    在结束关于高维自由度及其紧致化这个话题之前,我们所谈到的小环是否确实环绕在那些管子上,即环绕在卷起的报纸上呢?
    
    可以想象非常复杂的构形。如果仍用报纸模拟,设想有一个弦环,则可以将它绕在报纸管上一次、两次或更多。
    
    还可以扭曲它?
    
    是的,如果有更复杂的弦的话。可以设想所有的可能性。当你想象扭曲时,应该是一个有弹性的带子而不是一根弦。当然这些是理论提供的其他拓扑性质,尽管我并不认为我们真正懂得它们。
    
    似乎理论物理学家不得不深入到他们原来并不感兴趣的数学分支中,以便更好地把握超弦理论。
    
    确实。我总在书店里留意是否有数学百科全书,以便能够学习诸如同调、同伦这些数学概念以及所有这类我从前从未想过去学的东西!我们是否可以转向讨论对理论可能的实验证明?我们都同意弦理论是非常美妙动人的,但科学是完全建立在实验基础上的。对超弦理论来说有什么可信的实验依据吗?
    
    我刚才曾提到超弦提供的可能性之一是存在两种光子,甚至三种。这些额外的光子不可能是零质量的粒子,如我们的无线电节目所使用的光子。它们必须有质量,很像几年前在欧洲核子研究组织发现的W 和Z 粒子,也可能就是第二种Z 粒子,具有超弦所预言的性质。目前在欧洲核子研究组织人们做的一件事就是找寻有关这个额外的Z 粒子存在的迹象。
    
    该理论是否还预言了其他种类的粒子?
    
    除已提及的类电子粒子和相应中微子以及夸克以外,可能还有另外的物质粒子类型。它在某种意义上像夸克而又部分地类似电子,即所谓的轻子夸克。这种可能性是超弦提出的。虽然不能肯定轻子夸克确实存在,至少有理由在实验上去寻找它们。
    
    我们有多大的把握在不久的将来用实验来检验理论?
    
    很难说。我们现在对理论的理解还不足以判断新的光子或物质粒子是否真是超弦所预言的那样,甚至没法说那是真正的预言。我们不知道粒子的质量,以及在加速器中需要多大的能量才能产生它。现在我们只是在黑暗中摸索,希望能摸到些什么,也许什么也得不到!
    
    物理学家遇到这类问题时,常做的一件事是转向宇宙学寻找依据。大概在宇宙的最初期,即所谓的大爆炸时期,有许多能量释放出来,我们期望超弦活动在宇宙中留下了我们今天可以看到的痕迹,您认为是这样吗?
    
    那当然是可能的。例如,我们相信宇宙中有所谓的“暗物质”存在,它是不发光的物质,不与光子藕合,所以我们无法用望远镜看到它。但我们可以粗略地测量宇宙中不同粒子间的相互吸引力,看来在可见物质
    
    之外确实有隐藏的暗物质表现出引力效应。
    我们不知道暗物质是什么,但一种可能是它是某些从大爆炸时期遗留下来的漂浮在空中的粒子。在超弦理论中,至少有一种弦的振动方式(即一种谐波,如果你喜欢这么说的话)可能就是那样从大爆炸中遗留下来的一种稳定粒子。
    
    若超弦理论是正确的话,您是否认为早期宇宙会以非经典的模式演
    
    化,并且它的动力学也会因超弦而有所修改?
    
    我想那是肯定的。往宇宙早期推移,则所有我们发现的轻元素,如氦、氖和氖等等,大致是在宇宙诞生100 秒时产生的。大概现有的物理定律可以描述那时的过程,但若再往前走,很可能超弦会对宇宙早期的进化给出与标准模型不同的描述。不过由于对弦理论的理解尚不深入,所以还不能对这种修正作出准确的断言。但可以肯定的是我们必须考虑这样一种可能,即如果回到足够早的时刻,宇宙可能是多于三维空间加一维时间的高维时空。也许维数会增至26 维或十维。
    
    换句话说,我们谈论的卷缩并非一开始就存在的?
    
    对。很可能非常靠近宇宙起点时确实存在26 维或十维时空,随着宇宙演化,出于某些尚不十分清楚的原因,一些维数自发地卷缩起来,然后宇宙就变成了现在这个样子。
    
    从一个哲学味更浓的观点出发,历史地看待超弦理论,物理学家似乎是偶然涉足其中的。现在我们有一套数学方程式,虽然相当抽象,但似乎可以描述所有粒子和相互作用。人们不得不问为什么。是隐藏着什么更深刻的原则呢,还是仅仅出于偶然才找到了启开自然之谜的钥匙?
    
    我承认在十五六年前发现超弦确实是偶然的。事实上,当时人们并未把它当作包罗万象的理论,而仅仅认为它或许能够取代夸克描述核力。接着我们发现弦理论并不能很好地描述核力,而另外发展了规范场理论,其中基本的相互作用如电磁力、强的核力及弱相互作用,都是通过光子那样的自旋为1 的粒子传播的。最近15 年来讨论物理学的语言一直是规范场理论。
    回到弦理论,我们觉得它像是一种超级规范理论,有数目惊人的对称性,这些我们才刚开始了解。它应包括现在的规范理论,还有许多其他的东西,如爱因斯坦的广义相对论― 另一种有大量特殊对称性的理论。现在,弦理论被认为是统一四种相互作用的一种可能的理论。
    
    这引出另一个问题,我想许多外行可能感到困惑:为什么一个关于亚原子物质及其相互作用的理论需要以如此基本的方式处理引力?有什么简单的方法可以看出引力在粒子物理中的重要性?为什么引力是必要的?
    
    我们知道,基本粒子间确实存在引力。事实上,这甚至已经在实验室中观察到了。可以降低一个基本粒子的速度使它变慢,于是会发现其轨迹在地球引力下弯曲。所以基本粒子间一定存在引力,如果打算严格地提出一个包罗万象的理论,就必须在基本相互作用中包含引力。还有一个更深入的问题。自爱因斯坦及量子革命以来,存在一个从未解决的大难题,即怎样调和引力和量子理论。许多著名的物理学家曾徒劳地努力过,但从未找到正确的量子引力理论。现在好像超弦理论能够做到这一点。至少有些弦理论似乎在高阶修正上都是有限的,这对一个量子理论来说很不寻常。在此之前,人们试图建立量子引力理论时,一旦计算些什么,总是无法控制地出现无穷大,使人们不知所措。但这里我们好像找到了一个可行的理论,这也是人们对超弦理论激动不已的一个主要原因。它可能最终调和20 世纪两个最伟大的物理革命― 量子力学和爱因斯坦的广义相对论。
    
    这并非第一次尝试找寻完全统一的自然规律― 一个包罗万象的理论,但是否会是最后一次呢?
    
    这只有天知道!你让我窥视一个分明是充满云雾的水晶球。
    
    假如它是错误的,您是否认为这是最后一次机会,用简单的数学语言来描述自然?
    
    我并不认为这是最后一次机会。就我们的粒子物理实验而言,目前可以做到100Gev 的量级,即质子质量的100 倍。引力也有一个内在的质量标度,与所谓的普朗克能量有关。普朗克能量是101 " Gev ,比目前实验室可达到的能量高出许多个量级。
    也许在100Gev 到101 " Gev 之间某一处我们将最终找到构造一个包罗万象理论的所有要素。现行超弦理论是个很大胆的推测(有人甚至说它是一种发疯的想法),它假设从100Gev 附近的物理出发,我们可以跳到101 " Gev 。但那可能是一局不好赢的赌博。我们应该把实验能量提高到1000Gev 、10000Gev ,逐步积累知识,直到最终找到那个包罗万象的理论,那当然是很遥远的。
    
    即使目前建立的弦理论不能给出最终的答案,我认为它也达到了相当的高度,可以作为讨论基本物理、基本粒子物理和相对论物理的工具,在今后几年不存在被遗忘的危险。即使不能成为包罗万象的理论,或者在不远的将来不能证明这一点,它仍将保留为我们基本物理学词汇的一部分。
    
    让我们暂时把物理这个话题放一放,从社会学的角度来看一看。您曾写道,超弦已经带来一种“极端的热情”。确实,以我的经验,以往其他理论从未像超弦思想这样打动物理学界,很明显,人们的判断力受到兴奋情绪的影响。但客观地看,还留下哪些主要的问题,除维数压缩外还有其他的吗?
    
    我认为一个重要的问题就是紧致化是否确实必要,或者理论是否能以某种方式一开始就建立在四维时空上。但如果有紧致化,显然我们必须弄明白它是怎样发生的,是什么决定了“报纸”的一种卷缩方式优于其他方式,然后才能计算诸如电子和光子的种数,这是第二个极重要的问题。
    还有许多其他重要的问题。例如,我们必须知道为什么不同的基本粒子具有现在这样大小的质量,为什么某些比普朗克能标101 " Gev 轻那么多,以及那些非零的质量从何而来。我们相信质量来自一种神秘的、称为希格斯玻色子的东西,但为了给希格斯玻色子一种有意义的描述,理论必须补充某些东西。许多人相信那就是超对称,超弦的“超”字正是来自于超对称,因为超弦是超对称的弦。超对称似乎是给出粒子质量所必需的。
    
    关于自然界的四种基本相互作用我们已经谈了许多,但差不多在去年有了某种猜测,认为可能存在所谓的第五种力。如果有第五种力,是否可以纳入超弦理论中?
    
    我首先声明我并不很相信有第五种力。我认为它存在的证据非常牵强,我本人从未把它当真。有些人声称可以将第五种力纳入超弦框架之中,我还得说我是抱怀疑态度的,我宁愿静观其发展。
    
    听起来有一个包罗万象的理论确实很伟大,并且如果我们能写下并证明这样一个理论,那将是激动人心的。但那是否意味着物理学的终结,而物理学家该另谋职业了呢?
    我不这么认为。事实上大多数物理学家并非致力于发现新的自然规
    
    律,他们的工作是进一步理解自然界如何发挥那些已知的定律。他们利用前人建立的理论,或者说拉格朗日量(Lagrangian )和哈密顿量( Hami } ton 1 an )来做工作。只有基本粒子和引力论领域的物理学家在找寻新的自然规律。如果我们真的找到了一个包罗万象的理论,那么粒子物理与相对论物理就会变得像物理学的其他分支,如固体和凝聚态物理。
    
    也就是所谓的应用物理。
    
    比较之下,恐怕仍有应用和非应用之分。
    
    让我们回到理论的实验证明这个问题上来。大的加速器十分昂贵,我们不能期望将来会有许多大很多的加速器。所以我觉得似乎检验超弦(或其他包罗万象的理论)的重任将落在可以预见的加速器设计上。您在欧洲核子研究组织工作,那里现在建造的一个主要设备是所谓的LEp 加速器。LEp 是否有可能检验某些我们所谈论的想法?它有没有足够的能量探索验证超弦所需的能区?
    
    我想L 印很有可能找到第二个Z 粒子,尽管那也许很不可靠。或许我们只有找到超弦粒子的间接证据才会感到满意。LEp 计划做的一个实验便是非常仔细地观察第一个Z 粒子的性质。从不同角度详细地检查该粒子性质将有助于我们了解附近是否隐藏着另一个粒子。
    另外,在L 印上我们可以准确地数出宇宙中基本粒子的种数,于是按照超弦理论,这应该使我们能够约束紧致化空间的拓扑结构,并在某种意义上辨别“报纸”是如何卷缩起来的。
    还有一种可能,超弦理论中存在某种类型的额外粒子,也许可以在LEp 或其他加速器上找到它们,比如行为像传统的夸克和电子组合而成的轻子夸克。据信某些粒子能在L 印上产生。
    
    什么时候可望得到结果?
    
    噢,L 印定于1989 年开始做实验。
    
    最近几个月关于英国能否支付得起钱,使自己留在欧洲核子研究组织实验室,以继续作为这个组织的一员的问题存在一些争议。如果英国在验证这些激动人心的观点的时刻退出这个组织,这对英国的科学发展实际上是否可以说是个很大的打击?
    我想如果那样的话,将有力地表明英国将退出这门特殊的基础科
    
    学。请记住,科学需要创造理论,但同时必须用实验来验证。从欧洲核子研究组织退出意味着基本上断绝了做实验的可能,而没有实验,我无法想象能搞什么科学。
    
    7 .阿l 、杜斯· 萨拉姆(Abdus Sa 1 am
    
    阿卜杜斯· 萨拉姆是特里雅斯特(丁rieste )国际理论物理中心主任和伦敦帝国学院物理系教授。他曾对粒子物理和量子引力中的许多发展作出了贡献,并因在电弱统一上的工作被授予诺贝尔奖。近年来他把注意力转向了超弦理论。
    
    一百多年前,人们曾普遍认为物理学正在接近完结:牛顿力学,麦克斯韦电磁学和物理学的其他部分确实描述了自然界的一切,要做的只是填补最后一点细节了。然而随着世纪之交我们可以称为“新物理”的来临这一切都破灭了。但现在,尽管还很模糊,我们又有了这种感觉,要出现一个把自然界的一切都包含在一个统一的描述里的,包罗万象的理论的提纲。这是否仅仅是一个幻影,还是我们真的正在接近理论物理学的顶峰?对于这些新观念您有多大的热情?
    
    如果你是问超弦理论和它的重要性,我是热情的。不过,说到我们能达到一个包罗万象的理论,我个人并不相信。无论如何,我们不应该在一个理论可检验的范围之外相信它。现在,这个包罗万象的理论宣称可以告诉我们普朗克能量(101 " Gev )以下的所有现象。要在普朗克能量下直接检验一个理论,我们就得有这么高能量的加速器。在可预见的将来设计这样的加速器将至少长达10 光年!所以我们将永远无法直接检验任何在比方说107Gev 以上成立的包罗万象的理论。只有间接检验是可能的,但这些不能包容一切。超弦理论之所以令人激动是因为它本身的内在优点,至少我们发现了一个点粒子场论的真正替代品,正是这个点粒子的概念使我们过去的量子引力理论遇到难以克服的困难。
    
    为什么我们想取代点粒子?
    
    因为这使我们有希望建立一个量子的引力理论― 这是第一次,这是一个胜利,不管我们是否得到一个包罗万象的最终理论。这个引力理论还统一了夸克,统一了规范粒子― 光子,W 和Z 粒子,这是个额外的优惠,不过即使没有这种统一,我也会认为超弦是个重大的发展。
    
    弦和点粒子到底有什么区别,使它有这样好的前景?
    
    我们用有限大小的东西― 10 一33 厘米大小的弦代替点粒子。弦理论提供了玻尔会喜爱的东西― 一个有限的基本长度― 10 一33 厘米。但是
    
    尽管有这有限的长度,理论还是局域的。这是其妙不可言之处。
    
    在什么意义上说它是局域的?
    
    在它保存了因果性的意义上,类空的事件彼此不会互相干扰。弦理论的妙处在于,虽然我们处理的是延展的客体,弦的相互作用却发生在一点― 它们并不发生在整个弦上,弦在它们上的一个点分开或重连,并且弦彼此间也仅在一点处接触,这就是它们局域性的奥妙所在。
    
    那么弦不仅是物质粒子的一个模型,还是这些粒子的相互作用的模
    
    型?
    
    是的,从这个观点看,弦能否解释全部物理是第二位的。弦理论的提出已有十多年了,但就连它们最热烈的提倡者也没对这个特别的优点给予足够的强调― 它们可以提供一个局域的、满足因果关系的量子引力理论。
    
    是什么原因使它们突然这么流行?
    
    这理论可以消除反常,如果把引力与一特定的杨一米尔斯(丫ang 一Mi } } s )场统一起来的话。格林和施瓦茨的这个发现使得一方面可以把引力和一套特定的杨一米尔斯规范粒子统一起来,另一方面,反常的消除使这理论可能是有限的。
    
    我们还需要严格证明这理论给出有限的结果,不过,前景看来很好。
    
    您能解释一下有限性是怎么回事吗?
    
    过去提出的量子引力理论多数都给出无限大的结果。设想一个人想算一下一个引力子被另一个引力子的散射,弦以前的量子引力理论的回答总是:无限大。这样的理论是没有用的,也是不自洽的。超弦理论是第一个量子引力理论,例如保证给出引力子被引力子散射的有限结果。有趣的是,在以弦为基础的量子引力理论中引力并不是被强调的部分。
    这里我想重复一下帝国学院克里斯· 依沙姆(Ch : 15 } sham )的话。他说当他是个学生的时候,他开始在量子引力领域里工作时,希望在引力理论中加入适当的一致性条件有可能解答引力为什么应被量子化的奥
    
    秘。换句话说,他想从爱因斯坦的广义协变性导出普朗克量子化。弦理论表明情况恰好相反,我们发现普朗克量子化必须首先被引入,而爱因斯坦的引力将作为导出的概念出现,这是由于我们赋予弦的一种性质一一标度不变性,这是爱因斯坦的同事荷尔曼· 魏耳(Hermanwey } )引入的,而爱因斯坦并不喜欢。
    
    我想爱因斯坦一定会心烦意乱。
    
    他会心烦意乱的,他曾温和地责怪魏耳用他的观念把所有的人引入歧途。应当承认,魏耳的观念是在四维时空、没有弦概念的情况下引入的。不过,爱因斯坦写信给魏耳说,他到老也不会接受这些观念。
    
    如果这些超弦理论是有限的话,那确实很有说服力。不过,最好是有些确定的可以检验的新预言,而不仅仅是重新得出我们已经知道的物理。这些新观念是否将导出些确定的预言?
    
    噢,是的。有一些预言。比如,几乎所有超弦理论都预言有一个新的Z0 粒子。一个Z0 粒子就像一个重光子,一旦我们知道它的质量,将是一件很重要的事。我们现在还不知道这个质量应该是多大,但是设想它能被一个弦理论所预言并计算出来― 就像弦以前老的Z0 那样― 你或许能记起那是电磁力和弱核力统一的信号。新Z0 将提供所有相互作用力― 电弱的,加上强核的,加上引力的― 被弦所统一的决定性检验。
    
    那个质量是否在可以达到的范围内?
    
    这我们还不知道,一个象样的有预言力的质量理论还不存在。
    
    但是在弦的情况下,有一个关于被隐藏物质的预言― 一种新型的物质,这也可能会有它自己的生命力。
    
    这些现在还都是些苗头。我不能说这是像老的W 和Z0 粒子那样可靠的预言,它们已被鲁比亚(Rubb ia )验证了。我们希望能有这样可靠的预言并被验证。
    
    您刚才谈到超弦理论一个很迷人的方面,即由我们的物质的一个拷贝所形成的另一个宇宙。您能简单地描述一下吗?
    这个观念是,有一个副本宇宙,它与我们仅仅通过引力相联系。有
    
    趣的是这个不可见宇宙将决定我们的宇宙里超对称破缺的方式。这样一个理论将揭示:是什么决定了可见宇宙中的质量差。
    
    换句话说,另一个宇宙的存在将通过基本粒子的质量显示出来?并且我想我们的宇宙将对它有同样的影响?推测起来这应是一种对称的安排。
    
    我想是这样。我不认为任何人已经详细考虑过另一个宇宙将是什么样。谈到这另外的宇宙,您说我们只通过引力与之相互作用。那么,比方说我们将注意到其中的一个黑洞,但却无法发觉这屋里来自那个宇宙中的原子,这些原子将径直穿透我们?
    
    大黑洞除了它们产生的引力效果外也不会显示出来,就像《一千零一夜》 中的不可见的神灵。不可见宇宙中可能也有它自己的带电荷的夸克,自己的W ,自己的光子。不过这些光子不能使不可见物质闪光,因为它们不与可见宇宙中的任何东西相互作用。不过我想强调一下,超弦理论并不是唯一假设存在这种有引力但在其他方面则不可见的宇宙的理论。
    
    超弦和宇宙弦有什么联系吗?
    
    可能有。这种联系的可能的理由是,我们的宇宙在开始时很小,里面包含小的弦,当宇宙暴胀时这些弦被拽开了。
    
    爱德华· 魏廷对这些考虑得比别的任何人都多。我倒乐于知道他的观点。
    
    再回到超弦理论面临的问题上来,您曾提到有限性问题的重要性,因为理论有限就是一个重大的胜利。那么理论的发展有没有其他的问题或障碍呢?
    
    即使理论是有限的,要用它计算,也还是让人头痛的事情!
    
    那不正是我们用上研究生的地方吗!
    
    不!研究生不行。在最清晰易解的表述形式中,超弦也必须处在一个十维空间中:九维空间加一维时间。困难出在我们想得到一个我们生活的四维时空中理论,怎样紧致化其它六维,也许我们能训练电脑做这
    
    件事,但研究生不成。
    
    这个包罗万象的理论竟是如此不可思议的复杂,以致我们无法取得进展,这是否意味着有什么地方搞错了?大自然不应该是简单的吗?
    
    大自然是简单的,如果我们用正确的方式看待它,比方说吧,我相信上帝只创造了两个自由度:一维空间,一维时间,什么能更简单呢?然后在某个时刻,有一个到四维的相变,加上六维内部空间,二维是这事的核心,而且这个数不会变成三。
    
    你能在二维中建立起超弦的理论吗?
    
    可以,并且正如玻利亚可夫(Po } yakov )所指出的那样,这是最简单的情况,我相信理论是这样开始的:在二维中有十个基本的场,我们需要这个“十”来消除共形反常,摆脱至少一种潜在的无穷大,这十个场中的一部分会表现为四维时空,其余的六维则紧致化为内部自由度一一表现为电子或核子数,在这一图像中四维时空开始于相变的时刻。
    
    空间如何能够在不同维数间跳跃呢?
    
    这是一种普通的相变― 如果我们能在二维理论中激发它们,我是说“如果”,现在还没人能做到,但这是我的梦想。
    
    据说从十到四是个很大的问题?
    
    这不假,那是超弦理论中宇宙的复杂之处。
    
    您是否认为变换维数的问题和证明有限性的问题同样困难?
    
    不,我想证明有限性比较困难,因为要用到不熟悉的黎曼面的数学,显然塔赫穆勒(Teichmu } } er )在这个领域享有盛名― 他是二战期间死去的一位数学家。
    
    但是现在有一大批理论物理学家在忙于从事这方面的研究?
    
    并不是一大批,大多数人是忙着把十维变成四维时空加六维内部空间,这比较简单,现在已发现有许多途径能够做到这一点― 不过还没有哪一个足够优美使人心悦诚服,更难的任务― 有限性― 涉及弦的
    
    高阶圈图,它没有吸引很多人为之忙碌是因为太难。
    
    您能否描述一下这些小弦是什么样的?它们是闭合的还是开放的?描述引力的理论必须对应于闭合弦,这些弦的振动对应于物理粒
    
    子,这些粒子必须具有自旋1 , 2 , 3 .· · … 。自旋为1 和2 的粒子是无质量的,相应于弦的零频,而高自旋的粒子将具有普朗克质量倍数的质量,
    
    即101 ”质子质量,自旋5 / 2 , 7 / 2 .· · … 的粒子的质量也是以普朗克质量为单位的。
    
    建立自旋大于2 的物体的理论没有什么数学上的困难吗?
    
    这是理论的一大奇迹,正是由于高自旋它才有限,而更令人难以置信的是这理论还是局域的。
    
    这样的认识是否正确:我们一直以为是粒子的其实是一个在低能量下没有扭动的闭合弦圈?
    
    不,我们谈论的不是单个粒子,弦描述的是一整套高自旋的客体,它们是一块儿的。
    
    那么这样说是否正确:绕,而是在高维空间中环绕
    
    这些弦圈不是在我们看到的三维空间中环
    
    不,那不正确,这弦圈是四维时空中的圈,并可能在多余的六维内部空间中扭曲。
    
    我们的加速器能否真的让我们看到自旋大于2 的粒子?
    
    它们的质量都是普朗克质量量级的,所以它们不是在可预期的实验中能直接达到的。
    
    可是如果探测到任何新粒子,那将是很重大的。
    
    噢,是的。比方说,如果理论明确预言一个另外的Z0 粒子具有实验可达到的质量,那确实是重大的。
    
    那么如果这真的发生了,而超弦理论成为公认的关于物质和力的理论,又将怎样?您说过实际计算是难的可怕,那我们怎么办?难道我们
    
    仅仅盯着这些公式惊叹,把公式贴到墙上并说“这是一个伟大的成就”就完了?
    
    历来如此,拿爱因斯坦的引力(理论)来说,当那三个著名的实验证明爱因斯坦理论比其竞争者要好后,我们就接受它作为真理,后来很长一段时间没有再(用它)作任何进一步的计算,因为太复杂了。
    
    为什么只限于考虑弦呢?我们干吗不考虑更多的自由度?比方说
    
    膜?
    
    现在我们有一个否定性定理,它说你不能建立高维物体比如说膜的共形不变理论,所以从膜得不到什么,这是我个人不喜欢的否定性定理,因为这些定理证明中常有许多没写出来的假定,不过有这么个定理― 这是个挑战。
    
    显然弦理论深深植根于几何学,我想人们可以说科学始于几何(如果他追溯到古希腊),要是最终我们通过几何构造出世界上所有基本的东西,那将是非常迷人的。
    
    我最近和克里斯托弗· 塞曼(Ch : 1 stopherZeeman ) ― 建立瓦维克数学研究所(Warwick Instit 日teofMathematics )的拓扑学家谈过话,我问他怎样区分几何和分析,他说数学家们有一个简单的检验办法:一个人如果秃顶,他一定是个分析学家,如果他有很多头发,他一定是个几何学家!
    
    确实,看来随着粒子物理学的发展,需要引入越来越抽象的结构,以及越来越难懂的数学分支。
    
    我很高兴你能提到这点,因为这也是另一件使我很感兴趣的事,瑞茨· 约斯特(Resjost )有一次说,自从量子理论被发明以后,一个年轻人所要学的所有数学就是希腊和拉丁字母表的基本知识,以便给他的方程填满指标,如今再不是这样了!最近几年中我们看到拓扑学,同伦,上同调论和卡拉比一丘(Ca } abi 一丫au )空间,黎曼面,模空间― 真正的、活生生的数学正在渗透到物理中来,我们了解更多真正的数学,就可以具有更深的洞察力。
    
    有一天,我和我的合作者约翰· 斯特莱思迪(John strathd 一ee )谈话。我赞叹我们今天学到的真正的数学― 不是人造的数学,他说,你不觉
    
    得那会毁了我们的脑子吗?就像伯特兰· 罗素(BertrandR 日ssel )在他的自传中抱怨的,写作《数学原理》 的工作给他的头脑造成了永久的损伤,我想起了刘易斯。卡罗尔(Lewi sCa : : ol } )的诗《威廉老爹》 ( 0 1 dFatherwi 1 1 iam ) :
    
    威廉老爹你已老白发如雪鬓若霜却还常常拿大顶偌大年纪岂妥当?
    
    威廉老爹答子日:年轻时只怕把脑使现在既知本没有多练几趟又何妨。
    
    8 .希尔顿.格拉肖(She 1 donG 1 ashow )
    
    希尔顿· 格拉肖是哈佛大学的一位在希格斯粒子方面深有研究的教授,并且与波士顿大学和休斯顿大学的同僚们有很深的接触。他在粒子物理理论的许多方面已作出了重要的贡献,并由于在弱作用和电磁作用的统一中所做的基础性工作而获诺贝尔奖,他还积极投身于科学教育。从科学和哲学上讲,格拉肖是一位彻底的超弦理论的反对者,他说自己“正在等待超弦的断裂”。
    
    我能以一百年以前,人们普遍相信物理学走到了尽头,剩下的只是一些为“i ”加上一点和为“t ”加上一横的小事来开始谈话吗?最近,这种观点好像是又被提出来了。有一些人正在谈论着理论物理的登峰造极,谈论着关于自然界的一种包罗万象的理论。您是否认为这又是一个假象呢?
    
    理论物理当然还没有走到尽头。例如,在被称作凝聚态物理的研究中,人们非常迅速而又异常兴奋地找到了一个又一个的新发现。我想你在谈论的只是基本粒子物理而不是整个物理学。当今,粒子物理学家们正处于一个异常激动的境地,因为他们正和研究宇宙学的朋友们联起手来。这次,我们有了这样一个理论,它描述的是微观世界(高能量与短距离的世界)和宇宙的诞生以及我们所能感知到的世界的起源,因此在宇宙学家和基本粒子物理学家之间的这一新联合― 出于偶然的原因,由拥有一大批天体物理学家的美国费米实验室(Fermi lab )所代表― 标志着一种复兴,而不是垂死。
    
    但是,我们首次真正能够在很基础的水平上写出一个对自然界万物都起作用的完备的理论(一种包括所有粒子和所有力的理论)的希望是否只是幻觉而已?
    
    到目前为止,我们能够写出一个只考虑到核力和电磁作用力,而不考虑引力的关于所有基本粒子的理论。这一理论是构想出来的,而且是专为一些特定目的而构想的,具有一些内在的疑点。例如,为什么粒子质量之间的比例正好是现在这个样子呢?
    我们还不具备一种包含引力在内的理论。我们可能有了这样一种理论的开端― 但仅仅只是开端而已。我的那些研究弦理论的朋友们,在目前看来是真正的统一理论(即包括引力)的领域中工作,他们说,至少得花上20 年的时间才能开始在引力世界和基本粒子物理世界之间取得
    
    联系。
    
    看起来他们对于已经掌握了一个真正的统一理论的精髓这一点非常
    
    自信。
    
    正如爱德华· 魏廷所说,他们有这样一种感觉,即在有任何理由相信拥有了新的理论之前他们必须要建立5 个新的数学分支学科。事实上,他们还没有一个理论,有的只是一些观念的组合,这些观念并非显而易见地构成什么理论,同时,他们甚至也不能认定这样的结构是否概述了那些已经在实验室以及理论物理的研究中所取得的成果。
    
    您认为他们乐观的原因是什么?
    
    他们觉得他们第一次得到了关于引力的一个自洽量子理论,或许他们确信他们得到了唯一自洽的引力量子理论。这可能是真的,也可能不是。这一理论有可能是真实的,并且这一次他们认为爱因斯坦的梦想终于有机会成为现实了。我经常喜欢回忆爱因斯坦,在他生命的最后30 年里,他一直追求这一梦想,因而似乎对那段时期内在核物理学领域出现的激动人心的发展一无所知。
    
    您曾在一次演讲中说物理学家们似乎分立成两大阵营;炼金术士和中世纪神学家。您这句话是什么意思?
    
    我对那些研究弦理论的朋友们感到特别恼火,因为他们不能对客观物理世界作任何说明。他们当中有些人对其理论的优美性、唯一性,以及正确性都深信不疑。那么既然理论是唯一并且正确的,它理所当然地包含了对整个物理世界的描写。在他们看来,无须用任何实验去证实这样一个不证自明的真理,由此他们开始抨击实验的价值。而在英国,我们的有些朋友正从另一个方面,亦即从纯粹经济的角度来攻击物理学。
    
    因此您认为这种试图用非常抽象的方式去统一描述自然界万物的趋势,实际上是对物理学未来的威胁,因为他们动摇了对实验的信念,是吗?
    
    是的,我想这正如中世纪的神学毁坏了当时欧洲的科学一样。毕竟,正是在欧洲,人们才没有看见1054 年发生的超新星,因为那时他们正忙于争论一个针尖上能容纳多少位天使跳舞这一无聊的问题。
    
    但是不管人们怎么去考虑这些理论的细节,在我们希望去探索而又无法达到的高能领域里有着很多非常有趣的物理。这一说法是确实的,对吗?
    
    这不好说,它还不是很清楚,有些观点认为在那些高能区内不会有令人感兴趣的尚未发现的粒子,那里只是荒漠一片,没有任何粒子。另外的理论却声称在那片所谓的荒漠里充满了巫待发现的新事物。我不知道弦理论家们相信的是什么。我认为他们不知道该相信什么,因为他们无法同低能量情况建立联系,因而也就不能断定那一能区是否存在新的物理,但他们并不在乎,因为无论出现什么情况,他们的理论只要建立得适当,大抵总能解释得通。
    
    即使在这片荒漠中会有所发现,为理解所有的物理,为真正领会这种彻底的统一,我们可能必须直接涉及普朗克能量这些极高的能量。
    
    所以分界线开始起作用了,但我不知道那一分界线的划法是否正确。普朗克能量和基本粒子物理之间的相关性还没有人真正建立起来,它只是来自牛顿引力的一个包含质量量纲的数字。如果愿意,你可称之为普朗克质量,它也许会有某种重要作用,也可能没有。
    
    当然,有人会从您倡导的观点的反面来说向大统一迈进是行将成功,非常漂亮,非常激动人心的,并且会很好地刺激实验粒子物理的进一步工作,而不是阻碍其发展。用建立一个世界大统一理论的目标来劝说人们去为理论的建立付出些什么,劝说人们慷慨解囊以帮助检验这一理论,您认为会有这种可能吗?
    
    会的,只要我们的超弦空想家们认为有必要达到一个我们能支付得起并且能达到的更高的能量的话。如果他们争论说需要有实验,我将举双手赞成。但这不像是他们当中多数人的论点,他们当中很多人只是抽象地确信,建造更大的加速器固然好,就如同找到一种治疗癌症的方法不是坏事一样,但这和他们的工作毫不相干。他们不是高能发展的推动力,只有我们的那些想更多地观察宇宙,搞实验的朋友们才是物理学的真正动力,并且一直就是这样的。
    
    谁将是胜者?
    
    我希望实验工作者们是。我认为观察世界从而了解世界的这一老传统将继续保留下来,同时,我们决不可能只用纯粹理论本身来成功地解
    
    决基本粒子物理中的问题。
    现在有关实验物理出现了这样一个问题,有相当数量的实验结果是误报的,这些“重大成果”事实上无法被证实。您是否认为实验工作者们在宣布他们的结果时变得有点不大慎重了呢?
    
    实验工作者们向来就不谨慎,我敢保证过去的错误发现和现在的一样多,现在看似多一些只是因为现在没有真正的发现来陪衬而已,至少在过去的5 到10 年内是如此。这不是马虎的问题,而是可能影响今天的实验工作者获得全社会的经济与道德支持的一个问题。
    你已经谈过了新加速器的不断增长的费用。这一点千真万确,那就是现在的每一个新加速器都比过去的要昂贵得多。然而,今天我们的加速器越来越少了,因为几十年前,美国有30 个大的加速器,而现在只有3 个,同时我们还正在集中经费投资于数量越来越少的设备,也许有一天全世界将只拥有一台巨型的设备。反正我们国内的科研经费真正说来是在下降,估计欧洲的情况也如此。随着时间的推移,用不变货币单位计算,花费在粒子物理上的开支是在减少的,其结果是士气开始跌落。我希望欧洲核子研究组织LEp 的布署,以及其即将产生的使人惊异的新发现会逐步改变这一局面。
    
    您已提及过在诸如凝聚态物理的领域中真正的发现具有很高比例,而此领域在国家科研预算中只占很小的一部分。目前,英国正就如何在大的消费者(像粒子物理学家和天体物理学家)与那些在对社会或许更有潜在作用的其他领域中工作的人们之间进行预算分配的问题展开激烈的争论。您对将预算中的大部分分给粒子物理有何感想?虽然预算有所减少,但还是一笔巨大经费,不是吗?
    
    根本就不能说经费的绝大部分分配给了粒子物理,人们如何计算这个问题非常复杂。例如在我们国家,生物研究方面花费的钱就比在整个物理领域花费的钱要多,差不多相差一个数量级的大小。我相信这种情况也发生在英国,如固体物理学目前在那里可能就没有得到足够的支持,其原因可能和表面看上去所感受的不同。我认为英国在欧洲核子研究组织中分担的经费,和其在国内大概与之持平的基本粒子研究经费并不代表加给英国的额外负担。英国人说他们不能继续支付这笔钱,相反,意大利才刚刚把基本粒子物理预算翻了一番。难道英国就真是一个比意大利更贫穷,更吝音的国家吗?
    
    回到真正的物理学上来,您认为目前实验粒子物理学家所面临的最引人注目的问题是什么?
    
    有许多。一个问题是钱,就像刚才所讲的那样,另一个是新设备的及时出现。在欧洲,我们正在期待L 印的建立,它正以可以合理预期到的最快速度进展,但是从一台新机器的设想到最终动工兴建大概需10 年左右的时间,因此还有几年的时间要等。这就很难对我们年轻的实验工作者们有何激励了,因此也就很难相信在LEp 竣工之后就能有一大批较年轻的工作者马上投入LEp 的实验。
    还有一个问题是关于在新设备上工作人员队伍的规模。还是说LEp 吧,它有一个包括400 多名博士的实验工作组。这样就能工作了吗?像这样一个有400 多名科学家的团体能像当年的迈克尔· 法拉第一样工作吗?我不知道。当然,这是一种不同的模式,但那些聪明人真能出类拔萃吗?那些真正有大贡献的人能受到训练,并能在那群一起工作的科学家中出人头地吗?令人惊奇的是,到目前为止,我们在欧洲核子研究组织的经验是成功的,因此上面两个问题的回答是肯定的,我希望这一经验能继续下去。
    
    回顾过去大约10 年里粒子物理的发展,看起来使人觉得缺乏令人惊奇的新发现。这不是正好可能预示着粒子物理开始走到尽头了吗?既然情况如此,我们是否应该在未来的10 年中花费更多金钱去寻找这方面的新东西呢?
    
    我当然希望我们继续在粒子物理方面进行探索。如果探索的前方真是一个沙漠,唯一可以证明的办法是走进沙漠几公里。有一点是真的,即有很多更具戏剧性的理论的预言和期望还没有筛选出来。例如,大统一理论预言质子会衰变,但这一现象还没有观察到,这一理论还预言磁单极子的存在,但也没有找到过。正是这一理论预言了只有在高能量时上述现象才会发生。那么这些预言落空的事实可能意味着这些独创的幼稚理论(我说它们幼稚是因为我有点被它们给弄糊涂了)是错误的。因此前方不是沙漠,而是大量有趣的事物巫待人们去发掘。
    例如,当今众多反常中最终存留下来的一个令人着迷的例子是关于中微子的问题。我们都知道来自太阳的中微子比我们的太阳理论所预言的要少,因此我们用研究更宽的中微子谱来证实我们的理论。我们知道,如果谁拥有了30 吨稼,那他就可相对有效地证实这一理论。这个实验现在正在意大利和俄罗斯进行,太阳中微子也正被日本人用另一种方法进行研究。这些实验将告诉我们是否我们的太阳理论有很大的错误,或者从另一方面说,是否中微子有质量并且遵从简谐振动的规律。不管是怎样的,这都将代表着基本粒子物理领域中的一个辉煌的新进展。从其他领域探索太阳结构的实验工作者们正在越来越多地了解太阳。他们正在研究太阳地震引起的太阳运动,由此发现太阳应产生多于
    
    那些最幼稚的理论所预言的中微子。由此看出在整个研究领域中一定存在一些非常不合理的东西,也就是说存在着一些使人感到震惊的阴影,而笼罩在这阴影上的面纱最终一定是要被揭开的。
    从一个不同的角度看,近年来又有了一些使人心醉的新发现。你说近年没有使人惊奇的发现,但事实上还是有一些的。有一件奇怪的事,宇宙中物质的绝大部分都是不可见的,直到五六年前,天文学家们才开始发现这一点,这使得他们感到非常窘迫。他们曾自认为是研究宇宙中物质(所有宇宙中物质)的人,事实上,他们发现自己只是观察到宇宙中由于某种离奇的缘故碰巧能发光的一点点尘埃而已。大部分物质是不可见的。那么组成这部分物质的形式是什么呢?这对天文学家、粒子物理学家和实验工作者来说同样都是一个问题。也许我们能够在实验室中,在地球上探测到这些使人发狂的物质,从而辨认出它们是什么东西。
    
    展望未来10 年或20 年,假定所称的超导超级碰撞机已在美国建成,它将产生在几年前认为不可能想象的高能,那时最优先考虑的实验将是什么呢?那时又将要去寻找什么样的新东西呢?
    
    奇迹的美妙之处就在于我们不能完全知道奇迹是什么,因此我们所能做的只是去设计力所能及的最好实验范围。当然,在美国,当超导超级碰撞机建成后,我们也只是简单地重复欧洲人已做过两次的实验,不过是在更高能量区域里罢了。我们将在极高能量下,采取和欧洲核子研究组织现在几乎完全一样的作法,使质子和反质子碰撞,或者很可能是使质子与质子碰撞,与欧洲核子研究组织相比,我们的对撞粒子的能量比它们的高几乎100 倍。对撞大体上来说就是,让一个质子和一个朝相反方向运动的反质子相碰撞,同时在碰撞点周围放置着精密的,包括最新固体物理技术的探测器。碰巧,这就是一个凝聚态物理和粒子物理之间存在诸多反馈的领域。我们建造最好的探测器(和加速器相比,其造价只占加速器的十分之一),然后我们可以安坐在探测器后等待奇迹的出现。
    
    但是肯定会有些东西是您期望看到的吧?现行的理论告诉您应该看到而目前的技术又使您无法看到的是些什么呢?
    
    标准理论告诉我们应该看到标准的事物。我们将再次看见那些已在较低能量下见到过的喷注现象和其他奇怪的现象,只不过是能量更高而已。我们将能更好地检验量子色动力学和弱电统一理论。但我们的标准理论只是预言标准的结果,而“真正”的理论(非今天的标准理论)说些什么我们完全不知道。也许有新的力,新的粒子出现,也许将有曾被
    
    广泛地称作微光的东西或者是其他一些东西出现,而我的那些搞理论研究的同事给这些东西取了千奇百怪的名字。我们还不能预言它们究竟是什么东西,也许它们是动量异常不均衡的喷注,也许它们会是在碰撞中出现的用目前的标准理论无法解释的单个轻子,也许出现的是根本不属于我们今天的哲学范围内的长寿的粒子,或许还是其他一些我不可能告诉你名字的东西,因为毕竟这是无法预料的,而整个理论正是如此。
    
    能谈谈所谓的希格斯粒子吗?那是当今正在寻找的非常重要的一个粒子,是吗?
    
    这是个难以回答的问题,希格斯粒子可能会出现在L 印中。请记住,LEp 是一架还没有启用过的庞大机器,它根本就还没有建成。一旦建成,很有可能在某种可能的质量范围内找到希格斯粒子,同时超导超级对撞机也可能在另一质量范围内捕捉到它,在这一质量范围内,一个希格斯粒子将会衰变成两个W 和两个Z 粒子。是的,我们有很大的机遇找到希格斯玻色子,或者是在粒子质量相对来说较轻时在L 印上找到,或是在粒子质量相对来说较重时在超导超级碰撞机上找到。很凑巧,这正是标准理论所不能预言的东西之一― 希格斯粒子的质量。
    
    什么是超对称性?您认为这些新机器能够有机会揭示超对称性吗?超对称是最近很长一段时间来最精彩的观念,但现在好像又无半分证据来证明世界是超对称的。什么时候,我们才能见到目前已知的粒子的超对称对偶粒子呢?
    
    超对称是很有趣的笑料。你应该记得不久以前,在欧洲核子研究组织公布的实验数据中有一些严重不合常规的东西,并且在还不能用标准理论来解释的时候,有一些人就趁机跳出来了,大力鼓吹用超对称性去解释这些反常的东西。曾有3 种不同的超对称性理论得以发表用来解释这件事。现在我们见到欧洲核子研究组织那时发表的反常东西都被修正过来了,实际上并不存在不合常规,但是在更高能量区域的研究中,类似事件可能还会重演。也许欧洲核子研究组织本身在积累更多数据之后就会再次发生类似事情。无论如何,有可能在某一天超对称性被证实不仅漂亮,而且是正确的。于是会有彩色力理论― 另一个狂人理论,也称作复合物质理论,这一理论认为夸克由其他物质组成。毕竟,所有这些东西都可能会被证实为正确的,这当然令人无比惊奇了。
    
    除了在高能量下将聚在一起的粒子撞碎的技术之外,还有另外一种方法有可能用来探索高能领域中的一些现象。我所想的是质子衰变,或
    
    者是寻找宇宙中早先的残留物。这些研究是在低能量范围内的,同时也是可能的低预算实验,但它们却能异曲同工地去探索高能量领域。您认为利用这种实验方法的时代已经随着质子衰变实验的失败而结束了吗?
    
    没有。正如你所说,在质子衰变方面作的都是一些不太昂贵的实验,但也很明显,较多的资金投入能使这些实验的结果有一定的提高。正因为如此,日本人希望建造一个更大一些的质子衰变装置,这一装置比当前日本所用的大22 倍,它能够和在大加速器中进行的实验相提并论,成为一项昂贵的实验。同样,在意大利已经建立了一个巨大的地下实验室,该实验室的主要任务就是作你所描述的那类实验。这些实验中有一些将是非常昂贵的,有可能要花费掉5000 万英镑的资金,这绝不是一个便宜的实验。
    
    您认为那些不用加速器,而又能探测高能,并且与现有的实验设计完全不同的实验还占有一席之地吗?
    
    我想我们对正反事物都需要一点。顺便说一下在作质子衰变实验时得到的非常有趣的意外收获。日本人发明了一种专为衰变实验设计的20 英寸光电管,同时也高兴地发现这种管子拥有一个真正的商业市场,因此他们就制造这种管子出售,而英国的电子材料情报中心却不能生产。另一方面,如果人们不去作质子衰变实验的话,是不会造出这种20 英寸的管子的。因此我想我们对各种方面,各种各样的研究都有需要。这儿还有一个关于偶然发现的真实例子。日本人和美国人都没有在寻找质子衰变中取得成功,但是因为有了那些已经投入应用的探测器,他们却能够探测到来自新的超新星的中微子,这样就验证了天体物理学家的理论假定,也给中微子的质量定出了新的极限值。奇迹真是可以来自任何方面。
    我又想到了另一个科学的副产品,即将在美国兴建的超导超级对撞机包括一条非常长的隧道,因此现在美国正在隧道技术,以及怎样便宜地挖掘这一大隧道上大下功夫,有了那种技术,也许我们能只花半价就开通连接英法的隧道。
    
    非常有趣的想法!回到超弦话题上来,您认为这一理论将怎么发展?
    
    我很高兴我的好些同事们埋头于弦理论研究,因为他们这样做正好有效地使我能摆脱他们。那些弦理论与我所了解并且热爱的客观物理世界简直是格格不入,因此我对这些理论也就没有什么兴趣了。对这些在美国和英国进行这方面理论研究的人,我是极为钦佩的。而同时,我将
    
    尽我所能将这一比艾滋病更具传染性的超弦理论拒于哈佛之外,但很遗憾我的工作并不十分成功。无论怎样,有些哈佛人仍然正在试图找到一条向上的路,一条从实验到理论的路,而不是去追随那个超弦观念,一个要用做梦也无法达到的高能来建立一个关于脚下现实世界的理论的观念。
    
    承认围绕弦理论研究存在这种流行趋势,那么您认为现在正在进行
    
    的物理学研究在发展道路上和50 年前相比较是否有所变化呢?
    
    绝对没有。向来都是狂人们才喜欢突发奇想。这些人中最狂的一位,当然也是最才华横溢的一位正是爱因斯坦本人。经常有一些弦理论朋友在我耳边鼓吹说弦理论将在未来半个世纪中统治物理,其中爱德华· 魏廷就这么说过。我想把这话纠正一下,把它变成:弦理论将像克莱因一卡鲁扎理论(是另一种被弦理论基于其上的异想天开的理论,在过去50 年中一直统治着粒子物理)那样来统治未来50 年的物理,也就是说,它根本不能统治物理。
    
    9 .里查德.费曼(RichardFeynman )
    
    里查德· 费曼是加州理工学院物理系教授,他被看作是现代粒子物理和量子场论大量理论基础的奠基人,他因在量子电动力学方面的工作曾获得诺贝尔奖金。作为近代基础物理的元老之一,他对超弦理论的怀疑态度尤其中肯。他于1988 年初去世。
    
    几年以前斯蒂芬· 霍金说他认为理论物理的终结已为时不远。我想他是指近年来试图把所有物理学分支统一到一个简单的概括性体系的工作取得了进展。这是一个引起争议的话题。对于这样花费毕生的时间以试图统一物理学某些方面的作法,您是如何看待的呢?
    
    我花了毕生的时间来达到这步,人们认为答案就在眼前,但一次又一次地失败了。爱丁顿认为靠电子和量子力学的理论,任何事情将变得简单而可以猜测,但正因为简单才会猜错。爱因斯坦认为他就要得到一个近在眼前的统一理论,但他不知道原子核的内部情况,自然无法预测。今天,仍有大量的事物没有弄清,这一点却未被完全地意识到,人们总认为他们离答案很近了,但我不这么认为。
    
    您认为我们现在已经能设想自然界在最深的层次上是统一的吗?就是说,简单的数学表述就可以概括所有的客观实在吗?
    在我们的研究领域中,我们有权做任何我们想做的事。这仅是猜想。如果你猜想任何事物可以被概括为几个定律,你可以这样去试试。我们用不着怕什么,因为,如果什么地方猜错了,我们可以用实验检验,实验会告诉我们这是不对的。所以我们可以尝试任何我们想做的。作这样的猜想并没有什么危险。不过在错误的方向上做了大量的工作也许会有心理上的危险,但通常这不是个对错问题。自然界是否有一个终极的,简单的,统一的,优美的形式是一个可以讨论的问题,我不想做什么定论。我想找到答案,虽然我也许活不了那么久。我会尽力探索自然界,但不会预先假定什么。无论自然界是可以用一个简单的公式概括,还是非常复杂,都毫无区别。任何人的猜测都可能通向他所希望的道路。
    
    检验这些最新思想实验中存在的问题之一是一些现象暗示只有在极高能量的情形下才可能产生统一。我认为我们就要达到加速器所能提供高能粒子能量的上限。而由于财政限制,很难预测下一代实验的情况。您认为这些原因会导致理论物理退化为哲学吗?
    
    也许理论物理正在退化,但我不知道将退化成什么。让我先说一些事:我年轻时曾注意到,许多圈内的老人不能很好地理解新思想,用这种那种方式抵制它们,他们非常迂腐地认为这些新思想是错的― 比如爱因斯坦不能接受量子力学。我现在也是老人了,有些新思想在我看来很古怪,好像它们是在错误的轨道上运行。现在我意识到其他老人在评价新思想时显得非常迂腐,因此我这么说也可能是非常迂腐的。但我仍然坚持这一点,因为我确实强烈地感到这些新思想是毫无意义的。尽管我知道这种观点的危险,但我没有办法。所以我认为所有关于超弦的东西是奇怪的并且是在错误的方向上,这可能使未来的历史学家感到可天。
    
    您不喜欢它的什么?
    
    我不喜欢他们不做任何计算。不喜欢他们不检验他们的思想,不喜欢任何与实验不符的东西,他们拼凑了一个借口― 说“好,它可能还是正确的。”例如,这个理论需要十个自由度。那么也许有一个方法可以禁锢其中的六个,是的,这在数学上是可能的,但为什么不是七个呢?在他们写出方程时,方程本身就应该定出有几个自由度被禁锢,而不是与实验相符合的愿望。换句话说,在超弦理论中没有任何理由不是十个自由度中的八个被禁锢,而使结果为两个自由度,尽管这可能与实验完全不符。所以它可能与实验不符的事实是非常薄弱的,它不能引出什么,还必须经过许多时间来完善。它看上去不怎么好。
    
    这是研究风格的问题,还是这些人想要做的事情本身的问题。
    
    不,我不知道你可否把它称作研究风格。它是把你的想法与实验比较并确定你的理论是否足够精确的问题。数学上它当然是精确的,但对那些做这项工作的个人来说数学太困难了,他们没有得出严格的结论,而仅仅是猜测。
    
    您似乎在说他们做这件事的方法太不严谨了。
    
    不,这不是不严谨,而是这事太困难了。所以他们无法做一个精确的预言― 不是因为粗心,而是力所不及。然而,他们却不断地说这是一个前景很好的理论尽管事实上他们不得不掺入许多猜测。现在可能是十个自由度中的六个被禁锢,也可能是别的情况。例如,这个理论中有大量的粒子,大大超过了我们知道的数目。所
    
    以我们说,好吧,那些没看到的粒子或许有一个巨大的质量― 所谓的普朗克质量― 从而使我们看不到它们。而那些我们可以看到的粒子没有这么大的质量。但为什么是这样而不是别的呢?答案应该包含在他们写下的理论的结论中。但他们无法显示这点。换句话说,它们没有与实验的真正比较。另外,我们看到的粒子都有一个质量,但这质量的上限比普朗克质量小得多― 这是当前实验的限制。而对如何验证这个更大量级质量的情况我们还一无所知。
    最后,虽然人们说没有实验为我们提供线索,但这是不真实的。我们有24 个或更多个― 我不知道具体数目― 与质量有关的神秘的数。为什么p 子质量与电子相比恰好是206 或不论是什么数,为什么许多不同粒子如夸克的质量正好是它们现在的值?所有这些和其他一些类似的数― 大约有两打左右― 在弦理论中毫无解释― 绝对没有!目前,在我知道的任何理论体系中,还没有一个想法能够就这些质量为什么是现在这个样子提供一点线索。
    所以我们已收集到大量的实验事实,只是还没有为它们提供一个合理理论的想象力。这应该是我们的起步之处,也是我们真正的问题所在,因为我们有实验数据来检验,利用这些数据,我们可以摒弃任何能够轻易地拼凑起来的理论。迄今为止,仍然没有一个好的理论。当你看到这些数字时,它们绝对是随机且奇怪的,好像没有任何规律。这对理论物理是一个难题,这些超弦理论根本没有解释它。
    
    我对这些项目的印象是它们建立于广泛的概念之上。在这种情况下,有一些简单优美的数学概念能把每一件事联系到一起,但这只有在一个可能永远无法观测的体系中才是明显的。以后人们只是考虑理论的能量下限,试图拟合这些数据,这是非常琐碎和技术性的。您认为这种哲学方法― 万物之中存在某些普适的基本原理的想法― 是一个好的激励物理学家的方式吗?很显然它确实激励了一些物理学家,但它可能使我们在研究物理的方法上误入歧途。
    
    我已经回答过这个问题了,即你可以做任何想做的事。唯一的危险是所有的人在做同样的事!也许有一些奇妙的统一原理,事实上,他们猜测的可能是对的,如果我们能显示它就好了。但也存在别的可能,认为应该存在某种统一的表述并没有提出是哪种统一。有无穷多的可能,任何一种都可能正确,或者一种也不可能,我们必须去探索,在尽可能多的方向上尝试。
    
    对以弦而不是以粒子作为基本对象的思想您有何看法?您不认为这种思想有什么吸引人的特点吗?
    
    没怎么特别觉得如此。问题不在于这种或那种想法或是谁被这种想法所吸引― 问题在于要有大量不同的想法,并且把它们引向通过观测可以摆脱它们的道路。我的一个朋友― 我们都是麻省理工学院的本科生― 曾对我说“我认为我懂得了理论物理中的问题就是尽快证明你自己错了!”现在弦理论学家们做的不是这样,因为他们不深究他们的方程却说“噢,也许它就是禁锢了十个自由度中的六个而剩下四个。”而不证明它禁锢了六个或者不考虑为什么不禁锢七个。由于计算的困难他们没有用实验尽量地检验他们的想法是否正确。这意味着他们毫无根据而我不必在这儿浪费太多的精力。
    
    许多从事超弦理论工作的人相信研究这些理论的一个主要原因是它可以处理无限即发散,这些已困扰基础物理学几十年的问题。如果这些理论一劳永逸地解决了无限的这个问题,我想您也许会乐于接受这些理论。
    
    我们欢迎或不欢迎什么取决于它是否符合自然界现象。当然,如果超弦确实解决了无限这个问题,那是令人高兴的。然而我的感觉是― 我可能错了― 条条大路通罗马,我认为不止一条路可以避免无限。我认为理论不存在无限的说法并不只将我们引导到这个弦理论上来,它应该将我们引向各个方向,因为人的想象力是很丰富的,会找到许多其他摆脱无限的方法,任何方向都可能是正确的。一个理论能避开无限对我来说不是使我相信它是独一无二的一个充分理由。这是我的观点,也许不对,如我所说― 我是一个老人。也许这些年轻人比我更能理解没有别的路可走。也许研究得深入一点我也可以明白为什么只有这个方向。然而,我没有看到这一点。
    
    但是无限是那么难以避免以致于如果超弦理论真的证明是有限的,这会成为一个人们相信这个理论令人信服的原因。
    
    如果它与实验符合,那么会的。但是他们说的是,“设想我们认为无法避免无限而突然发现有一个方法可以做到,但无法得出结论。既然理论是令人相信的,它必然是正确的理论。”所以你坐在那儿说“那么,你看,你无法证明它不对。”这我明白。你已经给我解释了所有这些年轻人说的话以及我不理解时他们会说些什么。他们不是在推导,他们仅仅是在表示因为这是他们能构造的唯一模型;因为没法证明它是错的,所以就是对的。好,这可能是他们的动力。他们也许是对的,虽然我不这么看!
    
    假如我们回顾您发展量子电动力学的年代,无限当然是一个难题。从某种意义上说问题的解决是因为您设法把这些无穷大归置起来并放到一边而克服了困难,如果我能这么说的话。
    
    准确地说是的,这是完全符合事实的。
    
    所以无限这个难题困扰了量子场论足足一代人的时间。您认为不同粒子相互作用的基本理论仍然会包含无限吗?或者您认为狄拉克说他不相信任何包含无限的理论是正确的吗?
    好,很显然地,在观测中不存在无限― 电子的质量是有限的。当我们以传统的眼光考察电动力学并不加任何修正时,我们写出方程计算电子的质量却发现它是无穷大。然后我们不得不玩一个障眼法,说这不是我们预想的计算质量的方法。我们被想象成做东拼西凑的工作,这些所谓“重整”,或“重组”的规则会得出所有结果是有限值并与实验符合的理论。这似乎就是实际情况,但我们不知道这个重组形式是否在数学上自洽。非常有趣的是,这些年来我们从未证明过一种或其他方法是否自洽。但目前假定这个方法是自洽的,那么我们就有了一个“写出错误方程”的数学结构,即,当你得到无限时,通过做这个在1 947 年由三人发明的“减除”游戏,并取极限条件,它就成为一个“有限”的理论,而这就是答案。这是一个数学结构,尽管听起来很混乱。可能会有这么一天,某人仔细地用一种不同的方法计算出不含无穷大的方程组而得到相同的结果。我不是指发明一套新的物理学,而是通过重新组织一下做计算的表述使它不那么令人生畏。
    所以这种情况仅仅是数学技巧的问题。但也许是因为电动力学不是自洽的理论,如果是这种情况,从物理的角度看问题更严重得多。如果我们没有一个数学上自洽的理论,我们不得不更多地去了解自然界并且找到必要的对电动力学的修正。对这个分支我们的确有些线索。在一个类似的叫作量子色动力学的理论中(关于夸克和胶子,希望能解释质子等粒子性质的理论),我们可证明其在数学上是自洽的。它包含了无限,而无限可以用通常的方法避免。然而最终结果是数学上自洽的。因此不考虑无限而得到结果一定是可能的。所以我认为那些无限多少是技术性的问题。我们最初写出理论时就已经在错误地表述它了。
    
    当然,对无限的问题只有在不同作用统一的情形下才能得以解决的设想是很时髦的。
    是的,由于在量子色动力学情形下的解显然不包含无穷,并且假设
    
    能证明电动力学令人不满意而为了使之变得令人满意,它不得不成为一个相似理论的一部分。这意味着对所有相关的不同作用采用某种展开和对称扩充。这是暗示统一理论的思想之一,是一个有力的建议。我现在马上要说,我那时从未认为尝试避开无限会成为一种发现物理规律的方式,而我错了。在预测最佳方式时我已是经常出错的了。
    
    起初你问我关于这些超弦时,经验使我保持了礼貌。我不能肯定地说― 我就是不相信它。以前我相信一些理论不会很好,而它们证明是好的,所以我以前已经犯过错误了。
    
    当然,关于无限最困难的问题是有关引力的。在关于任何基本作用的统一的描述中,引力都将占有中心地位,这好像无法避免。对一些人来说引力在任何意义上涉及到粒子物理是很奇怪的,因为在原子的层次上,引力是非常微弱的。您认为有何捷径可以看出为什么引力与这些问题有关?
    
    我很惊奇听到你说认为引力可能不重要。这是物理定律之一。很明显,大质量的物质聚到一起时互相吸引。如果我们要得到一个关于物理世界的理论,然后却不能解释为何大量物质聚在一起,显然我们没有真实地描述世界!所以引力必须出现在任何我们设想的理论中。
    
    但您认为我们需要用引力对粒子物理作修正吗?
    
    修正什么方面?
    
    解决发散的问题。
    
    哦,我没有什么见解。它是可能的,但是我们需要引力的原因是因为引力存在着。我们必须有一个理论能解释我们所看到的。这就是为什么我们必须要有引力,而不管我们是否需要它来避开一些无限。另一个问题是引力理论是否必须是量子力学的理论,如与其他粒子有关的量子力学现象。这个世界不大可能是一半经典一半量子力学的。因此,例如,在同一时刻你不能看到精确轨道的位置和动量的事实― 这是我们从量子力学中得知的― 也应该适用于引力。我们不可能在用引力决定一个粒子的位置和动量时超过一定的精度,否则我们会陷入不自洽的境地。在试图修正引力理论使它变成量子理论时我们会遇到无限,就像我们在电动力学中所做的,但它们更难以避开,更为严重。我不知道引力对它们怎样起作用。它带来了大量的无限以外的其他问题。
    
    在量子场论中,有一个所谓“真空”的能态,任何物质都处在最低能级。而按照理论― 这个能量不是零。现在设想引力与任何形式的能量作用,所以也应该与真空能作用。因此,真空应该有一个质量― 等量的质量能― 并产生一个引力场。但是事实并非如此!电磁能产生的引力场在黑暗、沉寂、空无一物的真空中应该十分强大,相当明显,但事实上它却等于零!或者小得与我们从理论上的期望值完全不符。这个问题有时被称作宇宙常数问题。它暗示我们在引力理论的表述中忽略了什么东西。甚至有可能麻烦的根源― 无限― 来自于真空中引力与自身能量的作用。我们从一开始就错了,因为我们已知引力应与真空能作用的想法是错的。所以我认为我们第一件要弄明白的事是如何用公式表述引力理论,使引力不与真空能相互作用。或许我们需要重新表述场论使得从一开始就不存在真空能。换句话说,在引力量子化的问题中有某些神秘的东西必然从一开始就与理论的表述有关。
    
    还有一些概念性问题。如果您把量子力学应用于引力,那么在某种意义上您是把量子力学应用于时空。那么如果我们取整个时空的话,就会得到整个宇宙。近年来量子宇宙学很流行,它试图把量子力学应用于整个宇宙的某些简化模型上。您认为这些概念问题真的是基本的吗?或者它们仅是非主要的?换句话说,在我们对引力量子化取得进展前,我们真的必须理解我们意指的东西吗?比如说,量子力学的波函数?
    
    只有在我们取得进展后我们才能知道什么概念是理论上必需的,什
    
    么是不必要的。这是不易事先作判断的。
    
    许多在这方面工作的人赞成所谓的量子力学的“许多宇宙解释”。您如何评价这个解释?
    
    我不知道。你看在这个领域中我们比其他领域的人有更大的优势,因为我们可用实验来检验我们的想法。因此你怎么想除了在心理上有所不同外不会造成什么区别。如果你说“无限是不可能的,我一定要发现一个这样的新理论”,那么你就错定了;但是你设法发现一个新理论而且它与实验符合,即使那个使你考虑新理论的想法不正确。新理论与实验符合的事实很好,你确实发现了一些新的东西。这些预先的关于什么是哲学上自洽的,什么是必要的想法在心理上使你说“我不相信今天的理论,因为都是一派胡言,我必须去发现新的东西!”就像我年轻时说我不相信电子与自身的作用;我去寻找新的解释,并着手去做了;我没有找到合适的东西,而那本来是可能的。这不意味电子不能与自身作用肯定正确,只是心理上的推动力促使你去发现新理论。所以我不反驳也
    
    不与那些认为如何如何不可能又应该如何如何的人争论。我将设法找到一个有新的特点的理论,因为理论可能是正确的。对吗?我并不想钻牛角尖,对于人们关于事物必须是什么样的任何合理的想法,我都不想去阻挠,因为它促使人们考虑事物的运行机制。虽不一定正确,却能使人思考。
    
    所以您对这些事持实用主义的观点?
    
    是的,我想你可以称它实用主义,因为所有我感兴趣的只是设法实现一组与自然界行为相符合的规则;而非好高鹜远。我发现许多哲学上的讨论在心理上是有益的,但最后当你以历史的眼光回顾所说过的话,而且是以如此的魄力所说的这些话时,你发现它几乎总是― 在某种程度上― 废话!
    
    我确信许多人会同意这点!设想事情变得像乐观派相信的那样,在几年内这些超弦的思想被证明是沿着正确的方向而且您早些时候指出的困难也被解决了。那么理论物理会是什么样的一种状况?我们将有一个理论,在表面上,可以解决宇宙中发生的每一件事。但是您相信会吗?您认为一个确定了客观世界的基本单元的理论原则上能解决每一件事吗?如生命与意识的起源问题。
    
    这问题太大了。你说了许多,我得从它们谈起。让我们首先从物理问题开始。完全可能有那么一天,也许就是用超弦的想法,我们可得到一个理论,它能解释我们观察到的一切。通过一个理论假设下的完整的数学分析,我们可以解释p 子与电子的质量比恰好是观测到的数值,其他的也是如此。这个理论正确地预言了自然界的所有方面,也许,理论自身也包含了对宇宙来源的最佳描述。对吧?现在这些问题是基本理论的全部。在真实世界中,海浪拍打海滩,有暴风雨,闪电,风,噪音等等。对这些即使我们知道了所有物理定律也不能直接分析。事实上我们已掌握了足够多的物理定律,在原则上应该能够分析海浪拍击海岸,闪电,以及任何其他事物。但是风、水等的相互作用的细节是复杂的― 我们很难精确地分析它们。
    
    仅仅是因为复杂,还是有什么新的基本问题?
    
    很明显,弄清楚所有这些现象并不需要我们刚才谈论的高深物理。仅凭量子力学和原子一类的定律,而无需涉及原子核就足以解释天气了― 尽管由于它的复杂性我们事实上没能解释天气。我经常用一个象棋
    
    的例子来打比方:一个人可以学会下象棋的所有规则,但不一定能下得好。一个人可以学会物理学所有的规律,事实上,在地球的正常现象的范围内,我们已掌握得足够精确。在一般情况下,我们够了解它们的了。但这不意味着我们能解释所有的事情。自然现象是如此的繁复,我们不能把它们分析得很透彻。现在我相信生命起源问题是这些复杂现象中的一个。物理学已帮助我们了解了分子的行为。我们已取得了这一进展。我们在认识基本定律方面所作的尝试与宇宙的历史以及弄清每一个基本规则的最终结果有着更为密切的关系。物理学当前的局面就好像我们已经懂得象棋,而只是不知道其中的一两个规则。但在这物质运动的棋盘上,这一两个规则没有起太多的作用,不知道它们,我们也能处理得很好。我要说,关于生命、意识以及诸如此类的现象,情况正是如此。这些问题采用什么方法解决以及它们怎样从哲学上解释是一个细致的问题,但它不必等物理学家完全掌握基本定律。我们已经掌握了那些在地球表面的一定条件下产生生命的原子所遵循的定律。
    
    但是当然会有一些人坚持认为当考虑足够复杂的系统时,新的原理就出现了― 这些原理本身的运行相当简单,但它们原则上并未包含于基本的内在的物理中。
    
    除了最后一句话以外,你说的都是对的。人们可以那么说,但我看不到任何理由相信它。很自然当事情变复杂时我们应该用新的原理帮助我们分析。例如,在象棋中,把棋子走到棋盘中央可增强它们的威力。这是一个没有明显地写在象棋规则中的原则,但是可以以间接的方式从象棋规则方面来理解。这个原理显然只是得自象棋规则而非别的什么。是的,事实上有许多奇妙的原则,化合价思想,声音,压力和许多其他组成有机体的原则,可帮助理解复杂情况。但是说它们不包含于基本定律中是一个误解。基本定律适用于任何事物。这只是一个寻找方便的方法来分析复杂系统的问题。
    
    是的,我不想说这些新原理可能与基本定律不一致,只是说基本定律不能囊括这些原理。
    
    我不理解这是什么意思。
    
    是这样的,例如,这些原理可能包括了约束条件和系统的真实状态的细节,而这在基本定律中无法得到表达。
    我不知道,我不这么看。你可以作许多类比,比如计算机的分析。
    
    你发现如果你有某种基本的元件,如与非门,就可以把它们装到一起制成计算机,但另外,为了了解计算机,引入一些概念如中央处理器,存贮器部件和其他等等,这是一个很好的想法。虽然这些部件能用与非门做成仍是事实,但拥有那些高级原理更方便。对于风,不必考虑在基本定律支配下每个分子的确切运动,更实用的是知道何时大量分子大体上沿同一方向运动。我们可以用平均速度等来表示它,并得到风的概念,这并没有以明显的方式存在于定律中。“风”这个词不在基本定律中,但基本定律包含了风的概念。这就是我的看法。
    
    那么我所想的问题是物理与宇宙的关系,虽然我们对宇宙怎样在大爆炸中膨胀知道得很清楚,但基本物理定律中似乎没有考虑宇宙是如何开始的,所以必须给予一定的初始条件。您认为我们可以把宇宙看作完全可以由物理来解释的吗?还是需要什么别的原理。
    
    这是一个非常有趣的问题,因为迄今为止物理学总是具有下面的特点:以一定的形式给出一组定律,这种形式就是如果你给出一定的条件就可以预言下一步会发生的事。换句话说,如果在这儿放3 个一种形式的原子,而在那儿放5 个另一种形式的原子,就能预言下一步会发生什么。事实上那些定律有不依赖绝对时间的性质,任何时候的定律都一样。迄今为止,物理学从未有过关于这些定律如何开始起作用的历史问题。不存在演变的问题。例如牛顿定律,包括平方反比的万有引力定律,从未说过你何时做测量而定律怎样及时地起作用。对于电动力学、量子力学等定律也一样。可以说它们在时间上是定域的;你可以在任何时候应用这些定律。因此,它们不能应用于宇宙学,因为宇宙学必须包括事情如何开始等内容,这样才能明白其规律。
    现在可能的情况是物理学中这类定律可能不完备的。或许定律应绝对地随着时间改变。例如引力随时间变化而且引力的大小与某一时刻距离的平方成反比。换句话说,有可能在将来我们可以对每件事了解得更深入一些,物理学可能由于加上关于独立于物理定律的初始状况的内容而完备起来。
    
    所以您不同意约翰· 惠勒(John Whee } e : )的关于物理定律能解释宇宙的产生的思想吧?您认为我们还需要别的东西吗?
    
    你必须仔细对待约翰· 惠勒所说的。因为我不清楚他是在说物理定律应该如此还是就是如此。现在物理定律做不到。我相信就是惠勒也会同意现在已知的物理定律不能告诉我们宇宙如何开始― 这已由它们写下来的形式决定了。我了解惠勒,他可能是说当物理定律被全部掌握时
    
    会解释这些的。这是相当可能的。我也会这么说,也许未来完备形式的物理定律将不会独立于时间,但却能描绘出宇宙的整个历史。而不需要什么外部先决条件― 如宇宙万物如何开始。但这不是现在的情况。
    
    那么您怎样看待物理定律?您是否像柏拉图那样认为它们是一种独立于宇宙而凭自身抽象地存在的?
    
    你指现在还是将来?
    
    随便。
    
    我们说说现在,好吗?
    
    好吧。
    
    存在性问题是一个非常有趣而又困难的问题。在你做可以简单地计算出假设下的结果的数学时,例如当你把整数的立方相加时你会发现一一当然,这只是一个小命题― 一件奇怪的事。1 的立方是1 ; 2 的立方是2 乘2 乘2 ,得到8 ; 3 的立方是3 乘3 乘3 ,得到27 。求和,1 加8 加27 ,得到36 。在这停下来,36 是另一个数6 的平方,而6 也正好是同样这三个数1 加2 加3 的和。我们再试试别的数比如5 , 1 加2 加3 加4 加5 ,求平方,就恰好得到这五个数的立方和,对吧。这个我刚才讲的事实,可能你以前不知道,你可能会问,它在哪儿,它是什么,它放在什么地方,它有什么样的现实存在?而你发现了它。当你发现这些事时,你感到在你发现它们以前它们是存在的。所以你认为它们以某种方式在什么地方存在着,但实际上这些事实原来不存在,这只是一种感觉。这就是人,我们正在从心理上努力要理解的人。我们发现所有这些奇妙的东西,例如,贝塞尔(Bessel )函数和它们的相互关系,富立叶(Four 1 er ) 交换,它们就是真实地存在的,我们仅仅是发现了它们。
    但是,就物理学来说我们有双重困难。我们碰见这些数学的相互关系,而它们是应用于宇宙的,所以它们在何处的问题更加令人困惑。就纯数学来说毫无疑问这些贝塞尔函数等等根本不存在,它们必须被发现,但是这些关系至少在我们发现它们前存在。就物理来说因为定律是应用于物理世界和物理研究的,所以更难说它们存在于何处。但它们或许比数学定律更接近于现实。这些是我无法回答的哲学问题,你可以不一定回答这些问题而做大量物理研究,但想想它们很有趣。当然,有一个时期人们通常相信上帝解释了宇宙,现在看上去这些
    
    物理定律几乎扮演了上帝的角色― 它们无所不在,无所不知。
    
    刚好相反。上帝总是在为了解释神秘的事情时才被发明的。上帝总是在解释那些你不理解的事时被发明的。现在当你最终发现某些事物如何运行时,你就得到了一些从上帝那儿拿走的定律;你不再需要他了。但对其他神秘的事你还需要上帝。所以你认为他创造了宇宙,因为我们还没有了解宇宙,你需要他来解释那些你相信定律无法解释的事。比如意识,或者为什么你有一定的寿命― 生命与死亡― 等等诸如此类的事情。上帝总是与那些你不理解的事联系在一起,因此我认为这些定律不能与上帝同等看待,因为定律已被我们理解。
    
    但是它们看来确实非常有力并且似乎超越了物理上的宇宙。
    
    不,物理上的宇宙遵循它们。我不明白你说的超越指什么?
    
    如您所说,如果物理上的宇宙的诞生可以用这些定律来解释,这些定律必定在宇宙开始以前就在某种意义上存在了。
    
    但我们仍然没有掌握这些定律。你是在谈论物理定律描述了事物如何开始这一假想情况吗?
    
    是的。
    
    那么我们到达那一步时,我会与你讨论相互联系的哲学,没有看到它们我无法回答。
    
    但是,你相信有这样的定律吗?
    
    我不知道。
    
    那么,您认为我们在朝着某些存在的定律迈进,而我们现存的理论只是它们的某种近似吗?
    
    是的,当然。我有一种感觉我正在发现那些已经在那少L 的定律,与数学家发现他认为已经在那少L 的定律时相类似。但他知道他的定律没有物理地位。我知道我的定律对预测宇宙如何运行是有用的。但是,我仍然不能肯定它们在哪儿,这是我不必回答的问题。不回答它我也可以顺利地研究物理。这不意味着我不思考它。因为你看到,我做了类比。我
    
    发现这非常轻松,愉快,有趣,但不是很重要的。
    
    10 .斯蒂芬.温伯格(Stevenweinberg )
    
    斯蒂芬· 温伯格是德克萨斯州立大学物理系教授。他的工作涉及粒子物理、量子场论、引力理论和宇宙学学科的许多方面,并且都作出了非常重要的贡献。他曾因在弱― 电统一方面的卓越成就,而荣获诺贝尔奖金。现在温伯格正在从事超弦理论的研究工作,是该理论热情而雄辩的支持者。
    
    大约在一百年以前,人们普遍认为物理学已经发展到了尽头,也就是说,关于整个宇宙的完整理论马上就可以大功告成了。然而,我们所谓的新物理又向我们展示出,离那一步还很远。可是我们似乎的确有这种感觉:我们再次面临着那种包罗万象的理论的可能性。温伯格先生,您认为这又是一次水中月、镜中花似的感觉呢,还是我们有可能把那个支配整个世界的机制用单一的表述或原理写出来呢?
    
    我想我们这些物理学家们已经学得谦虚多了。我们的目标是要把人们对整个世界的观点统一起来,在这个图像中,至少在比较简单的原则的观念上,万事万物都遵从几条简单的基本定律,(虽然这几条基本定律并不会帮助我们去理解我们周围的花草树木和我们人类自身)。但是,我们在做这项工作的时候确实是有困难的,我们也明了困难之所在。例如,像我们知道的那样,把引力那样不同的东西纳入到像核作用与电磁作用那样机制中去就是非常困难的。在过去的十年间,在能量可以接近的范围内,基本粒子相互作用的大统一理论已有了不少进展,只是引力还没能被纳入这一机制中去。要跨出这最后的一步,现在看来仍然是困难重重的。
    
    那么为了要把引力并入大统一理论中去,现在是不是又有些什么新的方案呢?
    
    噢,如果你是在几年前问起这个问题的话,我的回答不免令人遗憾。现在倒真有那么一个方案,它大约从1 974 年起被称为弦理论,后来又采用了超弦理论这个专业名词。它最初大约是在1 968 年,为了解释核内作用在核子上,把核子结合在一起的强相互作用力而提出来的。但是,弦理论在一开始解释强相互作用时,并不成功。它面临的困难之一是它预言了一种零质量的粒子,而根据我们对核结构的认识,这类粒子显然不能和任何已知的粒子相对应。1974 年,约翰· 施瓦茨和乔尔· 谢尔克建议,应把超弦理论重新理解为包括所有相互作用的统一的理论,而不是
    
    只包括强相互作用的理论。这样引力就被统一到这一理论中去了,理论单独考虑强相互作用时的困难也解决了,其中所预言的零质量粒子和引力辐射量子― 引力子相对应。
    
    在弦理论一度萧条的年代里,它的发展情况又如何呢?
    
    在1 974 年以后的几年中,弦理论并没引起人们多大的兴趣和关注。至少我本人是这样的。我们这些粒子物理学家那时正满怀希望地为发展粒子物理的标准模型而努力工作着,那是一种把弱作用和电磁作用统一起来的机制― 我们希望强作用也统一到其中去― 它非常成功,并且被一系列非常精彩的实验很好地证实。当时,人们想把引力搁置在一边,因为我们实在看不出把引力也纳入这一模型的希望之所在。那些早期从事所谓弦理论研究的物理学家们继续在这个领域做着工作,但他们的工作却被其他的人给大大地忽视了。只是在最近的几年间,他们的工作才重新引起粒子物理学家们的广泛注意,原因一方面来自我们在其他尝试中的纷纷失败,另一方面是由于数学有了惊人的发展。
    人们发现这些理论可以是数学自洽而这在过去被认为是不可能的。并且这种自洽性只有在有限的几种理论中才能得以实现,因此弦理论具有一种物理学家们一直寻求的属性― 唯一性,弦理论是很严格的。和过去10 年间我们所研究的其他理论相比,弦理论并没有无穷多的花样,让我们永无止境地做工作。也许只有一个或很少几个类型,通过做好这几个类型的工作,我们认为就有可能向前迈一大步了。
    
    在最受推崇的弦理论中,弦实际上存在于一个十维空间,是吗?
    
    可以这么说。自然,这也是弦理论没有马上被接受的主要原因之一。它的数学形式很漂亮,所有的东西也都组合得很妙。但这只限于此特定情况:它引入26 维时空,后来利用进一步的限制,把它约化在较为真实的十维时空上了,其中空间维数是九,时间维数是一。当然,人们还没有观察到它的存在。人们通过实验测量和检验,最有把握和信心的是我们的时空是四维的,而不是其他维数。因此,当这些理论在1 974 年第一次被提出时,立刻被认为是无稽之谈,根本没有引起人们的重视。若要我们去欣然接受一个在十维时空中描述的引力理论,那真是难以想象,因为我们所要的是存在于四维时空中的理论。可是在过去的十年间,物理学家们又重新发掘了这样一个早在1 921 年就由于西奥多· 卡鲁扎提出的设想。他认为我们很可能生活在更高维数的时空中。只是除了现有的四维外,其他的维数却被卷缩成了很小的闭合圈,以致于正常情况下,我们观察不到它们的存在。这个观点首先由卡鲁扎提出,曾受到爱因斯
    
    坦的支持,当时是为了在更高维数的时空中以纯引力作用的形式来解释其他的自然力,比如电磁力。也是由于这个原因,在80 年代的初期这种观点又再一次地被提出来了,并且在最近几年成了理论物理学家的热门课题。我认为正是卡鲁扎和克莱因及其他人的那些老想法的“复活”为弦理论的创立奠定了基础,消除了人们对建立在十维时空中的弦理论的1 不疑。
    现在的情况是这样的,虽然弦理论是建立在十维时空中的,但是其中的六维要通过所谓的自发卷缩消失在我们的视野之外。力学作用使它们收缩得如此紧密,以致于我们根本没有注意到它们的存在,并且轻率地认为我们的时空就是由三维空间和一维时间组成的。当然,有可能在宇宙发展的早期阶段,摆在生活于那个时候的物理学家们(并不是那时真有物理学家)面前的是九维空间加上一维时间。
    
    一根弦在低能时怎样表现得像粒子呢?从现象上看它们有很多的不
    
    一根弦(比如一根首尾相接的弦)的存在状态,总可以分解为无穷多个振动模式的组合。每一个振动模式的表现行为如同一个某一种类的粒子。当振动的模式越来越高时,会有越来越多的波长满足环形弦的振动条件,表现出来的粒子也就具有越来越大的质量,弦的那些最低振动模式― 对应质量小的那些粒子― 就是我们在实验室所看到的那些模式,也就是普通粒子。其他的粒子― 更高的模式― 在可以预见的将来还可能是无法观察到的。
    
    有人认为:在低能时,弦像刚体那样运动,所以表现得像粒子那样;但是,随着能量的增加,弦开始摆动。因此,表现的行为是不同的。这种认识对不对呢?
    
    粗略地可以这么说。那只是一个粗略的解释。假设我们来考虑两个所熟悉的粒子的碰撞,在弦理论中它们对应着弦低模式振动。当我们通过计算它们所交换的引力子来计算它们的相互作用力时,有可能得出作用力是无穷大的结论来。因为那是我们在量子水平处理引力问题时所常遇到的问题。虽然我们在实验室研究的粒子对应着弦的低模式振动,但是在它们相互作用交换引力子时,它们是在所有模式上都进行引力子交换的,而且这无穷多模式之和给出一个有限的结果。这是十分精彩的一一简直是奇迹般地― 并且在数学上是十分可信的。
    超弦理论是怎样描述电子和中微子的差异的呢?例如,电子是电荷
    
    为一e 的费米子,中微子是不带电的费米子,超弦理论是怎样区别它们的不同的呢?
    
    就某种意义来讲,这不是个合适的问题。我们对于电子、中微子、光子等这些我们都很熟悉的粒子所进行的描述,在我们通常研究的能量范围上,并不要做出什么改动。它大体上采用“标准模型”的那一套方法。在标准模型理论中,电子和中微子是同一族粒子中的不同成员。的确,电子带有电荷,可以直接通过电磁场实现相互作用;而中微子没有电荷,没有直接参与电磁相互作用的性质。但是,中微子参与除电磁作用外的其他的相互作用。标准模型很成功、很漂亮,具有对称性,可以说是很优美了。只是,关系到电子和中微子的、电磁作用和其他相互作用的对称性时却都遭到了失败。
    这些都是老问题了,超弦理论的出现似乎也不能对此有所改变。现在的问题是,超弦理论能否把标准模型作为它在低能下的近似呢?在低能近似下,弦被视为点粒子。根据不同的振动模式,它表现为电子、中微子或其他粒子。并且我们还必须搞清楚标准模型里的许多粒子(包括中微子、电子)在弦理论中是否可以被一一描述出来。这是最重要的问题。
    具体地说,比如对于为什么电子具有电荷,而中微子不具有,超弦理论和标准模型的回答应该是一样的。这是因为,超弦理论并不是要取代标准模型,而是要从其低能近似中得到它,这时它的自由端被连结起来了。
    遗憾的是,理论工作者还没能成功地证明出标准模型就是超弦理论在低能下的近似。他们的工作也令人非常遗憾地告一段落了。在低能近似下,超弦理论本身有一些结果看起来很像标准模型的结论,但至今还没有人能够准确地给出标准模型就是超弦理论的低能近似的严格证明。我觉得您提的问题是很令人尴尬的。这就如同问:“如何在广义相对论时空中设计出吊桥的形状?”我们可以利用牛顿力学设计出吊桥来,而广义相对论的优点在于牛顿力学是它的一个近似,这种近似只有在地球表面的一个范围内才是适合的。因此,在发现广义相对论规律后,我不会再重新审视吊桥的设计问题。同样,在弦理论取得进展以后,我也不愿再重新研究标准模型的成功之处。
    
    超弦的荷是遍布整个弦呢还是具有局域性?
    
    电荷可有多种分布方式,最初认为电荷只有在敞开的弦上才可以存在。电荷、电磁场及其他和弱相互作用有关的成分,都附着于弦的端点上。您可以认为是弦的端点携带着这些成分。
    
    以上是最初设想。现在又发现了许多更精细的机制:电荷和由十维时空紧缩到四维时空的方法有关。
    
    还有一个与此相关的问题,如何从超弦理论得出夸克、电子、中微子等这许多不同类型的粒子呢?
    
    在超弦理论中,弦在所有这些额外维数上振动着,这就导致了众多的振动模式。是这些额外的维数(或称额外的物理量)产生了许多不同的模式。事实上,这也是弦理论特别鼓舞人心的地方。在接下去的工作中,人们很自然地发现了粒子按代出现的事实。不仅有包括电子和轻夸克在内的最低级代,还有含p 子和奇异夸克在内的第二代和其他的代。事实上,大多数这类模型都有很多代。在一篇早期的文章中,有人试图从弦理论出发建立起一个特定的低能理论,但他们还是得到了一百多个代。这与弦可以在其他维数上振动这一事实有关。
    
    是不是可以认为所有已经观测到的粒子和弦振动的最低频模式相对
    
    应?
    
    是的,它们都是些最低频率模式。其次的模式将因为太重而无法观察到。这样的低频模式有很多,按照弦理论的特征能量尺度,所有这些低频模式对应的粒子本质上无质量。
    
    建立在弦理论上的质量的自然尺度大约是101 " Gev ,我们讨论的所有粒子从本质上讲对应弦的零质量模式。
    
    然而,这些在额外维数中的振动并没涉及到普朗克质量,是不是?对的。有无穷多的振动涉及到普朗克质量,有有限个但数目很大的振动没有涉及到普朗克质量。这些没涉及到普朗克质量的被视为零质量粒子。实际上并不是绝对的零质量,因为有很多的精细效应使粒子获得一些质量。但是,由这种方式获得的质量与普朗克质量(101 " Gev )相比,实在太小。也许只有几百个这种轻的粒子,而那些质量特别大的粒子的数目则可以延伸到无穷大。
    
    有人认为有496 种荷的存在方式。
    
    那是格林一施瓦茨的著名理论,现在它已经过时了。在这理论中,弦是打开的,弦的两端是自由的,荷由弦端携带。它们自由组合的种数是496 。顺着这条途径再建立起自洽的量子理论,这个数目就只能是496 了。
    
    自从施瓦茨和格林开辟这项工作以来,又有了一些其他的方法来建立起比较满意的理论,并可以导致其他的荷的数目。这个数目可以不是496 ,但是我们认为,不同荷的取值的种数可能很大但却有限。
    
    怎么看待这些电荷呢?
    
    一如我的朋友阿卜杜斯· 萨拉姆所喜欢说的那样,我们应该期望自然界在原理上是简单的,但这并不意味着它在结构上也是简单的。弦理论就是建立在一套关于世界存在方式的简单的假设上的,接下去又用了相当复杂的数学,复杂的数学又导致了复杂的图像。人们就可以对这些复杂的图像在人类所能达到的能量范围内进行观测了。我不认为我们会因世界太复杂或者说弦理论预言它会很复杂而感到失望,关键并不在于结果有多复杂,而是前提有多复杂。前提,最基本的前提是十分简单的,我认为这是弦理论如此优美,如此吸引人的最重要的因素。就像有人去做一碗很复杂的汤时,他没有在放进每一种用料时品尝一番,才能显示他对这汤是多么的喜欢。不管饭菜做得多么精美,食谱却从一开始就定好了,并且还非常的简单。例如,496 看起来似乎意味着很大很复杂的结构,但是在格林和施瓦茨的假定之下,也就只有这么一种可能,并且它还是在很简单的假设基础上,自然而然地推出来的。我一直所重视的是假设的简单性,而不是结果的简单性。
    超弦理论是否能给出质子衰变的一些具体信息?
    
    是的。有些超弦理论在低能极限下陷入困境,就是因为它们给出的质子衰变率大得使人吃惊。这是我们评价一个超弦理论成功与否的一个很重要标志。如果它预言的质子的衰变寿命和JI 介子相似,那么它的计算结果是显然不能被保留的。所以一个人要想研究一个超弦理论的低能形式,他总要先研究一番质子的衰变问题。但我并不是想说对所有的超弦理论这都是约定俗成的。我也并不认为超弦理论促使衰变发生或不发生,或促使它发生的太快或太慢。这是一个特定解下的细节问题。
    
    在早期的弦理论中,格林和施瓦茨论证了反常相消问题。能不能认
    
    为那时只有十维和26 维的理论看起来才有反常相消?
    
    这种认识是正确的。有人研究过不同的可能性,但我们不知道这些可能性来自不同的理论还是来自同一理论的不同的解呢。如果它们是一个理论的不同的解,我们还不知道到底是什么规律决定着哪一个解对应着我们所看到的真实世界。当然,与格林和施瓦茨的工作相比,我们已知道了更广泛的可能性,但我们还不知道它到底是多么的普遍。也许到
    
    头来,它们还是要归结为相同的东西。
    现在要讨论弦理论的最终思想还为时过早,因为至今还有太多的不确定性。例如,那些不同的解中哪些是真实的解,哪些理论又是独立的理论呢?
    
    现在能否澄清关于496 种荷的观点?它们又都是哪些类型的荷?
    
    在格林一施瓦茨图像中,这些荷主要是人为放进去的。因此它们和额外维数无关。它们只是由弦端点所带的荷。荷的数值是为了平衡将要引起的反常和理论自洽性的破坏而设定的。但是它们是人为放进去的。如同你只说了一句:“这个理论拥有这个数目的自由度。”现在,当我说荷是人为加入的时候,我的意思是说,你考虑携带各种荷的弦的不同的情况,便会自然地发现那些作用在荷上的力― 电磁力、弱作用力和其他的由这些荷产生的,并作用在这些荷上的力― 都是自然而然地出现的。但是这些荷是根据数学的自洽条件而用特别的方式引入的。
    
    看来理论似乎有所变化。因为人们已注意到建立理论的其他方法了。听说现在有人正准备建立四维的弦理论。
    
    是的,我也正想通过这个问题补充一下呢。我谈到过额外的六维自由度被卷缩起来了,但现在人们可以不需要通过这途径也能够解决同样的问题了。有人把理论建立在四维的基础上,只是其中增加了一些额外的变量。这些额外变量在有些情况下可以被认为是额外自由度的坐标架,但也可以不这样理解。事实上有些情况这样是做不到的。因此,如果你放弃了额外自由度的几何图像,并且只考虑过去的比较好的四维坐标再加上根据理论协变性的需要而加进去的额外交量,你就真正理解普遍可能性的内涵了。
    自治性条件,要求加入额外交量,并且决定了应是些什么变量。虽然我们不能知道起决定作用的到底是什么,但是对这些变量我们不能随心所欲地增减。我们不得不在四维空间中加上一些额外的自由度,而这种附加自由度的方法受到数学协变性的严格制约。我们还没有找到如何满足这些条件的一般方法,也就是还没有证得适合于任何情况的完全令人满意的通解。但这是目前研究的方向。起初的十维图像― 其中六维卷缩了― 只是被当作一个特殊的情况。
    
    如果不认为额外的变量和更高的维数相对应,有没有可能对它做出一些物理的解释?
    
    我想不能。由于数学的自治性,最后的理论必将发展成为它该具有的形式。只有在解决了理论的问题,并且对可能达到的能量范围作出了预言后,才能对它们作出物理上的解释。这是一个不能为实验直接验证的领域的物理。并且物理上的直觉也不能成为我们的指导性原则,因为对于这个尺度的物理,我们还没有任何直觉。这个理论还要靠数学上的自治性才能得以发展。我们希望理论的结果是:在可以接近的能量范围内,看起来像我们的真实世界。
    我想,我们建立在经验及实验基础上的物理思想,在此并不能帮上太大的忙。
    
    迈克· 格林(MikeGreen )声称,我们将不得不改变我们的时空概念,使它“弦化”。目前,弦理论是建立在经典背景上的。
    
    我想在这些理论中,时空并不是很重要。时空坐标不过只是众多自由度中的四维,为了理论的自治性,我们才不得不把这些自由度一起来考虑。是我们自身觉得时空重要,而赋予它特殊的几何重要性。看来,在这点上,我并不能代表大多数的弦理论家。因为大多数的弦理论家在试图建立一个具有漂亮几何基础的弦理论― 就像爱因斯坦在广义相对论中所建立的那样。他们有可能成功。但我还是以为他们有可能做了一个错误的类比。我们还不至于想要建立一个新的时空观,而只是不去强调时空的重要性。时空坐标只是描述理论所需要的十维― 或15 维或26 维或其他维― 的自由度中的四维而已。它的几何重要性稍次之,比出现在基本规则中的东西更次之。
    
    几年前,弦理论引起了一次不小的争论,因为它似乎很明确地有限。据我所知,它只是在被近似展开到比较低的几项情况下,才被证明是有限的。但是,过去的超引力理论也声称是个有限的理论,后来却被证明是无限的。超弦理论会不会出现同样的问题呢?
    
    我认为它们之间有不同之处。论证超引力理论为有限理论的论据来自对微扰论的最低级近似的证明。证据显示出微扰论的第一、二级近似不会出现,而无穷项可以出现。
    弦理论的证据是很不一样的。有一些不太严格或许是不能完全使人信服的证据,它认为弦理论对任意阶都应该是有限的。当你按微扰论解出理论的最低几级近似时,就会发现这些证据确实在起着作用。超引力理论的低级微扰论近似被认为是有限的,其原因在于它被特别限制在低级微扰上,这里的理由是普遍的,可以由微扰论的低级近似推导出来。因此,我认为两种情况是很不同的。
    
    对于超引力理论,我想只有极端的乐观主义者才会从微扰论的低级近似的有限中得出整个理论有限的结论来。因为这里低级近似的有限是有特殊的原因的。对于超弦理论,我们假定它有限这是个很有理由的猜想。如果它不是有限的,我倒会感到吃惊!
    
    您对人们就超弦理论所提出的批评有什么要解释的吗?
    
    我想一个人只能干他自己能干的事― 那是物理学的第一原则。你做力所能及的事才能做出成绩来。15 年来,理论家提出新的模型,实验家来验证它;实验家发现了新的现象,理论家来解释它。如果现在仍有像这样的理论和实验的愉快合作,我会很高兴的。不幸的是,我们过去有了这么多的成功,以致已到了一个尽头。新一代的加速器有可能为我们的工作带来新的转机。我们希望超导超级碰撞机、LEp ,甚至像丁evtron 对撞机之类的现存加速器运行工作,期待我们的工作能因此增添一些活力,那将是大有可为的。同时,我们只能做些自己可以做的事。
    
    现在,你不妨试试利用我们现存的加速器和设备,在我们所有知识的基础上,思考和设想可以做出成就的新途径。我很高兴地知道有人正在这样做,我祝愿他们成功。自标准模型后,我们还没有真正做出过什么成绩,这一段时间的成果并不多。
    
    另外还有一条途径,你不妨试试一次大的跳跃,从最基础的水平入手研究。在非常简单的基础上进行推理,以此研究世界上到底在发生着什么事情。把一个好的设想付诸实施是值得的。我认为弦理论就是个很好的设想,继续在这方面做工作是很有意义的。我并不支持每个人都去搞弦理论,也不支持都去搞唯象理论和低能物理,我认为人应该去干他能干的事。然而我确实认为超弦理论正在让我们的研究生们领略到很强的数学锻炼的“风味”。让他们去学习所有这些数学是很有好处的。但我也担心他们中有些人会不知道二介子是什么了。实际上,今年我在奥斯汀开了“基本粒子物理”这一门课。在课上,我从1897 年汤姆逊( J . J . Thomson )发现电子讲起,回顾了90 年来所有的艰难的实验和理论工作,一直讲到我们今天的认识和观点。
    因此,我很满意自己一心一意地投入到现象学的研究之中,并且力图从理论上抓住现象背后的本质,但我也认为跳过17 个能量量级到达普朗克尺度,做一次展望是很值得的,也许最后的答案就在这里面呢?! 超弦理论到底是不是个好理论要取决于由它可以推出的结论。弦理论是很漂亮的,很有前途的,它在不清楚如何由它推导出与引力有关的信息的情况下,就得到了一些事情的定性上的成果。但因此就不再去作研究
    
    它的尝试的作法是狂妄的。这种工作值得去做。
    由这些弦理论得出结论来是很困难的。去决定它的哪些预言实验家和天文学家应该着手去证实或推翻也是困难的。我们还不知道这个理论到底真正预言了哪些新作用力和新粒子,它们是由早期宇宙遗留下来的或新加速器可以产生的。有迹象表明,理论涉及到除强、弱、电磁作用之外的一种新的相互作用,它在基本粒子物理的普通尺度下也在发生着作用。但是,具体的细节还没有弄清楚。
    理论物理学家中又有一部分人,特别是年轻人,来到这一领域做工作。但是,数学上的困难肯定是很可怕的,并且可能像过去那样时时出现。初看起来,理论和我们的真实世界很相似,但当用它来解释真实的物理时,又可能出现不可克服的困难。然而在将来的几年中,要把这项工作做好,肯定是其乐无穷的。
    
    词汇
    
    以太(Aether ) :一种假想充满空间,在其间电磁波传播的介质,被相对论所否定。
    原子论(Atomi sm ) :公元前1 世纪古希腊哲学家德漠克利特( Democ : it 日s )和留基伯(Le 日c 1 pp 日s )提出的理论,认为物质由不可分的微小粒子组成。
    重子(Baryons ) :由三个夸克组成的重强子。
    玻色子(Bosons ) :具有零或自旋的基本单位偶数倍的内桌自旋的一类粒子的名称。
    手征性(Chi : al ity ) :有确定左、右手性的系统或物体的技术性术语。
    费米子(Fermions ) :具有自旋的基本单位奇数倍的内桌自旋的一类粒子的名称。
    费曼图(Feynmand 1 agrams ) :图解研究粒子作用的技巧。虽然很有物理启发性,但它们只是图解性地代表计算中的一些项,不能代表实际过程。
    规范理论(Ga 日getheory ) :力由具有某些抽象对称性质的场来描写的一种理论。
    胶子(G }日。ns ) :夸克间传播强相互作用的粒子或量子。大统一理论(GrandUn 1 fiedTheo : ies ) ( CUTs ) :尝试统一自然的四种基本力中三种:电磁、弱和强相互作用的理论。
    引力微子(Grav itinos ) :按照引力的超对称理论传播部分引力的假设粒子。
    引力子(Grav itons ) :引力场的粒子或量子,它在物体的粒子之间的传播可被视为引力的起源。
    强子归adrons ) :以强的核力相互作用的亚核粒子(通常很重)。轻子(Leptons ) :以弱的核力,而非强的核力相互作用的物体的粒子(通常很轻)。
    介子(Mesons ) :由夸克和反夸克组成的中间质量的强子。p 子(Muons ) :轻子的一种,与电子很相似,但是更重,也不稳定。中微子(Neut : 1 nos ) :零电荷,也许是零质量的轻子。作用很弱,极难探测到。
    宇称(Pa : ity ) :亚原子粒子的镜象反射特性。
    唯象学(phenomenol ogy ) :关于现象的研究。指分析实验数据而不太注重基础的理论。
    光子(photons ) :光和其他电磁波的粒子或量子,传播电磁力。正电子(Pos 1 trons ) :电子的反物质伙伴。正电子是有电子的相反质
    
    量的电子,带正电荷。
    普朗克常数(Planck ' sconstant ) :记为h ,马克斯· 普朗克引入的光子的能量与频率比的量,是一个普适常数。普朗克常数在量子理论中很常用,也在像内桌自旋的其他术语中出现(通常除以2 二)。等价原理(Pr 1 nc 1 p 1 eofequ 1 va 1 ence ) :爱因斯坦的,l 贯· l 生力与引力等价的原则,最简单的观察例子是所有物体在引力下下落速度一样。夸克(O 日arks ) :强子(核的粒子)的基本组成。三个夸克组成重子如质子,夸克对组成介子。
    简化主义(Red 日ct 1 on 1 sm ) :所有物理过程和系统最终能由它们的基本组成来理解的一种哲学观点。
    强相互作用(St r ong force ) :强子(核及其有关粒子)间的作用力。以现代观点来看,它起源于夸克间的力。
    超引力(Supergravity ) :引力被作为时空几何超对称描述一部分的理论。
    超对称(S 日pe : symmet : y ) :共同描写玻色子和费米子的一种抽象的几何对称性。超对称是现代多数量子引力尝试的基础,并且是超弦的重要组成部分。
    拓扑学仃。Po } ogy ) :处理线、曲面等怎样连接的一个数学分支。拓扑不是太注意几何(像物体的形状或大小),而是研究像线有多少个结,面有多少个洞这样的问题。
    W 和Z 粒子(WandZpart 1 c 1 es ) :传播弱相互作用的粒子。在1 983 年被发现,但理论预言很早就作出。
    弱相互作用(weakforce ) :自然中四种基本力的一种。虽然往常被强相互作用和电磁相互作用所掩盖,物体的所有粒子都受它的作用。最易观测的效应是核子的p 放射性。


>>> 追寻引力的量子理论 >>>
Copyright © 梦境天华站
粤ICP备08035557号
2001-2024 all rights reserved · mst1739
Email:mst1739@qq.com